Раздел 9. Дифференциальные уравнения (задачник)

Page 1

ФЕДЕРАЛЬНОЕ АГЕНТСТВО ПО ОБРАЗОВАНИЮ

Государственное образовательное учреждение высшего профессионального образования

"УФИМСКИЙ ГОСУДАРСТВЕННЫЙ НЕФТЯНОЙ ТЕХНИЧЕСКИЙ УНИВЕРСИТЕТ" (УГНТУ)

Кафедра математики

УЧЕБНО – МЕТОДИЧЕСКИЙ КОМПЛЕКС дисциплины «Математика» ________________________________________________________________________________

РАЗДЕЛ 9 «Дифференциальные уравнения»

Контрольно – измерительные материалы

Уфа • 2007


Ответственный редактор д. ф.-м. наук, проф. Р.Н. Бахтизин Редколлегия: АкмадиеваТ.Р., Аносова Е.П., Байрамгулова Р.С., Галиуллин М.М., Галиева Л.М., Галиакбарова Э.В., Гимаев Р.Г., Гудкова Е.В., Егорова Р.А., Жданова Т.Г., Зарипов Э.М., Зарипов Р.М., Исламгулова Г.Ф., Ковалева Э.А., Майский Р.А., Мухаметзянов И.З., Нагаева З.М., Савлучинская Н.М., Сахарова Л.А., Степанова М.Ф., Сокова И.А., Сулейманов И.Н., Умергалина Т.В., Фаткуллин Н.Ю., Хайбуллин Р.Я., Хакимов Д.К., Хакимова З.Р., Чернятьева М.Р., Юлдыбаев Л.Х., Шамшович В.Ф., Якубова Д.Ф., Якупов В.М., Янчушка А.П., Яфаров Ш.А. Рецензенты: Кафедра программирования и вычислительной математики Башкирского государственного педагогического университета. Заведующий кафедрой д. ф.-м. наук, профессор Р.М. Асадуллин. Кафедра вычислительной математики Башкирского государственного университета. Заведующий кафедрой д. ф.-м. наук, профессор Н.Д. Морозкин.

Учебно-методический комплекс дисциплины «Математика». Раздел 9 «Дифференциальные уравнения». Контрольно-измерительные материалы. – Уфа: Издательство УГНТУ, 2007. – 175 с. Содержит комплект заданий в тестовой форме различной сложности по всем темам раздела 9 «Дифференциальные уравнения», предназначенный для оценки знаний студентов. Разработан для студентов, обучающихся по всем формам обучения по направлениям подготовки и специальностям, реализуемым в УГНТУ.

© Уфимский государственный нефтяной технический университет, 2007


СОДЕРЖАНИЕ

1. Обыкновенные дифференциальные уравнения I порядка. Теория 2. Обыкновенные дифференциальные уравнения I порядка. Типы уравнений. Задачи 3. Дифференциальные уравнения с разделяющимися переменными 4. Однородные дифференциальные уравнения I порядка 5. Линейные дифференциальные уравнения I порядка. Уравнения Бернулли 6. Уравнения в полных дифференциалах 7. Обыкновенные дифференциальные уравнения n-ого порядка. Типы уравнений. Теория. 8. Обыкновенные дифференциальные уравнения n-ого порядка. Типы уравнений. Фундаментальная система решений. Задачи 9. Дифференциальные уравнения n-ого порядка, допускающие понижение порядка. Три частных случая 10. Линейные однородные дифференциальные уравнения n-ого порядка с постоянными коэффициентами 11. Линейные неоднородные дифференциальные уравнения n-ого порядка с постоянными коэффициентами со специальной правой частью 12. Линейные неоднородные дифференциальные уравнения n-ого порядка с постоянными коэффициентами. Метод вариации произвольных постоянных 13. Системы обыкновенных дифференциальных уравнений

4 14 28 36 44 51 60 76 94 105 111 121 136


Разработаны тестовые задания различной сложности (А – легкие; В – средние; С – трудные), которые предназначены для проверки знаний основных положений теории и базовых практических навыков по данному разделу дисциплины математика.

Система нумерации тестовых заданий

1

номер темы

2

порядковый номер

А

сложность

Наименование тем заданий контрольно – измерительных материалов (КИМ) по разделу: «ДИФФЕРЕНЦИАЛЬНЫЕ УРАВНЕНИЯ» 1. Обыкновенные дифференциальные уравнения I порядка. Теория. 2. Обыкновенные дифференциальные уравнения I порядка. Типы уравнений. Задачи. 3. Дифференциальные уравнения с разделяющимися переменными. 4. Однородные дифференциальные уравнения I порядка. 5. Линейные дифференциальные уравнения I порядка. Уравнения Бернулли. 6. Уравнения в полных дифференциалах. 7. Обыкновенные дифференциальные уравнения n-ого порядка. Типы уравнений. Теория. 8. Обыкновенные дифференциальные уравнения n-ого порядка. Типы уравнений. Фундаментальная система решений. Задачи. 9. Дифференциальные уравнения n-ого порядка, допускающие понижение порядка. Три частных случая. 10. Линейные однородные дифференциальные уравнения n-ого порядка с постоянными коэффициентами. 11. Линейные неоднородные дифференциальные уравнения n-ого порядка с постоянными коэффициентами со специальной правой частью. 12. Линейные неоднородные дифференциальные уравнения n-ого порядка с постоянными коэффициентами. Метод вариации произвольных постоянных. 13. Системы обыкновенных дифференциальных уравнений.


1. Обыкновенные дифференциальные уравнения I порядка. Типы уравнений. Теория Номер: 1.1.В Задача: Обыкновенное дифференциальное уравнение имеет вид Ответы: 1). y = f ′(x ) 2). y = f ′(x , c ) 3). f (x , y, c ) = 0

⎛ ⎝

4). f ⎜ x , t , ... ,

(

∂y ∂y ⎞ , ⎟=0 ∂x ∂t ⎠

)

5). f x , y, y ′, y ′′, ... , y (n ) = 0

Номер: 1.2.В Задача: Порядком дифференциального уравнения называется Ответы: 1). наивысший порядок производной, входящей в уравнение 2). число переменных, входящих в уравнение 3). порядок расположения переменной x и функции y 4). порядок расположения переменной x , функции y и ее производных 5). упорядоченная запись уравнения Номер: 1.3.В Задача: Какие из следующих уравнений будут обыкновенными дифференциальными уравнениями I порядка ( x - независимая переменная, y = y(x ) ): 1) y ′ = f (x ) 2) P(x , y )dx + Q(x , y )dy = 0 3) f (x ′, y ) = 0 4) f (x , y, y ′) = 0 5) df (x , y ) = 0 Ответы: 1). все 2). 1 3). 1,4 4). 1,2,4 5). 1,2,4,5 Номер: 1.4.В Задача: Необходимо правильно закончить следующее предложение. Обыкновенным дифференциальным уравнением называется уравнение, связывающее независимую переменную x , функцию y(x ) и … 2). константы c1 , c 2 , ..., c n 3). их производные Ответы: 1). константу c 4). x ′, x ′′, ... , x (n ) , y ′, y ′′, ... , y (n )

5). y ′, y ′′, ... , y (n )

Номер: 1.5.В Задача: Всякое решение дифференциального уравнения, полученное из общего решения y = ϕ(x , c ) … , называется частным решением. Выберите пропущенные слова 2). при c = 0 3). при c = c 0 Ответы: 1). при любом c 5). при y = y 0 4). при x = x 0

4


Номер: 1.6.В дифференциального

Задача: Общим интегралом уравнения y ′ = f (x , y ) называется Ответы: 1). общее решение, записанное в неявном виде 2). интеграл вида y = ∫ f (x , y, c )dx 3). интеграл вида y = ∫ f (x , y )dx + c 4). интеграл вида y = ∫ f (x , y, c )dy

5). интеграл вида y = ∫ f (x , y )dy + c

Номер: 1.7.В Задача: Геометрически общее решение y = ϕ(x , c ) дифференциального уравнения y ′ = f (x , y ) представляет на плоскости XOY Ответы: 1). интегральную кривую, проходящую через точку (x 0 , y 0 ) 2). множество интегральных кривых 3). поле направлений 4). изоклины 5). касательную интегральных кривых Номер: 1.8.В Задача: «Задача отыскания частного решения дифференциального уравнения y ′ = f (x , y ) , удовлетворяющего … называется задачей Коши». Вставьте пропущенные слова. Ответы: 1). данному уравнению 2). данному уравнению при c = 0 4). условию y(x 0 ) = y 0 3). данному уравнению при любом c 5). условию f (x 0 , y 0 ) = c 0 Номер: 1.9.В Задача: Какие из следующих дифференциальных уравнений являются уравнениями с разделяющимися или с разделенными переменными: 1) M 1 (x ) ⋅ N 1 (y )dx + M 2 (x ) ⋅ N 2 (y )dy = 0 2) y′ = M (x ) ⋅ N(y ) 3) y ′ = M (x ) N (y ) 4) y ′ = M (x , y ) N(y, y ) 5) y ′ = M (x ) + N (y ) Ответы: 1). только 1 2). только 2 и 5 3). только 2,35 4). только 1,2 и 3 5). только 4 Номер: 1.10.В Задача: Дифференциальное уравнение M (x , y )dx + N(x , y )dy = 0 является уравнением с разделяющимися переменными, если Ответы: 1). M (x , y ) = M 1 (x ) ⋅ N 1 (y ) , N(x , y ) = M 2 (x ) ⋅ N 2 (y )

∂M ∂N = ∂y ∂x

2). M (x , y ) = M 1 (x ) N 1 (y )

3).

4). уравнение разделить на N(x , y )dx

5). уравнение разделить на M (x , y )dx

5


Номер: 1.11.В Задача: Дифференциальное уравнение M 1 (x ) ⋅ N 1 (y )dx + M 2 (x ) ⋅ N 2 (y )dy = 0 начинаем решать следующим образом:

∂N1 ∂M 2 = 2). делим на M 2 (x ) ⋅ N 1 (y ) ≠ 0 ∂y ∂x 4). делаем замену y = t ⋅ x 3). делаем замену y = u ⋅ v 5). интегрируем ∫ M 1 (x ) ⋅ N 1 (y )dx + ∫ M 2 (x ) ⋅ N 2 (y )dy = c

Ответы: 1). проверяем условие

Номер: 1.12.В Задача: Какие из следующих дифференциальных уравнений являются уравнениями с разделяющимися или с разделенными переменными: 1) M 1 (x ) ⋅ N 1 (y )dx + M 2 (x ) ⋅ N 2 (y )dy = 0 2) y′ = M (x ) ⋅ N(y ) 3) y ′ = M (x ) N(y ) 4) M (x )dx + N(y )dy = 0 5) N(y )dx + M (x )dy = 0 Ответы: 1). все уравнения 2). 1,2,3,4 3). 1,2,3 4). 1,2 5). только 1 Номер: 1.13.В Задача: Дифференциальное уравнение y ′ = f (x ) ⋅ g (y ) начинаем решать следующим образом 2). делим на g(x ) ≠ 0 Ответы: 1). делим на f (x ) ≠ 0 3). интегрируем обе части 4). умножаем на dx , затем интегрируем 5). умножаем на dx , затем делим на f (x ) ≠ 0 Номер: 1.14.В Задача: Дифференциальное уравнение I порядка P(x , y )dx + Q(x , y )dy = 0 будет однородным, если: Ответы: 1). P(λx , λy ) = Q(λx , λy ) 2). P(λx , λy ) = λn ⋅ P(x , y ) , Q(λx , λy ) = λn ⋅ Q(x , y ) 3). P(λx , λy ) = λP(x , y ) , Q(λx , λy ) = Q(x , y ) 5). Q(x , y ) ≠ 0 4). P(x , y ) ≠ 0 Номер: 1.15.В

⎛ y⎞ ⎝x⎠

Задача: Дифференциальное уравнение I порядка y ′ = f ⎜ ⎟ в процессе решения сводится к уравнению:

⎛t⎞ ⎝x⎠

Ответы: 1). y ′ = f ⎜ ⎟

2). t ′ ⋅ x = f (t )

3). t ′ ⋅ x = f (t ) − t

⎛u⎞ ⎝v⎠

4). u ′v + u ⋅ v ′ = f (u , v )

5). u ′v + u ⋅ v ′ = f ⎜ ⎟

6


Номер: 1.16.В P(x , y )dx + Q(x , y )dy = 0 Задача: Если дифференциальное уравнение однородное, то делаем замену: Ответы: 1). y = t (x ) ⋅ x , dy = dt ⋅ x + t (x ) ⋅ dx 3). y = u (x ) ⋅ v(x ) 2). y = t (x ) ⋅ x , dy = dt ⋅ x + t (x ) 4). y =

u v

5). y = λ ⋅ x , где λ - любое число, λ ≠ 0

Номер: 1.17.В Задача: Дифференциальное уравнение y ′ = f (x , y ) называется однородным относительно x и y , если Ответы: 1). f (x , y ) является однородной функцией любого порядка 2). f (x , y ) является однородной функцией 1 порядка 3). f (x , y ) является однородной функцией 0 порядка 5). y = λ ⋅ x 4). f (x , y ) = 0 Номер: 1.18.В

⎛ y⎞ ⎝x⎠

Задача: Дифференциальное уравнение y ′( x ) = f ⎜ ⎟ решается следующим способом: Ответы: 1). после замены y = t ⋅ x получаем уравнение с разделяющимися переменными 2). после замены y = t ⋅ x получаем однородное уравнение 3). после замены y = u ⋅ v получаем уравнение с разделяющимися переменными 4). непосредственно интегрируем обе части уравнения 5). другим способом Номер: 1.19.В

⎛ y⎞ ⎝x⎠

Задача: Дифференциальное уравнение y ′( x ) = f ⎜ ⎟ сводится к решению

⎛y⎞ ⎝x⎠

Ответы: 1). интеграла вида y = ∫ f ⎜ ⎟dx

2). интеграла вида y = ∫ f (t )dt

3). двух однородных уравнений 4). двух уравнений с разделяющимися переменными 5). одного уравнения с разделяющимися переменными

7


Номер: 1.20.В

⎛ y⎞ ⎝x⎠

Задача: Дифференциальное уравнение y ′( x ) = f ⎜ ⎟ решается с помощью замены:

y , где t = t (x , y ) x 4). x = t

y , где t = t (x ) x 5). y = u ⋅ v

Ответы: 1). t =

2). t =

3). y = t

Номер: 1.21.В Задача: Однородное дифференциальное уравнение I порядка может быть приведено к Ответы: 1). однородному уравнению 0 порядка 2). однородному уравнению более высокого порядка 3). уравнению с разделяющимися переменными 4). двум уравнениям с разделяющимися переменными 5). неопределенному интегралу Номер: 1.22.В Задача: Дифференциальное уравнение I порядка P(x , y )dx + Q(x , y )dy = 0 является однородным, если: Ответы: 1). сделать замену y = t ⋅ x 2). оно однородное без дополнительных условий 3). P(x , y ) , Q(x , y ) однородные функции одного порядка

∂P ∂Q = ∂y ∂x 5). P(x , y ) = P1 (x ) ⋅ Q1 (y ) , Q(x , y ) = P2 (x ) ⋅ Q 2 (y )

4).

Номер: 1.23.В Задача: Какие из следующих дифференциальных уравнений I порядка являются однородными: 1) P(x , y )dx + Q(x , y )dy = 0 2) P(x , y )dx + Q(x , y )dy = 0 , где P, Q - однородные функции одного порядка 3) y ′ = f (y x ) 4) y ′ = f (y ) f (x ) 5) y ′ =

f n (y )

g n (x )

Ответы: 1). 1,2,3

2). 2,3

3). 4,5

4). 3,4,5

5). все

Номер: 1.24.В

⎛ y⎞ ⎝x⎠

Задача: Дифференциальное уравнение I порядка y ′ = f ⎜ ⎟ является Ответы: 1). однородным 2). с разделяющимися переменными 3). линейным 4). однородным, если y = λx 5). линейным, если y = λx 8


Номер: 1.25.В Задача: Дифференциальное уравнение y ′ + P(x , y ) = Q(x , y ) будет линейным, если 2). Q(x , y ) = Q(x ) Ответы: 1). P(x , y ) = P(x ) ⋅ y 3). P(x , y ) = P(x ) ⋅ y , Q(x , y ) = Q(x ) 4). P(x , y ) = P(x ) , Q(x , y ) = Q(x ) 5). P(x , y ) = P(y ) ⋅ x , Q(x , y ) = Q(y ) Номер: 1.26.В Задача: Какие из следующих уравнений являются линейными дифференциальными уравнениями I порядка: 1) y ′ = P(x ) ⋅ y = Q(x , y ) 3) x ′ + P(y ) ⋅ x = Q(y ) 4) x ′ + P(y ) ⋅ x = Q(x , y ) 2) y ′ + P(x ) ⋅ y = Q(x )

⎛ y⎞ ⎝x⎠

5) y ′ = f ⎜ ⎟ Ответы: 1). все уравнения 4). 2 и 3

2). только 1 5). 1,2,3,4

3). только 2

Номер: 1.27.В Задача: Дифференциальное уравнение I порядка y ′ + P(x ) ⋅ y = Q(x ) является Ответы: 1). однородным 2). с разделяющимися переменными 3). линейным 4). однородным после замены y = t ⋅ x 5). линейным после подстановки y = u ⋅ v Номер: 1.28.В Задача: Дифференциальное уравнение I порядка x ′ + P(y ) ⋅ x = Q(y ) является Ответы: 1). однородным 2). с разделяющимися переменными 3). линейным 4). однородным после замены x = t ⋅ y 5). линейным после подстановки x = u ⋅ v Номер: 1.29.В Задача: Дифференциальное уравнение y ′ + P(x ) ⋅ y = Q(x ) сводится к решению Ответы: 1). одного уравнения с разделяющимися переменными после подстановки y = u ⋅ v 2). двух уравнений с разделяющимися переменными после подстановки

y = u⋅v

3). одного однородного уравнения после замены y = t ⋅ x 4). двух однородных уравнений после замены y = t ⋅ x 5). интегралов ∫ (y ′ + P(x ) ⋅ y )dx = ∫ Q(x )dx

9


Номер: 1.30.В Задача: Дифференциальное уравнение x ′ + P(y ) ⋅ x = Q(y ) можно решить Ответы: 1). непосредственным интегрированием обеих частей уравнения 2). с помощью замены y = u ⋅ v 3). с помощью замены x = u ⋅ v 4). с помощью замены x = t ⋅ y 5). сведением к двум уравнениям x ′ + P(y ) ⋅ x = 0 , Q(y ) = 0 Номер: 1.31.В Задача: Дифференциальное уравнение y ′ + P(x ) ⋅ y = Q(x ) можно решить Ответы: 1). непосредственным интегрированием обеих частей уравнения 2). с помощью замены y = u ⋅ v 3). с помощью замены x = u ⋅ v 4). с помощью замены y = t ⋅ x 5). сведением к двум уравнениям y ′ + P(x ) ⋅ y = 0 , Q(x ) = 0 Номер: 1.32.В Задача: Дифференциальное уравнение y ′ + P(x ) ⋅ y = Q(x ) в процессе сводится к решению уравнений: 2). u ′ + P(x ) ⋅ u = 0 , u ′v = Q(x ) Ответы: 1). v ′ + P(x ) ⋅ v = 0 , u ′v = Q(x ) 3). u ′ + P(x ) ⋅ y = 0 , Q(x ) = 0 4). u ′ + P(x ) ⋅ y = 0 , y ′ = Q(x ) 5). y ′ = P(x ), P(x ) ⋅ y = Q(x ) Номер: 1.33.В Задача: Дифференциальное уравнение I порядка называется линейным, если оно линейно относительно искомой функции y(x ) и Ответы: 1). независимой переменной x 2). ее производной 3). ее производных 4). решение уравнения имеет вид y = u (x , c ) ⋅ v(x ) 5). правая часть уравнения не равна нулю Номер: 1.34.В Задача: Дифференциальное уравнение I порядка называется линейным, если Ответы: 1). x , y, y ′ входят в уравнение в первой степени ( x - независимая переменная, y = y(x ) ) 2). y, y ′ входят в уравнение в первой степени 3). графически решение уравнения изображается только в виде прямой линии 4). y(x ) - линейная функция от x 5). y ′(x ) - линейная функция от x

10


Номер: 1.35.В Задача: Дифференциальное уравнение y ′ + P(x ) ⋅ y = Q(x ) можно решить Ответы: 1). только методом подстановки 2). только методом вариации постоянной 3). 1 или 2 способом 4). только графически 5). только методом изоклин Номер: 1.36.С Задача: Используя приведенные ниже пункты, составьте план решения уравнения y ′ + P(x ) ⋅ y = Q(x ) методом вариации произвольной постоянной: а) находим общее решение y = y(x , c ) однородного уравнения y ′ = P(x ) ⋅ y = 0 б) полагаем c = c(x ) в) подставляя c(x ) в равенство y = y(x , c(x )) получаем общее решение неоднородного уравнения г) находим c(x ) д) вычисляем y ′ е) подставляем y, y ′ в неоднородное уравнение y ′ + P(x ) ⋅ y = Q(x ) Ответы: 1). а,б,д,е,г,в 2). а,б,в,г,д,е 3). а,д,е,б,г 4). а,б,в 5). б,г,в Номер: 1.37.С Задача: Для того, чтобы уравнение P(x , y )dx + Q(x , y )dy = 0 , где P(x , y ) , Q(x , y ) - непрерывные функции, было уравнением в полных дифференциалах необходимо и достаточно Ответы: 1).

∂P ∂Q = ∂y ∂x

2). Py′ , Q ′x - непрерывные функции

3). Py′ , Q ′x - непрерывные функции,

∂P ∂Q = ∂y ∂x

4). P(x , y )dx + Q(x , y )dy = u (x , y ) 5). уравнение умножить на интегрирующий множитель Номер: 1.38.С Задача: Дифференциальное уравнение P(x , y )dx + Q(x , y )dy = 0 называется уравнением в полных дифференциалах, если

∂P ∂Q ∂P ∂Q = = 2). Pdx + Qdy = u (x , y ) 3). ∂y ∂x ∂x ∂y 5). P = P1 (x ) ⋅ Q1 (y ) , Q = P2 (x ) ⋅ Q 2 (y ) 4). Pdx + Qdy = du (x , y )

Ответы: 1).

Номер: 1.39.С Задача: В процессе решения уравнение P(x , y )dx + Q(x , y )dy = 0 в полных дифференциалах получаем выражение 2). u (x , y ) = ∫ Q(x , y )dy + ϕ(y ) Ответы: 1). u (x , y ) = ∫ P(x , y )dx + ϕ(y ) 3). u (x , y ) = ∫ P(x , y )dx + Q(x , y )dy 5). ∫ P(x , y )dx + Q(x , y )dy = c 11

4). ∫ P(x , y )dx + ∫ Q(x , y )dy = c


Номер: 1.40.С P(x , y )dx + Q(x , y )dy = 0 Задача: Решение уравнения дифференциалах ищем в виде x

y

x0 (x , y )

y0

Ответы: 1). ∫ P(x , y )dx + ∫ Q(x 0 , y )dy = c 3).

∫ P(x , y )dx + Q(x , y )dy = c

(x 0 , y0 ) x

y

x0

y0

x

y

x0 (x , y )

y0

в

полных

2). ∫ P(x , y )dx + ∫ Q(x , y )dy = c 4).

∫ P(x , y )dx + Q(x , y )dy = 0

(x 0 , y0 )

5). ∫ P(x , y )dx + ∫ Q(x , y )dy = 0 Номер: 1.41.С Задача: Для дифференциального уравнения P(x , y )dx + Q(x , y )dy = 0 , где Pdx + Qdy = dU(x , y ) функцию U(x , y ) находим из условий 2). U ′ = Pdx + Qdy 3). U ′ = P + Q Ответы: 1). U ′x = P , U ′y = Q

5). U ′y = (∫ Pdx )

4). U ′x = Py′ , U ′y = Q ′x

y

Номер: 1.42.С P(x , y )dx + Q(x , y )dy = 0 , где Задача: Дифференциальное уравнение P, Q, Py′ , Q ′x - непрерывные функции, Py′ ≠ Q ′x Ответы: 1). не может быть уравнением в полных дифференциалах 2). может быть уравнением в полных дифференциалах при умножении на интегрирующий множитель 3). будет однородным после замены y = t ⋅ x 4). будет уравнением с разделяющимися переменными, если

P(x , y ) = P1 (x ) ⋅ Q1 (y )

5). будет линейным после замены y = u ⋅ v Номер: 1.43.С Задача: Какие из следующих дифференциальных уравнений I порядка являются уравнениями Бернулли: 1) y ′ + p(x ) ⋅ y = q(x ) ⋅ y n 2) y ′ + p(x ) ⋅ y = q (x , y ) 3)

y ′ + p(x ) ⋅ y = y n 4) x ′ + p(y ) ⋅ x = q(y ) ⋅ x n 5) x ′ + p(y ) ⋅ x = q(y, x )

Ответы: 1). только 1 2). только 1 и 3 3). только 1 и 4 4). только 2 и 5 5). только 1,3 и 4 Номер: 1.44.С уравнение y ′ + p (x ) ⋅ y = q ( x ) ⋅ y n

Задача: Дифференциальное следующим образом : Ответы: 1). только замена y = u ⋅ v

12

решаем


2). только поделив на y n и сделав замену z = y1− n 3). 1 или 2 способом 4). непосредственным интегрированием 5). только замена y = t ⋅ x Номер: 1.45.С Задача: Дифференциальное уравнение y ′ + p(x ) ⋅ y = q(x ) ⋅ y n является: Ответы: 1). уравнением Бернулли 2). линейным относительно y ′, y 3). линейным после замены y = u ⋅ v 4). однородным после замены y = t ⋅ x 5). однородным, если p и q однородные функции Номер: 1.46.С Задача: Дифференциальное уравнение y ′ + p(x ) ⋅ y = q(x ) ⋅ y n решению Ответы: 1). двух уравнений с разделяющимися переменными 2). одного уравнения с разделяющимися переменными 3). двух однородных уравнений 4). двух интегралов: ∫ (y ′ + p(x )y )dx и ∫ q(x ) ⋅ y n dx

(

сводится к

)

5). двух уравнений: y ′ + p(x ) ⋅ y = 0 и q(x ) ⋅ y n = 0

Номер: 1.47.С Задача: Дифференциальное уравнение y ′ + p(x ) ⋅ y = q(x ) ⋅ y n после замены

z = y1− n становится уравнением Ответы: 1). с разделяющимися переменными 3). однородным, если p и q однородные функции 5). линейным после замены y = u ⋅ v

2). однородным 4). линейным

Номер: 1.48.С Задача: Дифференциальное уравнение y ′ + p(x ) ⋅ y = q(x ) ⋅ y n после замены y = u ⋅ v становится Ответы: 1). уравнением Бернулли 2). линейным уравнением 3). уравнением с разделяющимися переменными 4). однородным уравнением 5). уравнением, распадающимся на два уравнения

13


2. Обыкновенные дифференциальные уравнения I порядка. Типы уравнений Задача:

e

x +3y

Определите

тип

Номер: 2.1.А дифференциального

Задача:

Определите

(x + 1)y′ + y = x

3

+x

тип

Номер: 2.2.А дифференциального

порядка

Задача:

Определите

xy′ − y = x ⋅ tg

тип

y x

Определите

(1 + e )y ⋅ dy − e dx = 0 x

тип

уравнения

2). однородное 5). Бернулли

Номер: 2.4.А дифференциального

I

порядка

уравнения

3). линейное

I

порядка

3). линейное

I

порядка

y

Ответы: 1). с разделяющимися переменными 4). в полных дифференциалах Задача:

Определите

(

y = x y′ − e

y x

)

тип

)

Определите

тип

2). однородное 5). Бернулли

Номер: 2.5.А дифференциального

Ответы: 1). с разделяющимися переменными 4). в полных дифференциалах Задача:

уравнения

2). однородное 5). Бернулли

Номер: 2.3.А дифференциального

Ответы: 1). с разделяющимися переменными 4). в полных дифференциалах Задача:

2). однородное 5). Бернулли

2

Ответы: 1). с разделяющимися переменными 4). в полных дифференциалах

(

I

dy = xdx

Ответы: 1). с разделяющимися переменными 3). линейное 4). в полных дифференциалах

2

уравнения

уравнения

2). однородное 5). Бернулли

Номер: 2.6.А дифференциального

уравнения

3). линейное

I

порядка

3). линейное

I

порядка

x

x y + 3 dx − e ⋅ ydy = 0 Ответы: 1). с разделяющимися переменными 4). в полных дифференциалах

14

2). однородное 5). Бернулли

3). линейное


Задача:

(x

2

Определите

)

тип

Номер: 2.7.А дифференциального

уравнения

I

порядка

+ 1 y′ + 4 xy = 3

Ответы: 1). с разделяющимися переменными 4). в полных дифференциалах

2). однородное 5). Бернулли

3). линейное

Номер: 2.8.А x y

Задача: Определите тип дифференциального уравнения I порядка y′ = e − 1 Ответы: 1). с разделяющимися переменными 2). однородное 3). линейное 4). в полных дифференциалах 5). Бернулли Задача:

Определите

тип

Номер: 2.9.А дифференциального

уравнения

I

порядка

xy′ = x 2 − y 2 + y Ответы: 1). с разделяющимися переменными 4). в полных дифференциалах Задача:

Определите

тип

2). однородное 5). Бернулли

Номер: 2.10.А дифференциального

уравнения

3). линейное

I

порядка

x y′ = 2 xy + 3 2

Ответы: 1). с разделяющимися переменными 4). в полных дифференциалах Задача:

Определите

y′ = (2 y + 1)tg x

тип

Номер: 2.11.А дифференциального

Ответы: 1). с разделяющимися переменными 4). в полных дифференциалах Задача:

Определите

(

2

y′ = e x ⋅ x 1 + y 2

)

тип

(y +

)

Определите

тип

уравнения

2). однородное 5). Бернулли

Номер: 2.12.А дифференциального

Ответы: 1). с разделяющимися переменными 4). в полных дифференциалах Задача:

2). однородное 5). Бернулли

уравнения

2). однородное 5). Бернулли

Номер: 2.13.А дифференциального

уравнения

3). линейное

I

порядка

3). линейное

I

порядка

3). линейное

I

порядка

xy dx = xdy

Ответы: 1). с разделяющимися переменными 4). в полных дифференциалах 15

2). однородное 5). Бернулли

3). линейное


Задача:

3x

2

+y

Определите

тип

Номер: 2.14.А дифференциального

уравнения

I

порядка

dy + xdx = 0

Ответы: 1). с разделяющимися переменными 4). в полных дифференциалах Задача:

Определите

тип

2). однородное 5). Бернулли

Номер: 2.15.А дифференциального

уравнения

3). линейное

I

порядка

y + x y′ = xyy′ 2

2

Ответы: 1). с разделяющимися переменными 4). в полных дифференциалах Задача:

Определите

(xy′ − 1)ln x = 2 y

тип

Номер: 2.16.А дифференциального

Ответы: 1). с разделяющимися переменными 4). в полных дифференциалах Задача:

Определите

yx′ + x = 4 y + 3y 3

тип

уравнения

2). однородное 5). Бернулли

Номер: 2.17.А дифференциального

уравнения

3). линейное

I

порядка

3). линейное

I

порядка

2

Ответы: 1). с разделяющимися переменными 4). в полных дифференциалах Задача:

2). однородное 5). Бернулли

Определите

тип

2). однородное 5). Бернулли

Номер: 2.18.А дифференциального

уравнения

3). линейное

I

порядка

x x ⎛ ⎞ ⎜1 − e y ⎟dx + e y ⎛⎜1 + x ⎞⎟dy = 0 ⎜ y⎟ ⎜ ⎟ ⎠ ⎝ ⎝ ⎠

Ответы: 1). с разделяющимися переменными 4). в полных дифференциалах Задача:

Определите

тип

2). однородное 5). Бернулли

Номер: 2.19.А дифференциального

уравнения

3). линейное

I

порядка

⎛ ⎛ sin 2 x ⎞ sin x ⎞ + x ⎟⎟dx + ⎜⎜ y − 2 ⎟⎟dy = 0 ⎜⎜ y ⎠ ⎝ y ⎠ ⎝ 2

Ответы: 1). с разделяющимися переменными 4). в полных дифференциалах

16

2). однородное 5). Бернулли

3). линейное


Номер: 2.20.А Задача: Определите тип дифференциального уравнения I порядка y′ + y = x y Ответы: 1). с разделяющимися переменными 2). однородное 3). линейное 4). в полных дифференциалах 5). Бернулли Задача:

Определите

тип

Номер: 2.21.А дифференциального

уравнения

I

порядка

y′ + 2 xy = 2 x y

2 3

Ответы: 1). с разделяющимися переменными 4). в полных дифференциалах Задача:

Определите

тип

2). однородное 5). Бернулли

Номер: 2.22.А дифференциального

уравнения

3). линейное

I

порядка

y′ = y cos x + y ⋅ tg x 4

Ответы: 1). с разделяющимися переменными 4). в полных дифференциалах Задача:

y′ +

x3

Определите

тип

2). однородное 5). Бернулли

Номер: 2.23.А дифференциального

уравнения

3). линейное

I

порядка

y = 3y

Ответы: 1). с разделяющимися переменными 4). в полных дифференциалах

2). однородное 5). Бернулли

Номер: 2.24.А Задача: Определить тип дифференциального уравнения y′ =

3). линейное

sin y − 1 x +1

Ответы: 1). линейное дифференциальное уравнение 2). дифференциальное уравнение с разделяющимися переменными 3). однородное дифференциальное уравнение 4). уравнение Бернулли 5). в полных дифференциалах Номер: 2.25.А Задача: Определить тип дифференциального уравнения y′ = y ln y Ответы: 1). линейное дифференциальное уравнение 2). дифференциальное уравнение с разделяющимися переменными 3). однородное дифференциальное уравнение 4). уравнение Бернулли 5). в полных дифференциалах

17


Номер: 2.26.А

⎛ ⎝

y⎞ x⎠

Задача: Определить тип дифференциального уравнения xy′ = y cos⎜ ln ⎟ Ответы: 1). линейное дифференциальное уравнение 2). дифференциальное уравнение с разделяющимися переменными 3). однородное дифференциальное уравнение 4). уравнение Бернулли 5). в полных дифференциалах Номер: 2.27.А x−

1 x

Задача: Определить тип дифференциального уравнения x y′ − y = x e Ответы: 1). линейное дифференциальное уравнение 2). дифференциальное уравнение с разделяющимися переменными 3). однородное дифференциальное уравнение 4). уравнение Бернулли 5). в полных дифференциалах 2

Задача:

Номер: 2.28.А тип дифференциального

Определить

2

уравнения

y′ sin x cos x = y + sin 3 x Ответы: 1). линейное дифференциальное уравнение 2). дифференциальное уравнение с разделяющимися переменными 3). однородное дифференциальное уравнение 4). уравнение Бернулли 5). в полных дифференциалах Задача:

Номер: 2.29.А тип дифференциального

Определить

x (2 x − 1)y′ + y 2 = (4 x + 1)y

уравнения

Ответы: 1). линейное дифференциальное уравнение 2). дифференциальное уравнение с разделяющимися переменными 3). однородное дифференциальное уравнение 4). уравнение Бернулли 5). в полных дифференциалах Задача:

Номер: 2.30.А тип дифференциального

Определить

(2x y − 2x )y′ + 2y x − 6x y + 4x 2

3

2

2

3

=0

18

уравнения


Ответы: 1). линейное дифференциальное уравнение 2). дифференциальное уравнение с разделяющимися переменными 3). однородное дифференциальное уравнение 4). уравнение Бернулли 5). в полных дифференциалах Номер: 2.31.А Задача: Определить тип дифференциального уравнения y′ = y 2 sin x Ответы: 1). линейное дифференциальное уравнение 2). дифференциальное уравнение с разделяющимися переменными 3). однородное дифференциальное уравнение 4). уравнение Бернулли 5). в полных дифференциалах Номер: 2.32.А Задача: Определить тип дифференциального уравнения y′ + 4 x 3 y 3 + 2 xy = 0 Ответы: 1). линейное дифференциальное уравнение 2). дифференциальное уравнение с разделяющимися переменными 3). однородное дифференциальное уравнение 4). уравнение Бернулли 5). в полных дифференциалах Номер: 2.33.А Задача: Определить тип дифференциального уравнения 2 x 1 − y 2 dx + ydy = 0 Ответы: 1). линейное дифференциальное уравнение 2). дифференциальное уравнение с разделяющимися переменными 3). однородное дифференциальное уравнение 4). уравнение Бернулли 5). в полных дифференциалах Номер: 2.34.А Задача: Определить тип дифференциального уравнения xy′ = y + x 1 + e y x Ответы: 1). линейное дифференциальное уравнение 2). дифференциальное уравнение с разделяющимися переменными 3). однородное дифференциальное уравнение 4). уравнение Бернулли 5). в полных дифференциалах

(

Задача:

(3y

2

Номер: 2.35.А тип дифференциального

Определить

)

(

2

)

+ 2 xy + 2 x dx + 6 xy + x + 3 dy = 0

19

)

уравнения


Ответы: 1). линейное дифференциальное уравнение 2). дифференциальное уравнение с разделяющимися переменными 3). однородное дифференциальное уравнение 4). уравнение Бернулли 5). в полных дифференциалах Номер: 2.36.А Задача: Определить тип дифференциального уравнения y′x 3 sin x + 2 y = xy′ Ответы: 1). линейное дифференциальное уравнение 2). дифференциальное уравнение с разделяющимися переменными 3). однородное дифференциальное уравнение 4). уравнение Бернулли 5). в полных дифференциалах Номер: 2.37.А Задача: Определить тип дифференциального уравнения y′ + y cos x = e − sin x Ответы: 1). линейное дифференциальное уравнение 2). дифференциальное уравнение с разделяющимися переменными 3). однородное дифференциальное уравнение 4). уравнение Бернулли 5). в полных дифференциалах Номер: 2.38.А Задача: Определить тип дифференциального уравнения

(1 + x )ydx + (1 − y )xdy = 0

Ответы: 1). линейное дифференциальное уравнение 2). дифференциальное уравнение с разделяющимися переменными 3). однородное дифференциальное уравнение 4). уравнение Бернулли 5). в полных дифференциалах Номер: 2.39.А Задача: Определить тип дифференциального уравнения xy 2 dy = x 3 + y 3 dx Ответы: 1). линейное дифференциальное уравнение 2). дифференциальное уравнение с разделяющимися переменными 3). однородное дифференциальное уравнение 4). уравнение Бернулли 5). в полных дифференциалах

(

Задача:

Определить

(x − x )y′ + (2x 3

Номер: 2.40.А тип дифференциального

2

)

3

− 1 y − ax = 0 20

)

уравнения


Ответы: 1). линейное дифференциальное уравнение 2). дифференциальное уравнение с разделяющимися переменными 3). однородное дифференциальное уравнение 4). уравнение Бернулли 5). в полных дифференциалах Задача:

Определить

Номер: 2.41.А тип дифференциального

уравнения

xdy − ydx = x 2 + y 2 dx Ответы: 1). линейное дифференциальное уравнение 2). дифференциальное уравнение с разделяющимися переменными 3). однородное дифференциальное уравнение 4). уравнение Бернулли 5). в полных дифференциалах Номер: 2.42.А Задача: Определить тип дифференциального уравнения zdt − t 2 − a 2 dz = 0 Ответы: 1). линейное дифференциальное уравнение 2). дифференциальное уравнение с разделяющимися переменными 3). однородное дифференциальное уравнение 4). уравнение Бернулли 5). в полных дифференциалах

(

Задача:

Определить

y − y′ cos x = y cos x (1 − sin x )

Номер: 2.43.А тип дифференциального

)

уравнения

2

Ответы: 1). линейное дифференциальное уравнение 2). дифференциальное уравнение с разделяющимися переменными 3). однородное дифференциальное уравнение 4). уравнение Бернулли 5). в полных дифференциалах Номер: 2.44.А

⎛ ln x 3y 2 ⎞ 2 ydy Задача: Определить тип дифференциального уравнения ⎜⎜ 2 + 4 ⎟⎟dx = x ⎠ x3 ⎝ x Ответы: 1). линейное дифференциальное уравнение 2). дифференциальное уравнение с разделяющимися переменными 3). однородное дифференциальное уравнение 4). уравнение Бернулли 5). в полных дифференциалах

21


Номер: 2.45.А Задача: Определить тип дифференциального уравнения y ln 3 y + y′ x + 1 = 0 Ответы: 1). линейное дифференциальное уравнение 2). дифференциальное уравнение с разделяющимися переменными 3). однородное дифференциальное уравнение 4). уравнение Бернулли 5). в полных дифференциалах Номер: 2.46.А Задача: Определить тип дифференциального уравнения

(e

x+y

)

(

)

+ 3x 2 dx + e x + y + 4 ⋅ sin 3 y dy = 0

Ответы: 1). линейное дифференциальное уравнение 2). дифференциальное уравнение с разделяющимися переменными 3). однородное дифференциальное уравнение 4). уравнение Бернулли 5). в полных дифференциалах Номер: 2.47.А Задача: Определить тип дифференциального уравнения y′ +

2y = 3x 2 y 4 3 x

Ответы: 1). линейное дифференциальное уравнение 2). дифференциальное уравнение с разделяющимися переменными 3). однородное дифференциальное уравнение 4). уравнение Бернулли 5). в полных дифференциалах Номер: 2.48.А Задача: Определить тип дифференциального уравнения 1 + x 2 y′ + y = arctg x Ответы: 1). линейное дифференциальное уравнение 2). дифференциальное уравнение с разделяющимися переменными 3). однородное дифференциальное уравнение 4). уравнение Бернулли 5). в полных дифференциалах

(

)

Номер: 2.49.А Задача: Определить тип дифференциального уравнения x 2 + y 2 dx − xydy = 0 Ответы: 1). линейное дифференциальное уравнение 2). дифференциальное уравнение с разделяющимися переменными 3). однородное дифференциальное уравнение 4). уравнение Бернулли 5). в полных дифференциалах

(

22

)


Задача:

Определить

Номер: 2.50.А тип дифференциального

уравнения

dx dy + =0 x (ln y − 1) y(x + 2 ) Ответы: 1). линейное дифференциальное уравнение 2). дифференциальное уравнение с разделяющимися переменными 3). однородное дифференциальное уравнение 4). уравнение Бернулли 5). в полных дифференциалах Номер: 2.51.А Задача: Определить тип дифференциального уравнения y′ +

2y 2 y = x cos 2 x

Ответы: 1). линейное дифференциальное уравнение 2). дифференциальное уравнение с разделяющимися переменными 3). однородное дифференциальное уравнение 4). уравнение Бернулли 5). в полных дифференциалах Номер: 2.52.А Задача: Определить тип дифференциального уравнения

(x

2

)

+ sin y dx + (1 + x cos y )dy = 0

Ответы: 1). линейное дифференциальное уравнение 2). дифференциальное уравнение с разделяющимися переменными 3). однородное дифференциальное уравнение 4). уравнение Бернулли 5). в полных дифференциалах Номер: 2.53.А Задача: Определить тип дифференциального уравнения xyy′ = y 2 + 2 x 2 Ответы: 1). линейное дифференциальное уравнение 2). дифференциальное уравнение с разделяющимися переменными 3). однородное дифференциальное уравнение 4). уравнение Бернулли 5). в полных дифференциалах Задача:

( xy

5

)

Определить

− x dx +

(

Номер: 2.54.А тип дифференциального

)

xy + y ⋅ ln x dy = 0

23

уравнения


Ответы: 1). линейное дифференциальное уравнение 2). дифференциальное уравнение с разделяющимися переменными 3). однородное дифференциальное уравнение 4). уравнение Бернулли 5). в полных дифференциалах Номер: 2.55.А Задача: Определить тип дифференциального уравнения y' sin x − y cos x = 1 Ответы: 1). линейное дифференциальное уравнение 2). дифференциальное уравнение с разделяющимися переменными 3). однородное дифференциальное уравнение 4). уравнение Бернулли 5). в полных дифференциалах Задача:

Определить

Номер: 2.56.А тип дифференциального

уравнения

⎛ x2 ⎞ (y + x ln y )dx + ⎜⎜ + x + 1⎟⎟dy = 0 ⎝ 2y ⎠ Ответы: 1). линейное дифференциальное уравнение 2). дифференциальное уравнение с разделяющимися переменными 3). однородное дифференциальное уравнение 4). уравнение Бернулли 5). в полных дифференциалах Номер: 2.57.А

⎛ y⎞ ⎝x⎠

Задача: Определить тип дифференциального уравнения xy′ − y = xtg⎜ ⎟ Ответы: 1). линейное дифференциальное уравнение 2). дифференциальное уравнение с разделяющимися переменными 3). однородное дифференциальное уравнение 4). уравнение Бернулли 5). в полных дифференциалах Номер: 2.58.А Задача: Определить тип дифференциального уравнения

yy′ y +e =0 x

Ответы: 1). линейное дифференциальное уравнение 2). дифференциальное уравнение с разделяющимися переменными 3). однородное дифференциальное уравнение 4). уравнение Бернулли 5). в полных дифференциалах

24


Номер: 2.59.А Задача: Определить тип дифференциального уравнения y′ + 2 y − y 2 ⋅ sin x = 0 Ответы: 1). линейное дифференциальное уравнение 2). дифференциальное уравнение с разделяющимися переменными 3). однородное дифференциальное уравнение 4). уравнение Бернулли 5). в полных дифференциалах Номер: 2.60.А Задача: Определить тип дифференциального уравнения xy′ = x 2 − y 2 + y Ответы: 1). линейное дифференциальное уравнение 2). дифференциальное уравнение с разделяющимися переменными 3). однородное дифференциальное уравнение 4). уравнение Бернулли 5). в полных дифференциалах Номер: 2.61.А 2

Задача: Определить тип дифференциального уравнения y′ + 2 xy = xe − x Ответы: 1). линейное дифференциальное уравнение 2). дифференциальное уравнение с разделяющимися переменными 3). однородное дифференциальное уравнение 4). уравнение Бернулли 5). в полных дифференциалах Задача:

Определить

Номер: 2.62.А тип дифференциального

уравнения

⎛ ⎛ sin 2x ⎞ sin 2 x ⎞ ⎟dy = 0 ⎜ ⎜⎜ + x ⎟⎟dx + ⎜ y − 2 ⎟ y ⎠ ⎝ y ⎠ ⎝ Ответы: 1). линейное дифференциальное уравнение 2). дифференциальное уравнение с разделяющимися переменными 3). однородное дифференциальное уравнение 4). уравнение Бернулли 5). в полных дифференциалах Номер: 2.63.А Задача: Определить тип дифференциального уравнения y 2 + x 2 y′ = xyy′ Ответы: 1). линейное дифференциальное уравнение 2). дифференциальное уравнение с разделяющимися переменными 3). однородное дифференциальное уравнение 4). уравнение Бернулли 5). в полных дифференциалах 25


Номер: 2.64.А Задача: Определить тип дифференциального уравнения e y dx + xe y − 2 y dy = 0 Ответы: 1). линейное дифференциальное уравнение 2). дифференциальное уравнение с разделяющимися переменными 3). однородное дифференциальное уравнение 4). уравнение Бернулли 5). в полных дифференциалах

(

)

Номер: 2.65.А Задача: Определить тип дифференциального уравнения y′ − y tg x + y 2 cos x = 0 Ответы: 1). линейное дифференциальное уравнение 2). дифференциальное уравнение с разделяющимися переменными 3). однородное дифференциальное уравнение 4). уравнение Бернулли 5). в полных дифференциалах Номер: 2.66.А

1 + ey Задача: Определить тип дифференциального уравнения y′ = xy 1 + x 2

(

)

Ответы: 1). линейное дифференциальное уравнение 2). дифференциальное уравнение с разделяющимися переменными 3). однородное дифференциальное уравнение 4). уравнение Бернулли 5). в полных дифференциалах Номер: 2.67.А Задача: Определить тип дифференциального уравнения 4 y 2 + x 2 y′ = xy Ответы: 1). линейное дифференциальное уравнение 2). дифференциальное уравнение с разделяющимися переменными 3). однородное дифференциальное уравнение 4). уравнение Бернулли 5). в полных дифференциалах

(

Задача:

Определите 2

тип

Номер: 2.68.В дифференциального

уравнения

)

I

2

2x 3x + y dx − dy = 0 y3 y4 Ответы: 1). с разделяющимися переменными 3). линейное 4). в полных дифференциалах

26

2). однородное 5). Бернулли

порядка


Задача:

Определите

тип

Номер: 2.69.В дифференциального

уравнения

I

порядка

y′x sin y = xy′ − 2 y 3

Ответы: 1). с разделяющимися переменными 3). линейное 4). в полных дифференциалах

2). однородное 5). Бернулли

Номер: 2.70.В Задача: Определите тип дифференциального уравнения I порядка y′ = Ответы: 1). с разделяющимися переменными 3). линейное 4). в полных дифференциалах Задача:

Определите

(x + 1)(y′ + y 2 ) = − y

тип

Номер: 2.71.В дифференциального

Ответы: 1). с разделяющимися переменными 3). линейное 4). в полных дифференциалах

y 3x − y 2

2). однородное 5). Бернулли уравнения

I

порядка

2). однородное 5). Бернулли

Номер: 2.72.В Задача: Определить тип дифференциального уравнения yy′ + y 2 + 4 x (x + 1) = 0 Ответы: 1). линейное дифференциальное уравнение 2). дифференциальное уравнение с разделяющимися переменными 3). однородное дифференциальное уравнение 4). уравнение Бернулли 5). в полных дифференциалах Номер: 2.73.В Задача: Определить тип дифференциального уравнения y′ x 2 y 3 + xy = 1 Ответы: 1). линейное дифференциальное уравнение 2). дифференциальное уравнение с разделяющимися переменными 3). однородное дифференциальное уравнение 4). уравнение Бернулли 5). в полных дифференциалах

(

)

Номер: 2.74.В Задача: Определить тип дифференциального уравнения y'⋅ y 3 − e x y = y ⋅ e x Ответы: 1). линейное дифференциальное уравнение 2). дифференциальное уравнение с разделяющимися переменными 3). однородное дифференциальное уравнение 4). уравнение Бернулли 5). в полных дифференциалах

(

27

)


3. Дифференциальные уравнения с разделяющимися переменными Номер: 3.1.А

1

Задача: Найти общее решение дифференциального уравнения y' = Ответы: 1). y = 2 1 − x 2 4). y = arctgx + c

1− x2 3). y = ln 1 − x 2 ⋅ c

(

2). y = arcsin x + c 5). y = arccos x + c

)

Номер: 3.2.А Задача: Найти общее решение дифференциального уравнения y' = sin x Ответы: 1). y = − cos x + c 4). y = ln tg

x +c 2

sin 2 x 2). y = +c 2

3). y = ln sin x + c

5). y = arcsin x + c

Номер: 3.3.А Задача: Найти общее решение дифференциального уравнения y' = y

y2 Ответы: 1). y = +c 2

2). y = ln y ⋅ c

3). y = c ⋅ e

x

4). y =

1 c ⋅ex

5). y =

c x

Номер: 3.4.А Задача: Найти общее решение дифференциального уравнения xy'− y = 0 Ответы: 1). y = e x + c

2). y = ln cx

3). y =

1 +c ex

4). y = cx

5). y =

1 cx

Номер: 3.5.А Задача: Найти общее решение дифференциального уравнения xy'+ y = 0 Ответы: 1). y = cx

2). y = ln cx

3). y = e x + c

4). y =

1 +c ex

Номер: 3.6.А Задача: Решите дифференциальное уравнение y' =

(y + 1)2

2

= x +c 2 4). y + 1 = e 2 x + c

Ответы: 1).

y2 + y = x2 + c 2). 2 5). y + 1 = 2 x + c

28

2x y +1 y2 = x2 + c 3). 2

5). y =

1 cx


Номер: 3.7.А Задача: Найти частное решение дифференциального уравнения xy'− y = 0 при условии y(− 2 ) = 4 Ответы: 1). y = −

x 2

2). y = − x + 2

3). y = −2 x

4). y = 2 x

5). y =

Номер: 3.8.А Задача: Найти общее решение дифференциального уравнения y' = tgx Ответы: 1). y = ln cos x ⋅ c 4). y = ctgx + c

c cos x 5). y = tg 2 x + c

2). y = ln

3). y = ln c ⋅ sin x

Номер: 3.9.А Задача: Решите дифференциальное уравнение y' = (1 + x )(1 + y )

x2 Ответы: 1). arctgy = x + +c 2 x2 3). ln 1 + y = x + +c 2

( 1 x + 1)2 2). = +c 2 2 (1 + y ) 4). y = ln x + 1 + c

5).

( x + 1)2 y= +c 2

Номер: 3.10.А

2x y +1 y2 3). = −x 2 + c 2

Задача: Решите дифференциальное уравнение y' = Ответы: 1). y + 1 = 2 x + c

y2 4). + y = x2 + c 2

y2 2). = x2 + c 2 2 ( y + 1) 5). = 2x 2 + c 2

Номер: 3.11.А Задача: Решите дифференциальное уравнение y' = xy − y

y2 Ответы: 1). = ln c(x − 1) 2

x2 4). ln y = +c 2

x2 2). ln y = c − +x 2

x2 5). ln y = −x+c 2

29

1 x2 +x+c 3). 2 = − 2 y

x 2


Номер: 3.12.А Задача: Решите дифференциальное уравнение cos2 x ⋅ y' = y + 3

1 1 = − + c 2). ln y + 3 = tgx + c ctgx (y + 3)2 1 1 +c 3). ln y + 3 = − + c 4). ln y + 3 = − cos x cos3 x

Ответы: 1).

5). y + 3 = tgx + c

Номер: 3.13.А Задача: Решите дифференциальное уравнение Ответы: 1). (1 + y ) = tgx + c 2

4). ln y + 1 = tgx + c

sin x y' = cos x 1+ y

2). ln y + 1 = c sin x 5). −

3). 1 + y = c sin x

1 = ln c sin x (y + 1)2

Номер: 3.14.А

y2 + 1 , при начальном Задача: Решите дифференциальное уравнение y' = x ⋅ y условии y(0) = 1 2

⎛ x2 ⎞ + 2 ⎟⎟ − 1 Ответы: 1). y = ⎜⎜ ⎝ 2 ⎠ 4). y =

x2 −1 2

2

2). 2 y + 1 = ln x

3).

x2 y +1 = 2 2

x2 5). 2 y + 1 = + 2 2 2

Номер: 3.15.А Задача: Решите дифференциальное уравнение

( (

) )

6e x cos2 ydx + 1 − 2e x ctgydy = 0 1 2). 6e x cos 2 y + 1 − 2e x ln sin y = 0 Ответы: 1). 3 ln 1 − 2e x = cos y

(

3). tgy = c 1 − 2e x

(

)

(

)

3

5). 3 ln 1 − 2e x = −

(

)

)

4). 6e x cos 2 y − 1 − 2e x ⋅ ln sin y = 0

1 cos 2 y

Номер: 3.16.А Задача: Решите дифференциальное уравнение y' = 2 y ⋅ cos x

30


Ответы: 1).

y = 2 sin x + c

4). y = sin 2 x + c

2). 2 y = −2 sin x + c

3). y = 2 y sin x + c

5). y = 2 y y cos x + c

Номер: 3.17.А Задача: Решите дифференциальное уравнение 1 + x 2 ⋅ y'+1 + y 2 = 0 Ответы: 1). arcsin y = arcsin x + c 2). arctgy = −arctgx + c 3). y = c − x

(

(

)

((

4). ln 1 + x 2 = ln c 1 + y 2

))

)

5). y = tgx + c

Номер: 3.18.А Задача: Решите дифференциальное уравнение y' = e 2 x + 2 y Ответы: 1). 2 y = e 2 x + c

2). y =

4). 2 y = 2 x + c

5). e 2 y

1 2x 1 e + c 3). y = e 2 x + 2 y + c 2 2 = −e 2 x + c

Номер: 3.19.А Задача: Решите дифференциальное уравнение y' 1 − x 2 = 1 + y 2

(

3). arctgy = arcsin x + c

)

2). arcsin x = ln 1 + y 2 ⋅ c

Ответы: 1). arcsin y = arcsin x + c

4). arctgy = 2 1 − x 2 + c

5). y = tg 1 − x 2 + c

Номер: 3.20.А Задача: Решите дифференциальное уравнение y ⋅ y'+ x = 0

y2 x2 Ответы: 1). = − +c 2 2 2 x 4). y = − +c 2

2

2

2). y = x + c

x2 +c 2

3). y =

5). y = − x 2 + c

Номер: 3.21.А Задача: Решите дифференциальное уравнение x 2 y'+ y = 0 Ответы: 1). ln y = − 5). ln

1 +c x

2). y = c1e

−1

x

(

1 1 = +c y x

Номер: 3.22.А Задача: Решите дифференциальное уравнение

31

)

3). y = ln x 2 + 1 ⋅ c

xdx ydy = 1+ y 1+ x

4). y = c1e

1

x


( 1 + x ) x 2 (1 + y ) y 2 x2 y2 ⋅ = ⋅ +c 2). Ответы: 1). = +c 2 2 2 2 2 + 2 y 2 + 2x x 2 x 3 y 2 y3 3). + = + + c 4). y − ln 1 + y = x − ln 1 + x + c 2 3 2 3 2 1 x 1 y2 5). ⋅ = ⋅ +c 1+ y 2 1+ x 2 2

2

Номер: 3.23.А Задача: Решите дифференциальное уравнение y ln ydx + xdy = 0 Ответы: 1). ln y = x + c

2). y = e

c

x

3). y = ce− x

4). xy(ln y + 1) = c

5). y = ce x Номер: 3.24.А Задача: Решите дифференциальное уравнение y' x 3 = 2 y Ответы: 1). y =

c ex

2).

2

5). ln y = c −

1 c = y2 x 2

3). y = ce− x

2

4). ln y = c −

1 x2

1 x2 Номер: 3.25.А

dy dx = − 1 + y2 1+ x2 Ответы: 1). arctgy = −arctgx + c 2). arcsin y = arcsin x + c c c arctgx = ln 3). arctgy = ln 4). 5). arctgy = arctgx + c 1+ x2 1 + y2

Задача: Решите дифференциальное уравнение

Номер: 3.26.А Задача: Решите дифференциальное уравнение y'⋅ sin 2 x = y ⋅ ln y Ответы: 1). ln y = c ⋅ e − tgx 4). ln y = e

−1

2). ln ln y = −ctgx + c

3). ln y = cectgx

1 sin x

+c

5). ln y = e sin

2

x

+c

Номер: 3.27.А Задача: Найти общее решение дифференциального уравнения y'⋅tgx = y Ответы: 1). y = cectgx 5). y = −

2). y = cetgx

3). y = c sin x

1 sin 2 x 32

4). y = c ⋅ cos x


Номер: 3.28.А Задача: Решите дифференциальное уравнение y'⋅ sin x = y ln y

c Ответы: 1). ln y = sin 2 x 5). y = c ⋅ esin x

2). y = c ⋅ e

tgx

3). y = ce

cos x

4). y = ce

tg

x 2

Номер: 3.29.А Задача: Найти общее решение дифференциального уравнения x 2 y' = (x − 1) ⋅ y Ответы: 1). y = cx ⋅ e

1

x2 c 2). y = + y x

x

4). y = 3x 3 + c x

3). y =

x c + 3 x2

5). y = 3x x + cx 2

Номер: 3.30.А Задача: Найти общее решение дифференциального уравнения xy' = y ⋅ ln y 2

2). y = e x + cx 2

Ответы: 1). y = c ln x + x 4). y = cx 2 + x + ln x

5). y = e

x

3). y = e x + cx 2

c

Номер: 3.31.А Задача: Найти общее решение дифференциального уравнения xy'− y 2 + 1 = 0

1 + cx Ответы: 1). y = 1 + cx 2 c 1 − x2 5). y = 1 − cx

(

1 − cx 2 2). y = 1 + cx 2

)

cx 2 3). y = 1 − cx 2

1 − cx 3 4). y = 1 + cx 3

Номер: 3.32.В Задача: Найти общее решение дифференциального уравнения y'−e x − y + e x = 0 Ответы: 1). y = ln⎛⎜1 + ce

x

(

4). y = ln 2 + cex

2

2

)

⎞ ⎟ ⎠

(

2). y =

1 ex

)

ln 1 + ce x 5). y = − ln⎛⎜ 5 + ce 2 ⎞⎟ ⎝ ⎠

(

3). y = ln 1 + ce− e

Номер: 3.33.В Задача: Решите дифференциальное уравнение xydx + 1 − x 2 dy = 0 Ответы: 1). y = ce 2

x 2 −1

2). y = ce 2

1− x 2

4). yx 2 + y 1 − x 2 = c

3). y = cearcsin x 5). y = ce

33

1− x 2

x

)


Номер: 3.34.В

( (

) )

dy 1 + y 2 x = Задача: Решите дифференциальное уравнение dx 1 + x 2 y

(

)

(

)

Ответы: 1). 1 + y 2 x 2 = 1 + x 2 y 2 + c

(

3). 1 + y 2 = c 1 + x 2

2). y = c 1 + x 2 5). y 2 = x 2 + c

4). x = c 1 + y 2

Номер: 3.35.В Задача: Решите дифференциальное уравнение y 4 + e x dy = e x dx

(

Ответы: 1). y 2 = 2 ln c e x + 4

(

)

)

)

(

((

2). y 2 = ln c e x + 4

4). y 2 4 + e x = e x + c

(

)

))

)

(

3). y = 4 + e x

)

5). y = ln 4 + e x + c

Номер: 3.36.В Задача: Решите дифференциальное уравнение 1 − x 2 y' = xy Ответы: 1). y = ce1− x 5). ln y = −

(

2

2). y = c 1 − x 2

)

(

)

3). y =

c 1 − x2

4). ln y =

c

(1 − x )

2 2

c 1 − x2 Номер: 3.37.В

Задача: Решите дифференциальное уравнение Ответы: 1). ln x =

1

(1 + y )

2 2

2). cx =

1 1 + y2

dx ydy =− x 1 + y2 3). y = ctg(cx )

4). xc = 1 + y 2

5). cx = arctgy

Номер: 3.38.В Задача: Решите дифференциальное уравнение x + xy = y' (y − xy ) Ответы: 1). (y + 1) = ln c x + 1 2

3).

1+ x = x−y+c 1+ y

2). ln 1 + y = ln 1 + x c + y

4). x + ln x − 1 = − y + ln 1 + y + c

5).

1+ x =x+y+c 1− y

Номер: 3.39.В Задача: Решите дифференциальное уравнение ln(cos y )dx + xtgy ⋅ dy = 0 Ответы: 1). 2x ln(cos y ) = c 4). (x ⋅ c ) = ln cos y

2). cx = ln sin y

3). ln cos y =

5). нет правильного ответа 34

c x


Номер: 3.40.В Задача: Решите дифференциальное уравнение cos x ⋅ sin ydx − sin x ⋅ cos ydy = 0 Ответы: 1). 2 sin x sin y = c 4). sin x = c cos y

2). sin x = c sin y

3).

c 1 = sin 2 x sin 2 y

3). cos x = c sin y

Номер: 3.41.В Задача: Решите дифференциальное уравнение x 2 + x ⋅ y' = 2 y + 1

(

cx Ответы: 1). 2 y + 1 = x +1 x +1 4). 2 y + 1 = cx

2). 2 y + 1 = cx(x + 1) 5). 2 y + 1 =

)

⎛ cx ⎞ 3). 2 y + 1 = ⎜ ⎟ ⎝ x + 1⎠

2

c x2 + x

Номер: 3.42.В Задача: Найти общее решение дифференциального уравнения

(

)

y ⋅ e 2 x dx − 1 + e 2 x dy = 0 1 Ответы: 1). y = 1 + e2 x 4). y = 1 + e 2 x

(

2). y =

)

(

)

1 1 + e2x + c 2

3). y =

c 1 + e2 x

5). нет правильного ответа Номер: 3.43.В

Задача: Решите дифференциальное уравнение y'+

x sin x =0 y cos y

x2 y2 Ответы: 1). sin x − cos y = c 2). x ⋅ cos x + y sin y = c 2 2 − x cos x + sin x + c 3). y = 4). y sin y − cos y = x sin x − cos x + c y cos y 5). y ⋅ sin y + cos y = x sin x + cos x + c Номер: 3.44.В Задача: Решите дифференциальное уравнение x + xy + y ′(y + xy ) = 0 Ответы: 1). x + y = ln (c x + 1 ⋅ y + 1 )

3). x − y = ln(c(x + 1) ⋅ (y + 1)) 5). нет правильного ответа

2). x + y = ln 2 (c(x + 1))

4). y = ln(c(x + 1) ⋅ (y + 1)) − x 2

35


4. Однородные дифференциальные уравнения I порядка Номер: 4.1.А Задача: Решите дифференциальное уравнение y ⋅ y' = 2 y − x

x2 y2 = 2xy − + c Ответы: 1). 2 2 y−x y−x − = ln cx 3). ln x x

y x −1 − = ln cx x y−x x x2 4). y = 2 x − 5). = ln cx y−x 2y 2). ln

Номер: 4.2.А Задача: Решите дифференциальное уравнение xy' =

x 2 − y2 + y

2

y 1⎛ y⎞ 1 Ответы: 1). arcsin = ⎜ ⎟ + c 2). y = − x ⋅ sin 2 + c x 2⎝ x⎠ x 3). y = x ⋅ sin (ln cx ) 4). y = x cos(ln cx ) 5). нет правильного ответа Номер: 4.3.А Задача: Решите дифференциальное уравнение x 2 y' = (x − y )y

x Ответы: 1). y = ln cx y 5). = ln cx x

x2y 3). = y2 + c 2

x 2). = − ln cx y

4).

x 1 = ln y cx

Номер: 4.4.А Задача: Решите дифференциальное уравнение y' =

y y + sin x x

2

y ⎛ y⎞ y Ответы: 1). = ⎜ ⎟ − cos + c x ⎝x⎠ x 4). y = 2x ⋅ arcctgcx

2). y = 2x ⋅ arctgcx 5). y = 2 x ⋅ arcctg

3). y = 2x ⋅ arctg

1 cx

Номер: 4.5.А Задача: Решите дифференциальное уравнение xy'− y =

x arctg

36

y x

1 cx


2

y 1⎛ y⎞ y y y x2 2). y = yx + 3). ln = arctg + c Ответы: 1). − ⎜ ⎟ = arctg x 2⎝ x⎠ x 2arctgy x x y y 1 ⎛ x 2 + y2 ⎞ ⎟ = ln cx 5). нет правильного ответа 4). arctg − ln⎜⎜ 2 ⎟ x x 2 ⎝ x ⎠ Задача:

Найти

общее

Номер: 4.6.А решение дифференциального

уравнения

2

⎛ y⎞ ⎛ y⎞ y' = ⎜ ⎟ − ⎜ ⎟ + 1 ⎝x⎠ ⎝x⎠ Ответы: 1). y = ln cx + 1 4). y =

1 ln cx − x x

2). y = x +

x ln cx

3). y = x −

x ln cx

5). y = cx 3 + 2 ln x Номер: 4.7.А

⎛ 2y y2 ⎞ ⎜ Задача: Решите дифференциальное уравнение ⎜ 4 − 2 ⎟⎟dx + dy = 0 x x ⎝ ⎠ y2 1 3 2 2 Ответы: 1). y = cx + x 2). y = cx + x 3). 4 x + = c 4). y = cx 3 + x x 3 5). y = cx + 2 ln x Номер: 4.8.А Задача: Решите дифференциальное уравнение x 2 − 3y 2 dx + 2 xydy = 0

(

Ответы: 1). y = x + c ln x 4). y = x 2 +

c x

2). y 2 − x 2 = c ln 2 x

)

3). x 2 − cx = y

5). y 2 − x 2 = cx

Номер: 4.9.А Задача: Решите дифференциальное уравнение x 2 + y 2 dx − 2 xydy = 0

(

Ответы: 1). y = x + c ln x 4). y = x 2 +

c x

2). y 2 − x 2 = c ln 2 x

)

3). x 2 − cx = y

5). y 2 − x 2 = cx Номер: 4.10.А

Задача: Решите дифференциальное уравнение y' =

37

x + 3y 2x


1 = ln cx y −1 c 2 4). (x + y ) = 3 x

Ответы: 1). −

2). (x + y ) = cx 2

3). (x + y )2 = cx 3

2

5).

1 = cx 3 2 (x + y ) Номер: 4.11.А

x 2 + y2 Задача: Решите дифференциальное уравнение y' = xy 2 y2 x3 x y 2 2 + ln c 3). x ⋅ = + y2x + c Ответы: 1). y = 2 x (ln x + c ) 2). y = 2y x 2 3 ln x + c 2x 5). y = 4). y 2 = ln x + c x Номер: 4.12.А

dy 2 xy + 3y 2 Задача: Решите дифференциальное уравнение = dx x 2 + 2 xy y x 3y Ответы: 1). y = 2 ln − 2). xy − y 2 = cx 3 3). + y 2 = cx x y x x c 5). − y3 = 2 4). xy + y 2 = cx 3 y x Номер: 4.13.А

(

dy ) dx = xy − y

Задача: Решите дифференциальное уравнение x 2 + xy ⋅ Ответы: 1). y = x ⋅ ce 5). y = ce

x

x

y

2). xy = c ⋅ e

x

y

3). x = y ⋅ ce

y

x

2

4). y =

c yx e x

y

Номер: 4.14.А Задача: Решите дифференциальное уравнение xy' = x sin Ответы: 1). y = 2x + arctgcx 4). y = ln x + 2arctgcx

2). arctgcx = 2 x − y 5). arctgy = 2x + cy

y +y x

3). y = 2 xarctgcx

Номер: 4.15.А

xy + y 2 Задача: Найти общее решение дифференциального уравнения y' = x2 38


Ответы: 1). y = x ln cx

5). y = x ⋅ ln

2). y =

x c ln x

3). y =

ln cx x

c x 4). y = x ln

c x

Номер: 4.16.А Задача: Найти общее решение дифференциального уравнения x 3 y' = y x 2 + y 2

(

c x Ответы: 1). y = 2). y 2 = x 2 ln c x ln x 2 5). y ln cx = x 2 Задача:

Найти

общее

⎛y+x⎞ xy'− y = (x + y )ln⎜ ⎟ ⎝ x ⎠

)

2). y = x c1e x − 1

(

)

5). y =

1 c1e x − 1 x

x2 4). y = c x 2

Номер: 4.17.А решение дифференциального

(

Ответы: 1). y = x c1e x − 1 4). y =

ln cx 3). y = 2 x 2

(

)

(

)

3). y =

уравнения

x c1e x − 1

1 c1e x + 1 x

Номер: 4.18.А Задача: Найти общее решение дифференциального уравнения xy'− y = xtg Ответы: 1). y = x ⋅ arctgcx 4). y = cx ⋅ arcsin x

2). y = x ⋅ arctgcx 3). y = x ⋅ arccos(cx ) 5). y = x ⋅ arcsin (cx ) Номер: 4.19.А

Задача: Решите дифференциальное уравнение y' = Ответы: 1). y 2 = x 2 (2 ln cx )

x2 4). y = 2 ln cx 2

2). y = x ⋅ 2 ln cx 5). y = 2 ln cx − x 2

39

)

x y + y x 3). y = x 2 + 2 ln cx

y x


Номер: 4.20.А Задача: Решите дифференциальное уравнение x 2 + y 2 dx − xydy = 0

(

Ответы: 1). y 2 = 2 ln cx 2

2). y = x 2 ln cx

)

3). y 2 =

ln cx x2

x3 y2 2 5). + xy − x =c 3 2

2

4). y = x ln cx

Номер: 4.21.А Задача: Найти общее решение дифференциального уравнения

xy' cos

y y = y cos − x x x c x 4). y = x ⋅ arcsin x + c

2). y = x ln

Номер: 4.22.А Задача: Решите дифференциальное уравнение y' =

y y − = ln cx x x y y 2 4). ln + = ln (cx ) x x

Ответы: 1). ln

c x 5). y = x ⋅ ln cx

Ответы: 1). y = x ⋅ arcsin

3). y = x ⋅ arcsin⎜ ln

c⎞ ⎟ x⎠

x−y x+y

y y + = ln cx x x x x c 5). ln + = ln y y x

2). ln

3). ln

x x 1 + = ln y y cx

Номер: 4.23.А Задача: Найти общее решение дифференциального уравнения y − xy' =

x

y x ⎛ c⎞ 3). y = arcsin⎜ ln ⎟ ⎝ x⎠ cos

Ответы: 1). y = ln arcsin cx

4). y = x ⋅ arcsin⎜ ln

2). y = x ⋅ arcsin (ln cx )

1 ⎞ ⎟ cx ⎠

5). y = x ⋅ ln cx Номер: 4.24.А

xy − y 2 Задача: Решите дифференциальное уравнение y' = 2 x − 2 xy

40


y y Ответы: 1). − 2 ln = ln cx x x y y 4). + 2 ln = ln cx x x

x y 2). − 2 ln = ln cx y x x y 1 5). − 2 ln = ln y x cx

x y2 1 3). + 2 ln = ln y x cx

Номер: 4.25.А

y 1 + 2 x y ⎛ y⎞ 3⎜ ⎟ − 5 ⋅ + 1 x ⎝x⎠ 3 y y x2 ⎛ y⎞ 2). ⎜ ⎟ − 2,5 ⋅ + = +c x x x 2 ⎝ ⎠

Задача: Решите дифференциальное уравнение y′ =

3

y y ⎛ y⎞ Ответы: 1). ⎜ ⎟ − 2,5 ⋅ + = ln x ⋅ c x x ⎝x⎠ 3 y x2 ⎛ y⎞ +c 3). ⎜ ⎟ − 2,5 ⋅ + 1 = x 2 ⎝x⎠

3

y ⎛ y⎞ 4). ⎜ ⎟ − 2,5 ⋅ + 1 = ln x + c x ⎝x⎠

5). нет правильного ответа Номер: 4.26.А 2

y ⎛ y⎞ Задача: Решите дифференциальное уравнение y′ = + ⎜ ⎟ + 1 x ⎝x⎠ 2

y ⎛ y⎞ 2). + ⎜ ⎟ + 1 = x ⋅ c x ⎝x⎠

⎛ y⎞ Ответы: 1). arcsin⎜ ⎟ = ln x + c ⎝x⎠ 2

y ⎛ y⎞ 3). + ⎜ ⎟ + 1 = x + c x ⎝x⎠

⎛ y⎞ ⎝x⎠

4). arctg⎜ ⎟ = ln x + c

5). нет правильного ответа Номер: 4.27.А y

− y Задача: Решите дифференциальное уравнение y′ = + e x x y y c 2). = ln ln 3). y = x ⋅ ln ln x ⋅ c Ответы: 1). e x = − ln x + c x x 5). нет правильного ответа 4). y = − x ⋅ ln ln x ⋅ c

Номер: 4.28.А

y ⎛ y⎞ Задача: Решите дифференциальное уравнение y′ = + 4 − ⎜ ⎟ x ⎝x⎠

41

2


2

Ответы: 1). y = 4x ⋅ ln x + c ⋅ x

⎛ y⎞ 4 − ⎜ ⎟ = ln x + c ⎝x⎠

2).

2

y ⎛ y⎞ 3). + 4 − ⎜ ⎟ = x ⋅ c x ⎝x⎠

4). y = 2x ⋅ ln x + c ⋅ x

5). нет правильного ответа

Номер: 4.29.В Задача: Решите дифференциальное уравнение xy' = xe Ответы: 1). e

y

x

cx = 1 − cx

2). e

⎛ cx 2 ⎞ ⎟ 4). y = x ⋅ e⎜⎜ ⎟ 1 − cx ⎠ ⎝

y

x

cx 2 = 1 − ln x

y

x

+y+x

3). y = x ln

cx 1 + cx

2

5). y = x 2 ⋅ ecx + cx Номер: 4.30.B

Задача: Решите дифференциальное уравнение x ⋅ y′ = y +

x ⎛ y⎞ cos 3 ⎜ ⎟ ⎝x⎠

⎛ y⎞ ⎛ y⎞ ⎝x⎠ ⎝x⎠ ⎛ y⎞ ⎛ y⎞ ⎛ y⎞ ⎛ y⎞ 2). sin 3 ⎜ ⎟ − 3 ⋅ sin ⎜ ⎟ = 1,5x 2 + c 3). sin 3 ⎜ ⎟ − sin ⎜ ⎟ = 3 ln x + c ⎝x⎠ ⎝x⎠ ⎝x⎠ ⎝x⎠ ⎛ y⎞ ⎛ y⎞ ⎛ y⎞ ⎛ y⎞ 4). 3 sin 3 ⎜ ⎟ − sin ⎜ ⎟ = ln x + c 5). 3 sin ⎜ ⎟ − sin 3 ⎜ ⎟ = 3 ln x + c ⎝x⎠ ⎝x⎠ ⎝x⎠ ⎝x⎠

Ответы: 1). sin 3 ⎜ ⎟ − 3 ⋅ sin ⎜ ⎟ = 3 ln x + c

Номер: 4.31.B Задача: Решите дифференциальное уравнение x ⋅ y′ = y −

⎛ y⎞ ⎝x⎠

x ⎛ y⎞ sin 3 ⎜ ⎟ ⎝x⎠

⎛ y⎞ ⎝x⎠

Ответы: 1). 3 cos⎜ ⎟ − cos 3 ⎜ ⎟ = 3 ln x + c

⎛ y⎞ ⎛ y⎞ ⎝x⎠ ⎝x⎠ ⎛ y⎞ ⎛ y⎞ 4). 3 cos⎜ ⎟ − cos 3 ⎜ ⎟ = ln x + c ⎝x⎠ ⎝x⎠

2). − 3 cos⎜ ⎟ + cos 3 ⎜ ⎟ = 3 ln x + c

⎛ y⎞ ⎝x⎠

⎛ y⎞ ⎝x⎠

3). − 3 cos⎜ ⎟ + cos 3 ⎜ ⎟ = ln x + c 5). нет правильного ответа

42


Номер: 4.32.B

⎛ y⎞ ⎝x⎠

Задача: Решите дифференциальное уравнение x ⋅ y′ = y − x ⋅ ctg 2 ⎜ ⎟

y ⎛ y⎞ + tg⎜ ⎟ = ln x + c x ⎝x⎠ y ⎛ y⎞ 3). + ctg⎜ ⎟ = ln x + c x ⎝x⎠

Ответы: 1). −

y ⎛ y⎞ − tg⎜ ⎟ = ln x + c x ⎝x⎠ y ⎛ y⎞ 4). − + ctg⎜ ⎟ = ln x + c x ⎝x⎠ 2).

5). нет правильного ответа

43


5. Линейные дифференциальные уравнения I порядка. Уравнения Бернулли Номер: 5.1.А Задача: Найти общее решение дифференциального уравнения y'+

x2 +c Ответы: 1). y = ln x − 2 4 4). y = 2 + cx 2 x

2 y=x x

x2 c x2 c 2). y = 3). y = − + 4 x2 4 x2 x2 2 5). y = − +c 2 x2

Номер: 5.2.А Задача: Найти общее решение дифференциального уравнения y'+2 y = e3 x

e3x c Ответы: 1). y = + 3x 5 e e3x c 4). y = + 2x 3 e

e3x c e3x c 2). y = 3). y = + 2x − 2x 5 e 5 e 3x e c 5). y = + 2x 3 2e

Номер: 5.3.А Задача: Найти общее решение дифференциального уравнения y' e x + yex = 1 Ответы: 1). y = 1 + ce − x

2). y = e x + cx

4). y = e − x + ce x

3). y = e − x + ce x

5). y = 1 − ce x

Номер: 5.4.А Задача: Найти общее решение дифференциального уравнения y'− y ⋅ tgx = ctgx

c +1 cos x c −1 4). y = cos x

Ответы: 1). y =

2). y =

1 +c cos x

3). y =

c − cos x cos x

5). y = − cos x + c Номер: 5.5.А

Задача: Найти общее решение дифференциального уравнения y'− ytgx =

c cos x x+c 5). y = cos x

Ответы: 1). y =

2). y =

sin x x+c

3). y =

44

x+c sin x

4). y = cos(x + c )

1 cos x


Номер: 5.6.А Задача: Найти общее решение дифференциального уравнения y'+5 y = e 2 x

e2 x Ответы: 1). y = + ce−5 x 5 e2 x + ce− 5 x 4). y = 4

e2 x 2). y = + ce −5 x 7 e2 x 5). y = + ce − 5 x 3

e2 x 3). y = + c−5x 6

Номер: 5.7.А Задача: Найти общее решение дифференциального уравнения y'− y = x Ответы: 1). y = ce − x − x + 1 4). y = ce x − x + 1

2). y = ce x + x − 1

3). y = ce x + x + 1

5). y = ce x − x − 1

Номер: 5.8.А Задача: Найти общее решение дифференциального уравнения xy'+ y = x 3

x3 c Ответы: 1). y = + 4 x x c 5). y = + 3 3 x Задача:

Найти

(x + 1) y=

3).

3

2

− c(x + 1)

x3 c 3). y = − 4 x

x3 c 4). y = + 3 x

Номер: 5.9.А решение дифференциального

общее

(x + 1) ⋅ y'+ y = (x + 1)2 2 ( x + 1) + Ответы: 1). y = 2

x3 c 2). y = + 3 x

c x +1 4).

2). y = (x + 1) + 2

( x + 1)2 y= + 3

c x +1

c x +1

5).

( x + 1)2 y= −

Номер: 5.10.А Задача: Найти частное решение дифференциального уравнения

(x − 2) ⋅ y'− y = (x − 2)2 при условии y(4) = 10 ( ( x − 2 )3 x − 2 )3 Ответы: 1). y = + 3(x − 2 ) 2). y = − 3(x − 2 ) 3 3 ( x − 2) 3). y = + 3(x − 2 ) 2 ( x − 2 )2 + 3(x − 2) 5). y = 2

2 3 ( x − 2) 4). y = − 3(x − 2 ) 3

45

уравнения

3

c x +1


Номер: 5.11.А Задача: Найти общее решение дифференциального уравнения 2 xy'+ y = 2 x 3

x2 c Ответы: 1). y = + 7 x 3 4). y = 3x + c x

2 c x c + 2 3). y = x 3 + 7 3 x x 2 5). y = 3x x + cx 2). y =

Номер: 5.12.А Задача: Найти общее решение дифференциального уравнения xy'+ y = x ⋅ sin x Ответы: 1). y =

sin x − cos x +c x c cos x c + 4). y = sin 2 x − cos x + 5). y = sin x + x x x

sin x c − cos x + x x

3). y = sin x − cos x + cx

2). y =

Номер: 5.13.А Задача: Найти общее решение дифференциального уравнения xy'− y = Ответы: 1). y = cx 2 − x ln x

2). y = cx − x 2 ln ln x

x ln x

3). y = x ln x − c ⋅ ln x 2

5). y = cx + x 3 ln x

4). y = cx + x ln ln x

Номер: 5.14.А Задача: Найти общее решение дифференциального уравнения xy'− y = x 2 sin x

(

Ответы: 1). y = x 2 c − cos 2 x 4). y = x 2 (c − sin x )

)

2). y = x (c − cos x )

3). y = xc(cos x − sin x )

5). y = x (c − tgx ) Номер: 5.15.А

Задача: Найти общее решение дифференциального уравнения y'+2 xy = xe − x

x ⎞ Ответы: 1). y = e ⎜ + c ⎟ ⎝2 ⎠ x2 ⎛

4). y = e

Задача:

−x2 ⎛

⎞ x3 ⎜ + c⎟ ⎜ 3 ⎟ ⎝ ⎠

Найти

общее

x

(

2

2). y = e x + c 2

⎛1 ⎝2

2

⎛ x2 ⎞ + c ⎟⎟ 3). y = ln x ⎜⎜ ⎝ 2 ⎠

) ⎞ ⎠

5). y = e − x ⎜ x 2 + c ⎟ Номер: 5.16.А решение дифференциального

1 y'+ y cos x = sin 2 x 2

46

уравнения


Ответы: 1). y = cos x − 1 + ce − cos x 3). y = tgx − 1 + cesin x

2). y = sin x − 1 + ce − sin x

4). y = sin x − 2 + cecos x

5). y = sin x − cos x + ce x

Номер: 5.17.А Задача: Найти общее решение дифференциального уравнения

y'+ y cos x = e − sin x Ответы: 1). y = (x + c )e cos x

4). y = (x + c ) cos x

2). y = (2 x + c )e − x

5). y = (x + c )e − sin x

3). y = (3x + c ) ln x

Номер: 5.18.А Задача: Найти общее решение дифференциального уравнения y'+ ytgx = sin 2x Ответы: 1). y = −2 cos 2 x + c cos x 3). y = 3 cos x + c sin x

2). y = −2 cos 2 x + c sin x 5). y = −2 tg 2 x + ctgx

4). y = 2 tgx + c cos x

Номер: 5.19.В Задача: Найти общее решение дифференциального уравнения xy'+ xy 2 − y = 0

2x 2 2x 3 Ответы: 1). y = 2). y = 2 x+c x +c 4x 5). y = 2x 3 + c

2x x +c

3). y =

4). y =

2

2x 2x 2 + c

Номер: 5.20.В Задача: Решите дифференциальное уравнение y + xy 2 dx + 2 xydy = 0

(

x x Ответы: 1). + = c 2). y = cx 3 + x y 3 y x2 =c 5). 2 + 2 x

)

y x 3). + = c x 2

x x2 4). + =c y 2

Номер: 5.21.В

1 y = −y2 x (cx ) 4). y = ln x

Задача: Найти общее решение дифференциального уравнения y'+

1 x 2). y = x ln cx ln cx 1 5). y = 2 x ln cx

Ответы: 1). y =

3). y =

47

ln cx x


Номер: 5.22.В

(

2

)

2

Задача: Решите дифференциальное уравнение y'⋅e x = xe x − y 2 y

ex Ответы: 1). y = c + 2x cx 2 5). y = 2 ex

ln x 2). y 2 = c + 2x

2

2

ex 2 3). y = c + 2x

ce x 2 4). y = 2x 2

Номер: 5.23.В Задача: Найти общее решение дифференциального уравнения y'− xy 2 − 3xy = 0

⎛ x ⎞ Ответы: 1). y = 3⎜ ce 2 + 5 ⎟ ⎜ ⎟ ⎝ ⎠ ⎛ −x2 2 ⎞ − 1⎟⎟ 4). y = 3⎜⎜ ce ⎝ ⎠

⎞ ⎛ −3x 2 2 2). y = 3⎜⎜ ce − 1⎟⎟ ⎠ ⎝

−1

3). y = ce

−3 x

2

⎛ ⎛ x2 ⎞ ⎞ 5). y = 5⎜ c ln⎜⎜ ⎟⎟ + 3 ⎟ ⎜ ⎟ ⎝ ⎝ 2⎠ ⎠

Номер: 5.24.В Задача: Решите дифференциальное уравнение y'+2 x 3 y3 + 2 xy = 0

1 2

Ответы: 1). y 2 = − −

1 2x 2 + ce x2 2

4). y − 2

3). y = x 2 + e 2 x + c

1 2

5). y = − − 2x + ce 2 x

2 1 + x 2 + ce2 x 2 2 1 = − − x 2 + ce2 x 2

2). y =

3

Номер: 5.25.В Задача: Найти общее решение дифференциального уравнения

y'⋅ cos x + y ⋅ sin x = sin 2 x

Ответы: 1). y = cos x ⋅ ln 3). y = cos x ⋅ ln

c sin x

c cos 2 x

2). y = sin x ⋅ ln

4). y = cos x ⋅ ln c ⋅ cos 2 x

c sin 2 x 5). y = sin x ⋅ ln c ⋅ sin x

Номер: 5.26.В Задача: Найти общее решение дифференциального уравнения xy'+ y = ln x + 1

c +x x c 5). y = ln − x x

Ответы: 1). y = ln

2). y = ln x +

c x

48

3). ln x + cx = y

4). y = ln x + cx


Номер: 5.27.В Задача: Найти общее решение дифференциального 2 2 a + x y'+ xy = 1 , где a - произвольная постоянная

(

)

(

x+c x2 + a2 4). ln a 2 + x 2 c

2). y = (x + c ) x 2 + a 2

Ответы: 1). y =

(

)

(

5). y = c x 2 + a 2

)

3). y =

)

уравнения

−x+c x2 + a2

Номер: 5.28.В Задача: Решите дифференциальное уравнение (2 x + 1)y'+ y = x Ответы: 1). y = c 2 x + 1 − 3). y =

1 2

2x + 1 1 c − + 6 2 2x + 1

2). 2 x + 1 =

c 2x + 1

4). y = c ln 2 x + 1

5). y = 2 x + 1 − c 2 x + 1

Номер: 5.29.В Задача: Найти общее решение дифференциального уравнения

xdy − 2 ydx = x 3 ln xdx Ответы: 1). y = x 3 ln x + x 3 + cx 2 3). y = ln x + x 3 + cx

2). y = ln x + x 3 + cx 2

4). y = x 3 ln x − x 3 + cx 2

5). y = x 3 ln x − x 3 + cx

Номер: 5.30.В Задача: Решите дифференциальное уравнение 3x 2 + 2 y dx + (2 x − 3)dy = 0

(

Ответы: 1). x 3 + 2 xy = c

2). x 3 + 4 xy − 3y = c

4). x 2 + 2 xy = c

5). x 3 + 2 xy − 3y = c

)

3). 2 xy − 3y = c

Номер: 5.31.В Задача: Решите дифференциальное уравнение x + y 2 dx + xydy = 0

(

3

2 2

Ответы: 1). 2 x + 3x y = c 4). x 2 + y 2 + xy 2 = c

x 2 3xy 2 2). + =c 2 2 x2 2 5). 2 y x + =c 2

)

3). x 2 + y 2 + xy 2 = c

Номер: 5.32.В Задача: Решите дифференциальное уравнение x 2 + 2 y dxx − xdy = 0

(

49

)


x3 Ответы: 1). + xy = c 3 y 4). ln x + 2 x =c

x3 3). − xy = c 3

y 2). ln x = 2 + c x x2 5). ln x = +c y Номер: 5.33.В

y = xy 2 x 1 3). y = cx − x 2

Задача: Решите дифференциальное уравнение y'+ Ответы: 1). y = cx − x 2 5). y =

2). y =

1 x 2 + cx

4).

1 c = − x2 y x

c − x2 x

Номер: 5.34.В Задача: Решите дифференциальное уравнение x 3 y'− x 2 y = − cos x ⋅ y 4

x2 Ответы: 1). y = 2 sin x + c x3 3 4). y = 3 sin x + c 2

2). y =

x sin x + c

3). y = y ln x − y 4 sin x + c

5). y3 = x 3 (3 sin x + c )

Номер: 5.35.В Задача: Найти общее решение дифференциального уравнения

3y'−2 y = −e3x ⋅ y 4

2 xy e −3x 4 + ⋅y Ответы: 1). y = 3 3 2 xy e −3x 4 4). y = − y 3 3

6x 2). y = 2 x 3 − 3x 2 + c 6x 5). y = 2 x 3 + 3x 2 + c

50

x2 x 3). y = + +c 3 2


6. Уравнения в полных дифференциалах Задача:

(y

2

)

x

Решите

(

)

x

Номер: 6.1.А дифференциальное

уравнение

− e cos y dx + 2 xy + e sin y dy = 0

Ответ: 1). xy 2 + e x sin y = c

2). x 2 y − e x cos y = c

y2 4). x − e x cos y = c 2 Задача:

5). 2 xy 2 − e x cos y + e x sin y = c Номер: 6.2.А дифференциальное

Решите

(x ⋅ cos 2 y + 1)dx − x

2

3). xy 2 − e x cos y = c

уравнение

⋅ sin 2 ydy = 0

x2 cos 2 y + c = 0 Ответ: 1). x cos 2 y + x = c 2). 2 x3 x2 x cos 2 y + x + x 2 y cos 2 y = c 3). − sin 2 y + y + sin 2 y = c 4). 2 3 2 x2 cos 2 y + x = c 5). 2 2

Номер: 6.3.А Задача: Решите дифференциальное уравнение e y dx + xe y 2 y dy = 0

(

Ответ: 1). xe y = c

2). xe y + y 2 = c

3). xe y − y 2 = c

)

4). 2 xe y − y 2 = c

x2 y 5). xe − e − 2 xy = c 2 y

Номер: 6.4.А Задача: Решите дифференциальное уравнение x 2 + y 2 + 2 x dx + 2 xydy = 0

(

)

x3 Ответ: 1). + y2x + x 2 = c 3 4). x 2 y + xy 2 + 2 xy = 0

x3 2). + 2xy 2 + x 2 = c 3 5). x 2 y = c

Задача:

Номер: 6.5.А дифференциальное

(x

3

)

Решите

(

)

x3 3). 2 y + + x2 = c 3 2

уравнение

− 3xy 2 + 2 dx − 3x 2 y − y 2 dy = 0

x4 3 2 2 Ответ: 1). − x y + 2x = c 4 2

3 2 2 y3 2). x y − =c 3 2

51

3). x 3 y − xy 3 + 2 y = c


y3 x4 3 2 2 4). − x y + 2x + =c 3 4 2

5). x 3 y − y 2 x = c

Номер: 6.6.А

2x y 2 − 3x 2 Задача: Решите дифференциальное уравнение 3 dx + dy = 0 y y4 2

x 2 y 3 − 3x y Ответ: 1). 3 + =c y y4 3x x 3 4). + =c 2y4 y4

1 3x 2 3). − =c 3y 3 5 y 5

x2 1 2). 3 − 3 = c y 3y x2 5). 3 + c = 0 y

Номер: 6.7.А Задача: Решите дифференциальное уравнение ln (cos y )dx − xtgydy = 0 Ответ: 1). − 2 xtgy = c

x2 2). x ln (cos y ) − tgy = c 2

x2 tgy = c 3). y ln (cos y ) − 2 Задача:

4). x ln cos x = c

Номер: 6.8.А дифференциальное

Решите

5). x ln cos y − x ln sin y = c

⎞ + x + 1⎟⎟dy = 0 ⎝ 2y ⎠ x2 x2 x2 Ответ: 1). xy + ln y = c 2). ln y + +x =c 2 2 2 x3 x 2 x2 5). xy + ln y + y = c 4). + +x =c 6y 2 2

(y + x ln y )dx + ⎜⎜ x

Задача:

Решите

(3x y + sin x )dx + (x 2

уравнение

2

3

)

y2 3). +x+y=c 2

Номер: 6.9.А дифференциальное

уравнение

− cos y dy = 0

Ответ: 1). x 3 y − cos x − sin y = c

3 4). x 2 y 2 + y sin x = c 2

2). x 3 y − cos x = c

3). x 3 y − sin y = c

x4 5). − x cos y = c 4

Номер: 6.10.А Задача: Решите дифференциальное уравнение cos x ⋅ sin ydx + sin x ⋅ cos ydy = 0 Ответ: 1). − cos x sin y + cos y sin x = c 2). sin x sin y = c 52


3). sin x sin y − cos x cos y = c 5). sin x cos y − cos x sin y = c

4). cos x cos y − sin x sin y = c

Номер: 6.11.А Задача: Решите дифференциальное уравнение x 2 + y 2 + y dx + (2 xy + x )dy = 0

(

)

x3 y2 y3 y 2 x 2 2 Ответ: 1). + 2 xy + 3x = c 2). 2 x y + + + =c 3 2 3 2 2 x3 + y 2 x + xy = c 4). x 2 + y 2 + y + 2 xy + x = c 5). x 2 y + y 2 x + 2 xy = c 3). 3 Номер: 6.12.А Задача: Решите дифференциальное уравнение

(2x + sin y − ye )dx + (x ⋅ cos y + cos y + e )dy = 0 −x

−x

Ответ: 1). x 2 + x sin y + ye− x + sin y = c 3). x sin y + sin y + ye− x = c

2). x 2 + x sin y + ye− x = c 4). x 2 + x sin y + e − x ⋅ y − sin y = c

5). − sin y − sin y + e − x = c Номер: 6.13.А Задача: Решите дифференциальное уравнение xy 2 + y dx + x 2 y + x dy = 0

(

x2 2 y + xy = c 2). x 2 y 2 + xy = c Ответ: 1). 2 4). x 2 y 2 + ln x + ln y = c

)

(

)

3). 2 xy + x 2 y 2 = c 5). ln xy ⋅ c = x 2 y 2

Номер: 6.14.А Задача: Решите дифференциальное уравнение

⎛ x ⎞ − arccos y ⋅ dx + ⎜ + 2 y ⎟ ⋅ dy = 0 ⎜ 1 − y2 ⎟ ⎝ ⎠ 2). x 4 + arccos y + y 2 = c Ответы: 1). x 4 − arccos y + y 2 = c

(4x

)

3

3). 12 x 2 − x ⋅ arccos y + y 2 = c

4). 12 x 2 + x ⋅ arccos y + y 2 = c

5). x 4 − x ⋅ arccos y + y 2 = c

Задача:

(5x

4

2

)

Решите

(

y

2

)

Номер: 6.15.А дифференциальное

− 2 x ⋅ y ⋅ dx + e − 2 x ⋅ y ⋅ dy = 0

Ответы: 1). 20 x 3 − x 2 y 2 + e y = c

2). x 5 − x 4 + e y = c

53

уравнение


x 2 ⋅ y3 3). x − + ey = c 3 5

Задача:

4). x 5 − x 2 ⋅ y 2 + e y = c

Номер: 6.16.А дифференциальное

Решите

(y + cos x + 2xy )⋅ dx + (2 2

y

5). нет правильного ответа

)

2

уравнение

⋅ ln 2 + x + 2x y ⋅ dy = 0

Ответы: 1). − sin x + x 2 y 2 + xy + 2 y = c

2). sin x + x 2 y 2 + xy + 2 y = c

y2 3). sin x + x y + + 2y = c 2 2 2

4). − sin x + x 2 y 2 + xy + ln 2 2 ⋅ 2 y = c

5). нет правильного ответа Задача:

Решите

Номер: 6.17.А дифференциальное

уравнение

⎛ x ⎞ ⎛1 ⎞ ⎜ + ctgy ⎟ ⋅ dx + ⎜⎜ 2 y − 2 ⎟⎟ ⋅ dy = 0 sin y ⎠ ⎠ ⎝x ⎝ Ответы: 1). ln x + x ⋅ ctgy + y 2 = c 3). −

1 + ctgy + y 2 = c 2 x

2). ln x + x ⋅ ctgy − ctgy + y 2 = c 4). −

5). нет правильного ответа Задача:

Решите

1 + x ⋅ ctgy + y 2 = c 2 x

Номер: 6.18.А дифференциальное

⎛ ⎛1 x ⎞ x⎞ ⎜⎜ + e − y ⎟⎟ ⋅ dx + ⎜⎜ cos y − x − 2 ⎟⎟ ⋅ dy = 0 y ⎠ ⎝y ⎠ ⎝ x x 2). e x − y + − sin y = c Ответы: 1). e x − y + + sin y = c y y x 3). e x − xy + + sin y = c 4). e x − xy + ln y + sin y = c y 5). нет правильного ответа Номер: 6.19.А Задача: Решите дифференциальное уравнение

⎛ x 1 x2 ⎞ ⎛ ⎞ ⎜ − sin y + ⎟⎟ ⋅ dy = 0 + 2 y ⎟ ⋅ dx + ⎜ ⎜ xy + 2 2 ⎠ x −1 ⎝ ⎠ ⎝ y x −1 Ответы: 1). ln + 2x y + x 2 y + 2 cos y = c x +1

54

уравнение


x −1 + 4 x y + x 2 y − 2 cos y = c x +1 x −1 5). ln + 4 x y + x 2 y + 2 cos y = c x +1

x −1 + 4 x y + x 2 y − 2 cos y = c x +1 1 x −1 4). ln + 4x y + x 2 y + 2 cos y = c 2 x +1 2). ln

Задача:

Решите

3). ln

Номер: 6.20.А дифференциальное

уравнение

⎛ 2 x ⎞ y ⎞ ⎛ ⎜ ⎟ ⋅ dy = 0 ⋅ dx + 3 y + + tgx ⎜ 2x + xy + ⎟ 2 ⎜ ⎟ 2 cos x ⎠ ⎝ ⎝ ⎠ 2

Ответы: 1). x 2 + 0,5xy 2 + y ⋅ tgx + y3 = c

2). x 2 + 0,5x 2 y + y ⋅ tgx + y3 = c

3). x 2 + 0,5x 2 y + y ⋅ ctgx + y3 = c 5). нет правильного ответа Задача:

Решите

4). x 2 + 0,5x 2 y − y ⋅ ctgx + y3 = c

Номер: 6.21.А дифференциальное

(cos x − y + x ⋅ e )⋅ dx + (1 − x + 0,5 ⋅ x y

2

)

⋅ e y ⋅ dy = 0

Ответы:1). sin x − xy + 0,5 ⋅ x 2 ⋅ e y + y = c

2). − sin x − xy + 0,5 ⋅ x 2 ⋅ e y + y = c

3). cos x − xy + 0,5 ⋅ x 2 ⋅ e y + y = c 5). нет правильного ответа Задача:

Решите

уравнение

4). cos x − xy + 0,5 ⋅ x 2 ⋅ e y + 1 = c

Номер: 6.22.А дифференциальное

уравнение

⎛x ⎞ + x ⋅ ln y ⋅ dx + ⎜⎜ − 3y 2 ⎟⎟ ⋅ dy = 0 ⎝ 2y ⎠ 4 2 Ответы: 1). x + 0,5x ⋅ ln y − y 3 x = c 2). x 4 + 0,5x 2 ⋅ ln y − y3 = c

(4x

)

3

2

3). 12x 2 + 0,5x ⋅ ln y − y3 = c 5). нет правильного ответа

4). 12x 2 + 0,5x 2 ⋅ ln y − y 3 = c

Номер: 6.23.А Задача: Решите дифференциальное (xy + 5x + 0,5y 2 )⋅ dx + ⎛⎜⎜ 0,5x 2 + 21 + xy ⎞⎟⎟ ⋅ dy = 0 y +1 ⎝ ⎠ 2 x 2 Ответы: 1). x y + 2 ⋅ 5 ⋅ ln 5 + x ⋅ y + 2 ⋅ arctgy = c

5x + 2 ⋅ arctgy = c 2). x y + xy + ln 5 4). x 2 y + xy 2 + 2 ⋅ 5 x + 2 ⋅ arctgy = c 2

2

уравнение

2 ⋅ 5x 3). x y + xy + + 2 ⋅ arctgy = c ln 5 2

2

5). нет правильного ответа

55


Задача:

Решите

Номер: 6.24.А дифференциальное

уравнение

⎛ 2 ⎞ 1 1 ⎛ 2⎞ ⎜ ⎟ ⋅ dy = 0 + − ⋅ ⋅ + + − 2 xy 0 , 5 y dx x xy ⎜ ⎟ 2 ⎜ ⎟ x +1 ⎝ ⎠ y −1 ⎝ ⎠ Ответы: 1). x 2 ⋅ y − 0,5 ⋅ x ⋅ y 2 + ln x + 1 + ln y + y 2 − 1 = c 2). x 2 ⋅ y − 0,5 ⋅ x ⋅ y 2 + ln x + 1 + arcsin y = c 3). x 2 ⋅ y − 0,5 ⋅ x ⋅ y 2 −

1 + arcsin y = c (x + 1)2

4). x 2 ⋅ y − 0,5 ⋅ x ⋅ y 2 + ln x + arcsin y = c 5). нет правильного ответа Задача:

Решите

Номер: 6.25.А дифференциальное

уравнение

⎛ y2 ⎞ ⎛ 1 ⎞ 1 ⎜ ⎟ ⋅ dx + ⎜⎜ y ⋅ ln x − 0,5x 2 − 2 ⎟⎟ ⋅ dy = 0 + − xy ⎜ 2x ⎟ x sin y ⎠ ⎝ ⎝ ⎠ Ответы: 1). y 2 ⋅ ln x + 4 x − x 2 ⋅ y + y − x + ctgy = c 2). y 2 ⋅ ln x + 4 x − x 2 ⋅ y + y − x + 2ctgy = c 3). y 2 ⋅ ln x + 2 x − x 2 ⋅ y + y − x − ctgy = c 4). y 2 ⋅ ln x + 2 x − x 2 ⋅ y + y − x + 2ctgy = c 5). нет правильного ответа Задача:

Решите

Номер: 6.26.А дифференциальное

уравнение

⎞ ⎛ 1 x + y ⋅ sin x ⋅ dx + ⎜⎜ 2 − cos x ⎟⎟ ⋅ dy = 0 ⎠ ⎝y −4 y−2 =c Ответы: 1). 18x x − 12 y ⋅ cos x + 3 ⋅ ln y+2

(

2).

)

4 y−2 − 12 y ⋅ cos x + 3 ⋅ ln =c y+2 x

4). 8x x − 12 y ⋅ cos x + 3 ⋅ ln

y−2 =c y+2

3). 18x x + 12 y ⋅ cos x + 3 ⋅ ln 5). нет правильного ответа

56

y−2 =c y+2


Номер: 6.27.А Задача: Решите дифференциальное уравнение

⎛ ln x ⎞ 1 + ln y ⋅ dx + ⎜⎜ − e 2 y ⎟⎟ ⋅ dy = 0 x ⎝ y ⎠

Ответы: 1). ln x ⋅ (1 + ln y ) − 0,5 ⋅ e 2 y = c

2). ln x ⋅ (1 + ln y ) − e 2 y = c

3). ln x ⋅ (1 + ln y ) − 2 ⋅ e 2 y = c 5). нет правильного ответа

Задача:

4). ln x + ln y − 2 ⋅ e 2 y = c

Номер: 6.28.А дифференциальное

Решите

уравнение

⎛ 1 y ⎞ y⎞ ⎛ ⎜ ⎟ + + − dx sin y dy = 0 ⎜ 2⎟ ⎜ x 2x 2 ⎟ x ⎝ ⎠ ⎝ ⎠ y2 y2 2). 4 x − Ответы: 1). 4 x − + cos y = c − 2 cos y = c x x y2 y2 3). 4 x − + 2 cos y = c 4). 4 x + + 2 cos y = c x x 2

5). нет правильного ответа

Задача:

(xy

Номер: 6.29.В дифференциальное

Решите

)

⎛ 1 ⎞ + sin 2 x dx = −⎜⎜ + x 2 ⋅ y ⎟⎟dy ⎝ y+3 ⎠ 2 Ответы: 1). 2 cos 2x − x 2 y 2 − ln (y + 3) = c 2

3). 2 cos 2 x + x 2 y 2 + ln (y + 3) = c 5). нет правильного ответа 2

Задача:

уравнение

Решите

(

)

2). cos 2 x + x 2 y 2 + ln (y + 3) = c 2

4). cos 2x − x 2 y 2 − ln (y + 3) = c

Номер: 6.30.В дифференциальное

2

уравнение

y ⎞ ⎛ ⎜ tgx + 2 ⎟dx + arctgx + 3 y dy = 0 x + 1⎠ ⎝ Ответы: 1). y ⋅ arctgx − ln cos x + 0.75 y ⋅ 3 y = c 2). y ⋅ arctgx + ln cos x + 0.75 y ⋅ 3 y = c

3). y ⋅ arctgx + ln sin x + 0.75 y ⋅ 3 y = c

4). arctgx + ln sin x + 0.75 y ⋅ 3 y = c

5). нет правильного ответа

57


Задача:

Решите

Номер: 6.31.В дифференциальное

уравнение

⎛ x ⎞ ⎛ ln x ⎞ 2y ⎟ ⎜ e dy = 0 − arcsin y dx − + ⎜ ⎟ 2 ⎜ ⎟ x ⎝ ⎠ ⎝ 1− y ⎠ Ответы: 1). 2 x ⋅ arcsin y − ln 2 x + 2 ⋅ e 2 y = c 3). 2 x ⋅ arcsin y − ln 2 x + e 2 y = c 5). нет правильного ответа Задача:

(2

Решите

2). 2 ⋅ arcsin y − ln 2 x + 2 ⋅ e 2 y = c 4). 2 x ⋅ arcsin y + ln 2 x − 0.5 ⋅ e 2 y = c

Номер: 6.32.В дифференциальное

уравнение

)

⎞ ⎛ 1 + y ⋅ cos(x ⋅ y ) dx + ⎜⎜ + x ⋅ cos(x ⋅ y )⎟⎟dy = 0 ⎠ ⎝ y 2x 2x + 2 y = c 2). sin (x ⋅ y ) + +2 y =c Ответы: 1). y ⋅ sin (x ⋅ y ) + ln 2 ln 2 2x − sin (x ⋅ y ) + 2 y = c 4). нет правильного ответа 3). ln 2 2x + sin (x ⋅ y ) + y = c 5). ln 2 x

Задача:

Решите

Номер: 6.33.В дифференциальное

уравнение

⎛ 1 ⎞ 2y ⎞ ⎛ y ⎜ ⎟⎟dy = 0 − + + dx tgx ⎜ ⎟ 2 2 ⎜ + y 9 ⎝ cos x x + 2 ⎠ ⎝ ⎠ y2 + 9 =c Ответы: 1). y ⋅ tgx + ln x+2 2 x 3). y ⋅ tgx − ln x + 2 + ⋅ arctg = c 3 2

2). y ⋅ tgx + ln

(

x+9 =c y2 + 2

)

4). ln y 2 + 9 − ln x + 2 = c

5). нет правильного ответа

Задача:

Решите

Номер: 6.34.В дифференциальное

уравнение

⎞ ⎛ x ⎛ arccos x ⎞ ⎜⎜ + 2 y ⎟⎟dx + ⎜⎜ − y 2 ⎟⎟dy = 0 2 ⎝ 1− x ⎠ ⎠ ⎝ y 2 Ответы: 1). 12 x ⋅ y + 3 ⋅ arccos x − 2 y3 = c 2). 8 ⋅ y y − 3 ⋅ arccos2 x − 2 y3 = c

3). 12 x ⋅ y − 3 ⋅ arccos2 x − 2 y3 = c

4). нет правильного ответа

5). 12 ⋅ y + 3 ⋅ arccos2 x − 2 y3 = c 58


Задача:

Решите

Номер: 6.35.В дифференциальное

⎛ 1 ⎛ x 1⎞ x⎞ ⎜⎜ cos − ⎟⎟dx + ⎜⎜ 2 + 2 ⎟⎟dy = 0 2 y⎠ ⎝ ⎝y −4 y ⎠ y−2 Ответы: 1). 4 sin 0.5x − 4xy + ln y+2 3). 8 sin 0.5x − 4 xy + ln

y−2 y+2

уравнение

2). − 4 sin 0.5x − 4 xy + ln

y−2 y+2

4). 8 sin 0.5x − 4 xy + 2 ⋅ ln

y−2 y+2

5). нет правильного ответа Задача:

Решите

Номер: 6.36.В дифференциальное

уравнение

⎛ 1 x ⎞ ⎛ 1 ⎞ − ctgy ⎟dx + ⎜⎜ + 2 ⎟⎟dy = 0 ⎜ 2 ⎠ ⎝x +9 ⎝ y ⋅ ln y sin y ⎠ x x Ответы: 1). arctg − 3x ⋅ ctgy + ln ln 3 y = c 2). arctg − x ⋅ ctgy + ln ln y = c 3 3 4). arctgx − 3x ⋅ ctgy + ln ln y = c 3). arctgx − x ⋅ ctgy + ln ln y = c 5). нет правильного ответа Задача:

Решите

Номер: 6.37.В дифференциальное

уравнение

⎛ 1 ⎞ ⎛ 1 2 ⎞ 2 ⎟ ⎜ + x y dy = 0 ⎜ 2 + y x ⎟dx + ⎜ 1 − y2 ⎟ ⎝x ⎠ ⎝ ⎠ Ответы: 1). x 2 y 2 + 2 x + 2 ⋅ arcsin y = c

2). x 2 y 2 − 2 x + 2 ⋅ arcsin y = c

3). xy 2 − 2 x + 2 ⋅ arcsin y = c 5). нет правильного ответа Задача:

Решите

4). xy 2 + 2 x + 2 ⋅ arcsin y = c

Номер: 6.38.В дифференциальное

⎛ ey x ⎞⎟ ⎜ + dy = 0 x ⋅ x + arcsin y dx + 2 ⎟ ⎜ 1 − e2 y 1 − y ⎝ ⎠ Ответы: 1). 2 x ⋅ x + 5x ⋅ arcsin y + 5 ⋅ arcsin e y = c

(

)

( ) x + x ⋅ arcsin y + arcsin (e ) = c x + arcsin y + arcsin (e ) = c

2). 0.4 x ⋅ x + x ⋅ arcsin y + arcsin e y = c 3). 0.4 x 2 ⋅

4). 0.4 x 2 ⋅ 5). нет правильного ответа

y

y

59

( )

уравнение


7. Обыкновенные дифференциальные уравнения n-ого порядка. Типы уравнений. Теория. Номер: 7.1.А Задача: Всякая … функция, которая обращает дифференциальное уравнение F x , y, y ′, ... , y (n ) = 0 в тождество называется решением этого уравнения. Вставьте пропущенные слова. Ответы: 1). n раз дифференцируемая 2). непрерывная 3). неотрицательная 4). удовлетворяющая начальным условиям 5). ничего не надо вставлять

(

)

Номер: 7.2.А Задача: Решением дифференциального уравнения F x , y, y ′, ... , y (n ) = 0 называется функция, которая Ответы: 1). n раз дифференцируема и обращает уравнение в тождество 2). удовлетворяет начальному условию задачи Коши 3). при подстановке в уравнение понижает его порядок 4). имеет вид y = ϕ(x ) 5). имеет вид y = ϕ(x , c1 , c 2 , ... , c n )

(

Задача:

Функция

Номер: 7.3.А y = ϕ(x , c1 , c 2 , ... , c n ) называется

(

)

общим

)

решением

дифференциального уравнения F x , y, y ′, y ′′, ... , y (n ) = 0 , если она Ответы: 1). удовлетворяет данному уравнению при любых c1 , c 2 , ... , c n 2). обращает уравнение в тождество 3). при соответствующем выборе c1 , c 2 , ... , c n является решением любой задачи Коши, поставленной для данного уравнения 4). n раз дифференцируема 5). n раз интегрируема Номер: 7.4.А Задача: Решение дифференциального уравнения y (n ) = f (x ) находится Ответы: 1). n – кратным интегрированием 2). n – кратным дифференцированием 3). подстановкой y ′ = z(x ) 4). подстановкой y ′ = p(y ) 5). методом вариации произвольных постоянных Номер: 7.5.А Задача: Решение дифференциального уравнения y (n ) = f (x ) имеет вид

60


Ответы: 1). y = ∫∫ ... ∫ f (x )dx n 123

2). y = C1 x n −1 + ... + C n −1 ⋅ x + C n

n ðàç

4). y = f (n ) (x )

3). y = ∫∫ ... ∫ f (x )dx n + C1 x n −1 + ... + C n −1 ⋅ x + C n 123 n ðàç

5). y = f (n ) (x ) + C1 + C 2 + ... + C n Номер: 7.6.А Задача: Дифференциальное уравнение вида F x , y (k ) , y (k +1) , ... , y (n ) = 0 , не содержащие искомой функции y(x ) решается методом понижения порядка с помощью Ответы: 1). замены y (k ) = z(x ) , тогда z (k +1) = z ′ и т.д.

(

)

2). замены y (k ) = z(x ) , тогда z (k +1) = z ⋅ z ′ и т.д.

3). замены y (k ) = p(y ) , тогда z (k +1) = p ′ и т.д.

4). замены y (k ) = p(y ) , тогда z (k +1) = p ⋅ p ′ и т.д. 5). n – кратного интегрирования

Номер: 7.7.А Задача: Порядок дифференциального уравнения (k ) (k +1) (n ) (k ) F x, y , y , ... , y = 0 после замены y = t понижается Ответы: 1). 1 единицу 2). k единиц 3). (n − k ) единиц 4). (n − k + 1) единиц 5). n единиц

(

вида

)

Номер: 7.8.А Задача: Дифференциальное уравнение F x , y (k ) , y (k +1) , ... , y (n ) = 0 после

(

замены y (k ) = t примет вид

(

)

Ответы: 1). F t , t ′,..., t (n − k ) = 0 3). F(x , t , t ⋅ t ′,...) = 0

(

(

)

)

2). F x , t , t ′,..., t (n − k ) = 0

)

4). F x , t , t ′,..., t (n − k +1) = 0

(

)

5). F t , t ′,..., t (n − k ) = 0

Номер: 7.9.А Задача: Дифференциальное уравнение вида F y, y ′, y ′′, ... , y (n ) = 0 не содержащие явно независимую переменную x решается методом понижения порядка с помощью Ответы: 1). замены y ′ = z(x ) , тогда y ′′ = z ′ и т.д. 2). замены y ′ = z(x ) , тогда y ′′ = z ⋅ z ′ и т.д. 3). замены y ′ = p(y ) , тогда y ′′ = p ′ и т.д. 4). замены y ′ = p(y ) , тогда y ′′ = p ′ ⋅ p и т.д. 5). n – кратного интегрирования

(

61

)


Номер: 7.10.А Задача: Порядок дифференциального уравнения вида F y, y ′, y ′′, ... , y (n ) = 0 после замены y ′ = t понижается на Ответы: 1). 1 единицу 2). n единиц 3). (n − 1) единиц 4). (n + 1) единиц 5). 0 единиц

(

)

Номер: 7.11.А Задача: Дифференциальное уравнение F y, y ′, y ′′, ... , y (n ) = 0 не содержащие явно независимую переменную x после замену y ′ = t примет вид

(

( ) 3). F(t , t ′, ... , t ( ) ) = 0 5). F(y, t , t ′, ... , t ( ) ) = 0

Ответы: 1). F y, t , t ′, t ′′, ... , t (n −1) = 0

)

2). F(y, t , t ⋅ t ′, ...) = 0

4). F(t , t ⋅ t ′, ...) = 0

n −1

n

Номер: 7.12.А Задача: Дифференциальное уравнение n-го порядка называется линейным, если Ответы: 1). оно первой степени относительно искомой функции y(x ) 2). оно первой степени относительно независимой переменной x 3). оно первой степени относительно y, y ′, y ′′,..., y (n ) 4). решение уравнения есть функция только первой степени относительно x 5). решение уравнения первой степени относительно x , c1 , c 2 ,..., c n Номер: 7.13.А Задача: Линейное однородное дифференциальное уравнение n-го порядка имеет вид Ответы: 1). a 0 y + f x , y ′, y ′′,..., y (n ) = 0 2). a 0 x + f y, y ′, y ′′,..., y (n ) = 0

(

3). a 0 y (n ) + a 1 y (n −1) + ... + a n y = 0

)

5). a 0 y (n ) + a 1 y (n −1) + ... + a n y = x Задача: Если

y1 , y 2 , ... , y n

(

)

4). x + a 0 y (n ) + a 1 y (n −1) + ... + a n y = 0

Номер: 7.14.А - линейно независимые частные решения

дифференциального уравнения a 0 y (n ) + a 1 y (n −1) + ... + a n y = 0 , то общее решение этого уравнения имеет вид Ответы: 1). y = c1 ⋅ y1 + c 2 ⋅ y 2 + ... + c n ⋅ y n 2). y = e x (c1 ⋅ y1 + c 2 ⋅ y 2 + ... + c n ⋅ y n )

3). y = e c1x ⋅ y1 + ... + e c n x ⋅ y n

4). y = c1 ⋅ e y1 + c 2 ⋅ e y 2 + ... + c n ⋅ e y n

5). y = e c1y1 +...+ c n y n

62


Номер: 7.15.А Задача: Линейным однородным дифференциальным уравнением n-го порядка с постоянными коэффициентами называется уравнением вида Ответы: 1). F x , y (n ) , a 1 ⋅ y (n −1) ,..., a n −1 ⋅ y ′, a n y = 0

(

)

2). y (n ) + a 1 ⋅ y (n −1) + a n −1 ⋅ y ′ + a n y = 0 3). x + y (n ) + a 1 ⋅ y (n −1) + ... + a n y = 0

4). y (n ) + a 1 ⋅ y (n −1) + ... + a n y = f (x )

5). y (n ) + a 1 (x ) ⋅ y (n −1) + ... + a n (x )y = 0

Номер: 7.16.А Задача: Частные решения линейного однородного дифференциального уравнения n-го порядка ищем в виде Ответы: 1). e k1x + e k 2 x + ... + e k n x 2). k n ⋅ e kx 3). k ⋅ e kx 5). e kx 4). y = c1 ⋅ y1 + c 2 ⋅ y 2 + ... + c n ⋅ y n Номер: 7.17.А Задача: Решения y1 (x ) и y 2 (x ) дифференциального y ′′ + a 1 y ′ + a 2 y = 0 называются линейно независимыми, если Ответы: 1). 3).

y1 ≠ const y2

2).

уравнения

y1 = const , где const ≠ 0 y2

y1 ≠ const , где y 2 ≠ 0 y2

4). w (y1 , y 2 ) = 0

5).

x x

y1 ≠0 y2

Номер: 7.18.А Задача: Вронскиан функций y1 (x ) и y 2 (x ) это

y2 ⎞ ⎟ y ′2 ⎟⎠ y1 y 2 4). число (− 1) ⋅ 3). уравнение y1 ⋅ y ′2 − y 2 ⋅ y1′ = 0 y1′ y ′2 5). решение дифференциального уравнения y ′′ + a 1 ⋅ y ′ + a 2 ⋅ y = 0

Ответы: 1). определитель

y1 y1′

⎛ y1 ⎝ y1′

y2 y ′2

2). матрица ⎜⎜

Номер: 7.19.А Задача: Если y1 (x ) и y 2 (x ) линейно независимы, то вронскиан Ответы: 1). равен нулю 2). не равен нулю 3). равен нулевой матрице 4). равен единице 5). не существует

63


Номер: 7.20.А Задача: Если y1 (x ) и y 2 (x ) - линейно независимые частные решения дифференциального уравнения y ′′ + a 1 y ′ + a 2 y = 0 , то общее решение этого уравнения имеет вид 2). y = c1 y1 + c 2 y 2 3). y = e c1x ⋅ y1 + e c 2 x ⋅ y 2 Ответы: 1). y = y1 + y 2 4). y = e x ⋅ (c1 y1 + c 2 y 2 )

5). y = c1 ⋅ e y1 + c 2 ⋅ e y 2

Номер: 7.21.А Задача: Совокупность n решений линейного однородного дифференциального уравнения n-го порядка определенных и линейно независимых на промежутке (a , b ) называется Ответы: 1). частной системой решений 2). независимой системой решений 3). фундаментальной системой решений 4). линейной системой решений 5). общим решением

Задача:

Подставляя

Номер: 7.22.А y = e kx в дифференциальное

уравнение

y (n ) + a 1 y (n −1) + ... + a n −1 y ′ + a n y = 0 , получим

Ответы: 1). e kx (1 + a 1 + ... + a n −1 + a n ) = 0

(

)

2). e kx x n + a 1 x n −1 + ... + a n −1 x + a n = 0 4). k n + a 1 k n −1 + ... + a n = 0

3). k n + a 1 k n −1 + ... + a n k = 0

5). n ⋅ k n + (n − 1) ⋅ a 1 k n −1 + ... + a n = 0

Номер: 7.23.А Задача: Характеристическое уравнение, соответствующее дифференциальному уравнению y (n ) + a 1 y (n −1) + ... + a n −1 y ′ + a n y = 0 имеет вид Ответы: 1). e kx (1 + a 1 + ... + a n −1 + a n ) = 0

(

)

2). e kx x n + a 1 x n −1 + ... + a n −1 x + a n = 0 4). k n + a 1 k n −1 + ... + a n = 0

3). k n + a 1 k n −1 + ... + a n k = 0

5). n ⋅ k n + (n − 1) ⋅ a 1 k n −1 + ... + a n = 0

Номер: 7.24.А Задача: Характеристическое уравнение, соответствующее дифференциальному уравнению y ′′ + a 1 y ′ + a 2 y = 0 имеет вид

Ответы: 1). e kx (1 + a 1 + a 2 ) = 0 3). k 2 + a 1 k + a 2 k = 0

(

)

2). e kx x 2 + a 1 x + a 2 = 0 4). k ′′ + a 1 k ′ + a 2 k = 0

5). k 2 + a 1 k + a 2 = 0

64


Номер: 7.25.А Задача: Если характеристическое уравнение, соответствующее дифференциальному уравнению y ′′ + a 1 y ′ + a 2 y = 0 имеет 2 различных корня k 1 и k 2 , то фундаментальная система решений состоит из функций Ответы: 1). e k1x , e k 2 x

2). e k1x , e k 2 x , .k 1 ⋅ e k1x , k 2 ⋅ e k 2 x

4). e k1x ⋅ cos x , e k 2 x ⋅ sin x

3). c1e k1x , c 2 e k 2 x

5). c1e k1x + c 2 e k 2 x

Номер: 7.26.А Задача: Если характеристическое уравнение, соответствующее дифференциальному уравнению y ′′ + a 1 y ′ + a 2 y = 0 имеет действительные кратные корни, то фундаментальная система решений состоит из функций 2). e kx , .k ⋅ e kx 3). e kx , x ⋅ e kx Ответы: 1). e kx , e kx 4). e kx ⋅ cos x , e kx ⋅ sin x

5). e kx ⋅ cos kx , e kx ⋅ sin kx

Номер: 7.27.А Задача: Если характеристическое уравнение, соответствующее дифференциальному уравнению y ′′ + a 1 y ′ + a 2 y = 0 имеет комплексносопряженные корни k 1 = α + β i , k 2 = α − β i , то фундаментальная система решений состоит из функций 2). c1e αx ⋅ cos β x , c 2 e αx ⋅ sin β x Ответы: 1). e αx ⋅ cos β x , e αx ⋅ sin β x 3). e βx ⋅ cos αx , e βx ⋅ sin αx

4). e α +β , e α −β

5). c1 ⋅ e k1x , c 2 ⋅ e k 2 x

Номер: 7.28.А Задача: Пусть y1 и y 2 - линейно независимые частные решения y ′′ + a 1 y ′ + a 2 y = 0 . Подставим дифференциального уравнения y = c1 y 1 + c 2 y 2 в левую часть этого уравнения, получим c1 (y1′′ + a 1 y1′ + a 2 y1 ) + c 2 (y ′2′ + a 1 y ′2 + a 2 y 2 ) . Почему это выражение будет равно нулю? Ответы: 1). т.к. в самом дифференциальном уравнении правая часть равна нулю 2). т.к. y является общим решением уравнения 3). т.к. (y1′′ + a 1 y1′ + a 2 y1 ) = 0 и (y ′2′ + a 1 y ′2 + a 2 y 2 ) = 0 , ведь y1 и y 2 частные решения уравнения 4). т.к. ñ1 = 0 , ñ 2 = 0 5). т.к. y1 и y 2 - линейно независимые функции

65


Номер: 7.29.А Задача: Пусть характеристическое уравнение, соответствующее дифференциальному уравнению y ′′ + a 1 y ′ + a 2 y = 0 имеет действительные корни k 1 ≠ k 2 , тогда общее решение уравнения имеет вид Ответы: 1). y = e k1x + e k 2 x

2). y = c1e k1x + c 2 e k 2 x

3). y = k 1e c1x + k 2 e c 2 x

4). y = c1e k1x ⋅ cos x + c 2 e k 2 x ⋅ sin x

5). y = x ⋅ e k1x + x ⋅ e k 2 x Номер: 7.30.А Задача: Пусть характеристическое уравнение, соответствующее дифференциальному уравнению y ′′ + a 1 y ′ + a 2 y = 0 имеет действительные кратные корни, тогда общее решение уравнения имеет вид 2). y = c1e kx + c 2 ⋅ k ⋅ e kx Ответы: 1). y = ñ1e kx + ñ 2 e k 2 x 3). y = c1e kx + c 2 ⋅ x ⋅ e kx 5). y = e kx (c1 ± kc 2 )

4). y = e kx (c1 ⋅ cos x + c 2 ⋅ sin x )

Номер: 7.31.А Задача: Пусть характеристическое уравнение, соответствующее дифференциальному уравнению y ′′ + a 1 y ′ + a 2 y = 0 имеет комплексносопряженные корни k 1 = α + β i , k 2 = α − β i , тогда общее решение уравнения имеет вид Ответы: 1). y = e αx (ñ1 cos β x + ñ 2 sin β x ) 2). y = ñ1e αx ⋅ cos x + ñ 2 e αx ⋅ sin x 4). y = c1e βx cos αx + c 2 e βx sin αx (n )

3). y = c1e (α +β )x + c 2 e (α −β ) x

5). y = e βx (c1 cos x + c 2 sin x )

Номер: 7.32.А

Задача: Уравнение вида y + a 1 ⋅ y (n −1) + ... + a n −1 ⋅ y ′ + a n y = f (x ) есть Ответы: 1). неоднородное уравнение теплопроводности 2) характеристическое уравнение 3). дифференциальное уравнение в частных производных 4). линейное неоднородное дифференциальное уравнение n порядка с постоянными коэффициентами 5). линейное однородное дифференциальное уравнение n порядка с постоянными коэффициентами Номер: 7.33.А Задача: Пусть y - частное решение линейного неоднородного дифференциального уравнения II порядка (ЛНДУ), y - общее решение ∗

66


соответствующего однородного уравнения (ЛОДУ), тогда общее решение ЛНДУ имеет вид 2). y = c1 ⋅ y + + c 2 ⋅ y ∗ 3). y = y + c ⋅ y ∗ Ответы: 1). y = y + y ∗ ∗

4). y = y ⋅ cos x + y ∗ ⋅ sin x

5). y = e y ⋅ cos x + e y ⋅ sin x

Номер: 7.34.А Задача: Пусть y - общее решение y ′′ + a 1 y ′ + a 2 y = 0 (1), y ∗ - частное 2). Подставим вместо y в левую часть решение y ′′ + a 1 y ′ + a 2 y = f (x ) ( уравнения (2) y + y ∗ , получим

(y ′′ + a 1 y ′ + a 2 y ) + ⎜⎛ y ∗

3). Почему выражение (3) равно f (x ) Ответы: 1). т.к. в правой части уравнения (2) f (x )

2). т.к. y = y + y ∗ общее решение уравнения (2) (y ′′ + a 1 y ′ + a 2 y ) = 0 , т.к. y - решение 3).

′ ⎞ + a1y∗ + a 2 y∗ ⎟ ( ⎠

уравнения

(1),

⎛ ∗″ ⎞ ∗′ + + a 2 y ∗ ⎟ = f (x ) , т.к. y ∗ - решение уравнения (2) y a y ⎜ 1 ⎝ ⎠ 4). выражение (3) не будет равно f (x )

5). выражение (3) будет равно нулю Номер: 7.35.А - частное решение линейного неоднородного Задача: Пусть y дифференциального уравнения II порядка y = c1 y1 + c 2 y 2 (c1 , c 2 − const ) ∗

общее решение соответствующего однородного уравнения. Тогда y ∗ будем искать в виде 2). 3). y ∗ = c(x ) ⋅ y Ответы: 1). y ∗ = y1 + y 2 4). y ∗ = c1 y1 + c 2 y 2

5). y ∗ = c1 (x )y1 + c 2 (x )y 2

Номер: 7.36.А

Задача: Пусть y = ñ1 ⋅ y1 (x ) + c 2 ⋅ y 2 (x )(c1c 2 − const ) - общее решение линейного однородного дифференциального уравнения II порядка. Будет ли

⎧ñ1′ (x ) ⋅ y1 (x ) + ñ′2 (x ) ⋅ y 2 (x ) = 0 иметь единственное решение? ′ ′ ′ ′ ( ) ( ) ( ) ( ) ( ) ⋅ + ⋅ = ñ x y x ñ x y x f x ⎩ 1 1 2 2

система ⎨

Если да, то почему? Ответы: 1). да, т.к. y1 (x ) и y 2 (x ) линейно зависимы 2). да, т.к. c1′ (x ) ≠ 0 , c ′2 (x ) ≠ 0 3). да, т.к. y1 (x ) и y 2 (x ) образуют фундаментальную систему решений, поэтому главный определитель этой системы – вронскиан не равен нулю 67


4). система неверно составлена 5). нет, системы имеет множество решений, т.к. c1 (x ) , c 2 (x ) - произвольные функции Номер: 7.37.А Задача: Пусть y = ñ1 (x ) ⋅ y1 + c 2 (x ) ⋅ y 2 - частное решение линейного неоднородного дифференциального уравнения II порядка, (ЛНДУ) y = ñ1 ⋅ y1 + c 2 ⋅ y 2 (c1c 2 − const ) - общее решение соответствующего однородного уравнения. Для нахождения c1 (x ) и c 2 (x ) надо ∗

Ответы: 1). y ∗ подставить в ЛНДУ

2). y = y + y ∗ подставить в ЛНДУ

3). y = c1 y + c 2 y ∗ подставить в ЛНДУ 4). решить систему уравнений:

ñ1 (x ) ⋅ y1′ + c 2 (x ) ⋅ y ′2 = 0 ,

5).

ñ1′ (x ) ⋅ y1 + c ′2 (x ) ⋅ y 2 = 0 ,

ñ1′ (x ) ⋅ y1 + c ′2 (x ) ⋅ y 2 = f (x ) решить

систему

ñ1′ (x ) ⋅ y1′ + c ′2 (x ) ⋅ y ′2 = f (x )

Задача:

Номер: 7.38.А решение дифференциального

Частное

y ′′ + a 1 y ′ + a 2 y = Pn (x )

ищем в виде Ответы: 1). x r ⋅ M n (x )

4). x r ⋅ Pn (x ) ⋅ e αx

уравнений:

(a 1 , a 2 − const, Pn (x ) − ìíîãî÷ëåí

2). Pn (x )

3). x r ⋅ e αx ⋅ M n (x ) ⋅ cos β x

уравнения

4). x r ⋅ e βx ⋅ A

Номер: 7.40.А решение дифференциального

y ′′ + a 1 y ′ + a 2 y = e αx ⋅ Pn (x ) n − é ñòåïåíè ) ищем в виде

Ответы: 1). x r ⋅ e αx ⋅ Pn (x )

)

5). α1 y1 + a 2 y 2

Ответы: 1). x r ⋅ e αx 2). x r ⋅ e βx 3). x r ⋅ e αx ⋅ A 5). x r e βx (A ⋅ cos β x + B ⋅ sin β x ) Частное

n − é ñòåïåíè

3). x r ⋅ Pn (x )

Номер: 7.39.А Задача: Частное решение дифференциального αx y ′′ + a 1 y ′ + a 2 y = e (a 1 , a 2 − const ) ищем в виде

Задача:

уравнения

уравнения

(a 1 , a 2 − const, Pn (x ) − ìíîãî÷ëåí

2). x r ⋅ e αx ⋅ M n (x ) 4). a 1 y1 + a 2 y 2

68

5). a 1 (x )y1 + a 2 (x )y 2


Задача:

Частное

Номер: 7.41.А решение дифференциального

y ′′ + a 1 y ′ + a 2 y = e ⋅ Pn (x ) ⋅ ños βx n − é ñòåïåíè ) ищем в виде

Ответы: 1). e αx ⋅ M n (x ) ⋅ cos β x

уравнения

(a 1 , a 2 − const, Pn (x ) − ìíîãî÷ëåí

αx

2). x r ⋅ e αx ⋅ M n (x ) ⋅ cos β x

3). e αx ⋅ (M n (x ) ⋅ cos β x + N n (x ) ⋅ sin β x )

4). x r ⋅ e αx ⋅ (M n (x ) ⋅ cos β x + N n (x ) ⋅ sin β x )

5). x r ⋅ e αx ⋅ (Pn (x ) ⋅ cos β x + N n (x ) ⋅ sin β x ) Задача:

Частное

Номер: 7.42.А решение дифференциального

y ′′ + a 1 y ′ + a 2 y = e ⋅ Q m (x ) ⋅ sin βx m − é ñòåïåíè ) ищем в виде

Ответы: 1). e αx ⋅ N m (x ) ⋅ sin β x

уравнения

(a 1 , a 2 − const, Q m (x ) − ìíîãî÷ëåí

αx

2). x r ⋅ e αx ⋅ N m (x ) ⋅ sin β x

3). e αx ⋅ (M m (x ) ⋅ cos β x + N m (x ) ⋅ sin β x )

4). x r ⋅ e αx ⋅ (M m (x ) ⋅ cos β x + N m (x ) ⋅ sin β x ) 5). x r ⋅ e αx ⋅ (M m (x ) ⋅ cos β x + Q m (x ) ⋅ sin β x )

Номер: 7.43.А Задача: Частное решение дифференциального y ′′ + a 1 y ′ + a 2 y = cos βx (a 1 , a 2 − const ) ищем в виде Ответы: 1). a 1 y1 + a 2 y 2

2). a 1 (x )y1 + a 2 (x )y 2

4). x r ⋅ A ⋅ (cos β x + sin β x )

3). x r ⋅ A ⋅ cos β x

5). x r ⋅ (A ⋅ cos β x + B ⋅ sin β x )

Номер: 7.44.А Задача: Частное решение дифференциального y ′′ + a 1 y ′ + a 2 y = sin βx (a 1 , a 2 − const ) ищем в виде Ответы: 1). a 1 y1 + a 2 y 2

2). a 1 (x )y1 + a 2 (x )y 2

Задача:

Номер: 7.45.А решение дифференциального

4). x r ⋅ A ⋅ (cos β x + sin β x ) Частное

3). x r ⋅ Pn (x ) ⋅ cos β x

уравнения

3). x r ⋅ A ⋅ sin β x

5). x r ⋅ (A ⋅ cos β x + B ⋅ sin β x )

y ′′ + a 1 y ′ + a 2 y = Pn (x ) ⋅ cos β x

степени) ищем в виде Ответы: 1). M n (x ) ⋅ cos β x

уравнения

уравнения

(a 1 , a 2 − const, Pn (x ) − ìíîãî÷ëåí

2). x

r

⋅M

n

(x ) ⋅ cos

βx

4). x r ⋅ (Pn (x ) ⋅ cos β x + N n (x ) ⋅ sin β x ) 69

n−é


5). x r ⋅ (M n (x ) ⋅ cos β x + N n (x ) ⋅ sin β x ) Задача:

Номер: 7.46.А решение дифференциального

Частное

y ′′ + a 1 y ′ + a 2 y = Q m (x ) ⋅ sin β x

степени) ищем в виде Ответы: 1). N m (x ) ⋅ sin β x

уравнения

(a 1 , a 2 − const, Q m (x ) − ìíîãî÷ëåí

2). x r ⋅ N m (x ) ⋅ sin β x

4). x r ⋅ (M m (x ) ⋅ cos β x + N m (x ) ⋅ sin β x )

m−é

3). x r ⋅ Q m (x ) ⋅ sin β x

5). x r ⋅ (M m (x ) ⋅ cos β x + Q m (x ) ⋅ sin β x )

Номер: 7.47.А Задача: Частное решение дифференциального αx y ′′ + a 1 y ′ + a 2 y = e ⋅ cos βx (a 1 , a 2 − const ) ищем в виде Ответы: 1). x r ⋅ e αx (A ⋅ cos β x + B ⋅ sin β x ) 3). e αx ⋅ M(x ) ⋅ cos β x

2). x r ⋅ e αx ⋅ A ⋅ cos β x

4). x r ⋅ e αx ⋅ M n (x ) ⋅ cos β x

Номер: 7.48.А Задача: Частное решение дифференциального y ′′ + a 1 y ′ + a 2 y = e αx ⋅ sin βx (a 1 , a 2 − const ) ищем в виде

Ответы: 1). e αx ⋅ N m (x ) ⋅ sin β x

3). a 1 (x ) ⋅ y1 + a 2 (x ) ⋅ y 2

уравнения

5). a 1 y1 + a 2 y 2 уравнения

2). x r ⋅ e αx ⋅ N m (x ) ⋅ sin β x

4). x r ⋅ e αx ⋅ (A ⋅ cos β x + B ⋅ sin β x )

5). x r ⋅ e αx ⋅ B ⋅ sin β x Номер: 7.49.А Задача: Пусть характеристическое уравнение k 2 + a 1 k + a 2 = 0 имеет комплексные корни α ± βi ( D < 0 ), тогда частное решение дифференциального уравнения y ′′ + a 1 y ′ + a 2 y = Pn (x ) (a 1 , a 2 − const , Pn (x ) - многочлен n-й степени) следует искать в виде Ответы: 1). M n (x )

2). x ⋅ M n (x )

4). M n (x ) ⋅ cos β x + N n (x ) ⋅ sin β x

3). x 2 ⋅ M n (x )

5). e αx (M n (x ) ⋅ cos β x + N n (x ) ⋅ sin β x )

Номер: 7.50.А Задача: Пусть характеристическое уравнение k 2 + a 1 k + a 2 = 0 имеет комплексные корни α ± βi ( D < 0 ), тогда частное решение дифференциального уравнения y ′′ + a 1 y ′ + a 2 y = e αx ⋅ Pn (x ) (a 1 , a 2 − const , Pn (x ) - многочлен n-й степени) следует искать в виде 70


Ответы: 1). e αx ⋅ M n (x )

2). x ⋅ e αx ⋅ M n (x )

3). e αx (M n (x ) ⋅ cos β x + N n (x ) ⋅ sin β x )

4). x ⋅ e αx (M n (x ) ⋅ cos β x + N n (x ) ⋅ sin β x )

5). x 2 ⋅ e αx (M n (x ) ⋅ cos β x + N n (x ) ⋅ sin β x ) Номер: 7.51.А Задача: Пусть характеристическое уравнение k 2 + a 1 k + a 2 = 0 имеет комплексные корни α ± βi ( D < 0 ), тогда частное решение

y ′′ + a 1 y ′ + a 2 y = e αx ⋅ Pn (x ) ⋅ cos βx дифференциального уравнения (a 1 , a 2 − const , Pn (x ) - многочлен n-й степени) следует искать в виде

Ответы: 1). e αx ⋅ M n (x ) ⋅ cos β x

2). x ⋅ e αx ⋅ M n (x ) ⋅ cos β x

3). e αx (M n (x ) ⋅ cos β x + N n (x ) ⋅ sin β x )

4). x ⋅ e αx (M n (x ) ⋅ cos β x + N n (x ) ⋅ sin β x )

5). x 2 ⋅ e αx (M n (x ) ⋅ cos β x + N n (x ) ⋅ sin β x ) Номер: 7.52.А Задача: Пусть характеристическое уравнение k 2 + a 1 k + a 2 = 0 имеет комплексные корни ( D < 0 ), тогда частное решение α ± βi

y ′′ + a 1 y ′ + a 2 y = e αx ⋅ Q m (x ) ⋅ sin β x дифференциального уравнения (a 1 , a 2 − const , Q m (x ) - многочлен m-й степени) следует искать в виде

Ответы: 1). e αx ⋅ N m (x ) ⋅ sin β x

2). x ⋅ e αx ⋅ N m (x ) ⋅ sin β x

3). e αx (M m (x ) ⋅ cos β x + N m (x ) ⋅ sin β x )

4). x ⋅ e αx (M m (x ) ⋅ cos β x + N m (x ) ⋅ sin β x )

5). x 2 ⋅ e αx (M m (x ) ⋅ cos β x + N m (x ) ⋅ sin β x ) Номер: 7.53.А Задача: Пусть характеристическое уравнение k 2 + a 1 k + a 2 = 0 имеет двукратный корень k=α ( D = 0 ), тогда частное решение дифференциального уравнения y ′′ + a 1 y ′ + a 2 y = e αx ⋅ Pn (x ) (a 1 , a 2 − const , Pn (x ) - многочлен n-й степени) следует искать в виде Ответы: 1). e αx ⋅ M n (x ) 4). a 1 y1 + a 2 y 2

2). x ⋅ e αx ⋅ M n (x ) 3). x 2 ⋅ e αx ⋅ M n (x ) 5). a 1 (x ) ⋅ y1 + a 2 (x ) ⋅ y 2

71


Номер: 7.54.А Задача: Пусть характеристическое уравнение k 2 + a 1 k + a 2 = 0 имеет двукратный корень k≠α ( D = 0 ), тогда частное решение αx дифференциального уравнения y ′′ + a 1 y ′ + a 2 y = e ⋅ Pn (x ) (a 1 , a 2 − const , Pn (x ) - многочлен n-й степени) следует искать в виде

Ответы: 1). e αx ⋅ M n (x ) 4). a 1 y1 + a 2 y 2

2). x ⋅ e αx ⋅ M n (x ) 3). x 2 ⋅ e αx ⋅ M n (x ) 5). a 1 (x ) ⋅ y1 + a 2 (x ) ⋅ y 2

Номер: 7.55.А Задача: Пусть характеристическое уравнение k 2 + a 1 k + a 2 = 0 имеет двукратный корень ( D = 0 ), тогда частное решение k=α αx y ′′ + a 1 y ′ + a 2 y = e ⋅ Pn (x ) ⋅ cos βx дифференциального уравнения (a 1 , a 2 − const , Pn (x ) - многочлен n-й степени) следует искать в виде

Ответы: 1). e αx ⋅ M n (x ) ⋅ cos β x

2). x ⋅ e αx ⋅ M n (x ) ⋅ cos β x

3). x 2 ⋅ e αx ⋅ (M n (x ) ⋅ cos β x + N n (x ) ⋅ sin β x ) 4). x ⋅ e αx ⋅ (M n (x ) ⋅ cos β x + N n (x ) ⋅ sin β x )

5). e αx ⋅ (M n (x ) ⋅ cos β x + N n (x ) ⋅ sin β x )

Номер: 7.56.А Задача: Пусть характеристическое уравнение k 2 + a 1 k + a 2 = 0 имеет двукратный корень k≠α ( D = 0 ), тогда частное решение αx y ′′ + a 1 y ′ + a 2 y = e ⋅ Pn (x ) ⋅ cos βx дифференциального уравнения (a 1 , a 2 − const , Pn (x ) - многочлен n-й степени) следует искать в виде

Ответы: 1). e αx ⋅ M n (x ) ⋅ cos β x

2). x ⋅ e αx ⋅ M n (x ) ⋅ cos β x

3). x 2 ⋅ e αx ⋅ (M n (x ) ⋅ cos β x + N n (x ) ⋅ sin β x ) 4). x ⋅ e αx ⋅ (M n (x ) ⋅ cos β x + N n (x ) ⋅ sin β x )

5). e αx ⋅ (M n (x ) ⋅ cos β x + N n (x ) ⋅ sin β x )

Номер: 7.57.А Задача: Пусть характеристическое уравнение k 2 + a 1 k + a 2 = 0 имеет двукратный корень k=α ( D = 0 ), тогда частное решение y ′′ + a 1 y ′ + a 2 y = e αx ⋅ Q m (x ) ⋅ sin β x дифференциального уравнения (a 1 , a 2 − const , Q m (x ) - многочлен m-й степени) следует искать в виде

Ответы: 1). e αx ⋅ (M m (x ) ⋅ cos β x + N m (x ) ⋅ sin β x ) 2). x ⋅ e αx ⋅ (M m (x ) ⋅ cos β x + N m (x ) ⋅ sin β x ) 72


3). x 2 ⋅ e αx ⋅ (M m (x ) ⋅ cos β x + N m (x ) ⋅ sin β x )

4). e αx ⋅ N m (x ) ⋅ sin β x

5). x ⋅ e αx ⋅ N m (x ) ⋅ sin β x

Номер: 7.58.А Задача: Пусть характеристическое уравнение k 2 + a 1 k + a 2 = 0 имеет двукратный корень k≠α ( D = 0 ), тогда частное решение y ′′ + a 1 y ′ + a 2 y = e αx ⋅ Q m (x ) ⋅ sin β x дифференциального уравнения (a 1 , a 2 − const , Q m (x ) - многочлен m-й степени) следует искать в виде

Ответы: 1). e αx ⋅ (M m (x ) ⋅ cos β x + N m (x ) ⋅ sin β x ) 2). x ⋅ e αx ⋅ (M m (x ) ⋅ cos β x + N m (x ) ⋅ sin β x )

3). x 2 ⋅ e αx ⋅ (M m (x ) ⋅ cos β x + N m (x ) ⋅ sin β x ) 4). e αx ⋅ N m (x ) ⋅ sin β x

5). x ⋅ e αx ⋅ N m (x ) ⋅ sin β x

Номер: 7.59.А Задача: Пусть характеристическое уравнение k 2 + a 1 k + a 2 = 0 имеет двукратный корень k≠α ( D = 0 ), тогда частное решение дифференциального уравнения y ′′ + a 1 y ′ + a 2 y = Pn (x ) (a 1 , a 2 − const , Pn (x ) - многочлен n-й степени) следует искать в виде 2). x ⋅ M n (x ) Ответы: 1). x 2 ⋅ M n (x ) 5). a 1 (x )y1 + a 2 (x )y 2

3). M n (x )

4). a 1 y1 + a 2 y 2

Номер: 7.60.А Задача: Пусть характеристическое уравнение k 2 + a 1 k + a 2 = 0 имеет действительные корни k 1 = 0 , k 2 ≠ 0 ( D > 0 ), тогда частное решение дифференциального уравнения y ′′ + a 1 y ′ + a 2 y = Pn (x ) (a 1 , a 2 − const , Pn (x ) - многочлен n-й степени) следует искать в виде Ответы: 1). M n (x ) 4). a 1 y1 + a 2 y 2

2). x ⋅ M n (x ) 3). x 2 ⋅ M n (x ) 5). a 1 (x )y1 + a 2 (x )y 2

Номер: 7.61.А Задача: Пусть характеристическое уравнение k 2 + a 1 k + a 2 = 0 имеет действительные корни k 1 ≠ 0 , k 2 ≠ 0 ( D > 0 ), тогда частное решение дифференциального уравнения y ′′ + a 1 y ′ + a 2 y = Pn (x ) (a 1 , a 2 − const , Pn (x ) - многочлен n-й степени) следует искать в виде Ответы: 1). M n (x ) 4). a 1 y1 + a 2 y 2

2). x ⋅ M n (x ) 3). x 2 ⋅ M n (x ) 5). a 1 (x )y1 + a 2 (x )y 2 73


Номер: 7.62.А Задача: Пусть характеристическое уравнение k 2 + a 1 k + a 2 = 0 имеет действительные различные корни k 1 ≠ α , k 2 ≠ α ( D > 0 ), тогда частное

y ′′ + a 1 y ′ + a 2 y = e αx ⋅ Pn (x ) дифференциального уравнения (a 1 , a 2 − const , Pn (x ) - многочлен n-й степени) следует искать в виде

решение

Ответы: 1). e αx ⋅ M n (x ) 4). a 1 y1 + a 2 y 2

2). x ⋅ e αx ⋅ M n (x ) 3). x 2 ⋅ e αx ⋅ M n (x ) 5). a 1 (x )y1 + a 2 (x )y 2

Номер: 7.63.А Задача: Пусть характеристическое уравнение k 2 + a 1 k + a 2 = 0 имеет действительные различные корни k 1 = α , k 2 ≠ α ( D > 0 ), тогда частное

y ′′ + a 1 y ′ + a 2 y = e αx ⋅ Pn (x ) дифференциального уравнения (a 1 , a 2 − const , Pn (x ) - многочлен n-й степени) следует искать в виде

решение

Ответы: 1). e αx ⋅ M n (x ) 4). a 1 y1 + a 2 y 2

2). x ⋅ e αx ⋅ M n (x ) 3). x 2 ⋅ e αx ⋅ M n (x ) 5). a 1 (x )y1 + a 2 (x )y 2

Номер: 7.64.А Задача: Пусть характеристическое уравнение k 2 + a 1 k + a 2 = 0 имеет действительные различные корни k 1 ≠ α , k 2 ≠ α ( D > 0 ), тогда частное решение дифференциального уравнения y ′′ + a 1 y ′ + a 2 y = e αx ⋅ Pn (x ) ⋅ cos β x (a 1 , a 2 − const , Pn (x ) - многочлен n-й степени) следует искать в виде

Ответы: 1). e αx ⋅ M n (x ) ⋅ cos β x

2). x ⋅ e αx ⋅ M n (x ) ⋅ cos β x

3). e αx ⋅ (M n (x ) ⋅ cos β x + N n (x ) ⋅ sin β x )

4). x ⋅ e αx ⋅ (M n (x ) ⋅ cos β x + N n (x ) ⋅ sin β x )

5). x 2 ⋅ e αx ⋅ (M n (x ) ⋅ cos β x + N n (x ) ⋅ sin β x ) Номер: 7.65.А Задача: Пусть характеристическое уравнение k 2 + a 1 k + a 2 = 0 имеет действительные различные корни k 1 = α , k 2 ≠ α ( D > 0 ), тогда частное

решение дифференциального уравнения y ′′ + a 1 y ′ + a 2 y = e αx ⋅ Pn (x ) ⋅ cos β x (a 1 , a 2 − const , Pn (x ) - многочлен n-й степени) следует искать в виде Ответы: 1). e αx ⋅ M n (x ) ⋅ cos β x

2). x ⋅ e αx ⋅ M n (x ) ⋅ cos β x

3). e αx ⋅ (M n (x ) ⋅ cos β x + N n (x ) ⋅ sin β x )

4). x ⋅ e αx ⋅ (M n (x ) ⋅ cos β x + N n (x ) ⋅ sin β x )

5). x 2 ⋅ e αx ⋅ (M n (x ) ⋅ cos β x + N n (x ) ⋅ sin β x ) 74


Номер: 7.66.А Задача: Пусть характеристическое уравнение k 2 + a 1 k + a 2 = 0 имеет действительные различные корни k 1 ≠ α , k 2 ≠ α ( D > 0 ), тогда частное решение дифференциального уравнения αx y ′′ + a 1 y ′ + a 2 y = e ⋅ Q m (x ) ⋅ sin βx (a 1 , a 2 − const , Q m (x ) - многочлен mй степени) следует искать в виде 2). x ⋅ e αx ⋅ N m (x ) ⋅ sin β x Ответы: 1). e αx ⋅ N m (x ) ⋅ sin β x 3). x 2 ⋅ e αx ⋅ (M m (x ) ⋅ cos β x + N m (x ) ⋅ sin β x ) 4). x ⋅ e αx ⋅ (M m (x ) ⋅ cos β x + N m (x ) ⋅ sin β x )

5). e αx ⋅ (M m (x ) ⋅ cos β x + N m (x ) ⋅ sin β x )

Номер: 7.67.А Задача: Пусть характеристическое уравнение k 2 + a 1 k + a 2 = 0 имеет действительные различные корни k 1 = α , k 2 ≠ α ( D > 0 ), тогда частное решение дифференциального уравнения y ′′ + a 1 y ′ + a 2 y = e αx ⋅ Q m (x ) ⋅ sin βx (a 1 , a 2 − const , Q m (x ) - многочлен mй степени) следует искать в виде 2). x ⋅ e αx ⋅ N m (x ) ⋅ sin β x Ответы: 1). e αx ⋅ N m (x ) ⋅ sin β x 3). x 2 ⋅ e αx ⋅ (M m (x ) ⋅ cos β x + N m (x ) ⋅ sin β x ) 4). x ⋅ e αx ⋅ (M m (x ) ⋅ cos β x + N m (x ) ⋅ sin β x )

5). e αx ⋅ (M m (x ) ⋅ cos β x + N m (x ) ⋅ sin β x )

75


8. Обыкновенные дифференциальные уравнения n-ого порядка. Типы уравнений. Фундаментальная система решений. Задачи. Номер: 8.1.А Задача: Составьте линейное однородное дифференциальное уравнение, зная ее фундаментальную систему решений: e 2 x , xe 2 x Ответы: 1). y′′ − 4 y′ + 4 = 0 4). y′′ − 16 y′ + 4 = 0

2). y′′ + 4 y′ + 4 = 0

3). y′′ − 16 y = 0

5). y′′ − 16 y′ + 16 = 0

Номер: 8.2.А Задача: Составьте линейное однородное дифференциальное уравнение, зная ее фундаментальную систему решений: e 2 x , e −2 x Ответы: 1). y′′ − 4 y = 0

2). y′′ + 4 y = 0

4). y′′ + 8 y = 0

5). y′′ − 8 y = 0

3). y′′ − 4 y′ = 0

Номер: 8.3.А Задача: Составьте линейное однородное дифференциальное уравнение, зная ее фундаментальную систему решений: e3 x , e 2 x Ответы: 1). y′′ − 5 y′ + 6 y = 0 4). y′′ + 5 y′ = 0

2). y′′ − 5 y′ = 0

3). y′′ + 5 y = 0

5). y′′ − 6 y′ = 0

Номер: 8.4.А Задача: Составьте линейное однородное дифференциальное уравнение, зная ее фундаментальную систему решений: sin x , cos x Ответы: 1). y′′ + y = 0

2). y′′ − y = 0

3). y′′ − y′ = 0

4). y′′ + 3y = 0

5). y′′ − 3y = 0 Номер: 8.5.А Задача: Составьте линейное однородное дифференциальное уравнение, зная ее фундаментальную систему решений: e −3 x , e 2 x Ответы: 1). y′′ + y′ − 6 y = 0 4). y′′ + y′ − 4 y = 0

2). y′′ − 6 y = 0

3). y′′ + 6 y′ = 0

5). y′′ + y′ + 6 y = 0

76


Номер: 8.6.А Задача: Составьте линейное однородное дифференциальное уравнение, зная ее фундаментальную систему решений: e −3 x , e −2 x Ответы: 1). y′′ + 5 y′ + 6 = 0

2). y′′ − 6 y = 0

3). y′′ − 6 y = 0

5). y′′ − 5 y = 0

4). y′′ + 6 y = 0

Номер: 8.7.А Задача: Составьте линейное однородное дифференциальное уравнение, зная ее фундаментальную систему решений: e x , e3 x 2). y′′ − 4 y = 0

Ответы: 1). y′′ − 4 y′ + 3 = 0

3). y′′ + 3y = 0

5). y′′ − 14 y = 0

4). y′′ + 4 y = 0

Номер: 8.8.А Задача: Составьте линейное однородное дифференциальное уравнение, зная ее фундаментальную систему решений: 1, x , e x Ответы: 1). y′′′ − y′′ = 0

2). y′′′ + y′ = 0

4). y′′′ + 2 y = 0

5). y′′′ + y = 0

3). y′′′ + y′ = 0

Номер: 8.9.А Задача: Составьте линейное однородное дифференциальное уравнение, зная ее фундаментальную систему решений: e 2 x , xe 2 x Ответы: 1). y′′ − 4 y′ + 4 y = 0 4). y′′ + 4 y′ = 0

2). y′′ − 4 y′ = 0

3). y′′ + 4 y = 0

5). y′′ − 2 y′ = 0

Номер: 8.10.А Задача: Составьте линейное однородное дифференциальное уравнение, зная ее фундаментальную систему решений: e x , xe x Ответы: 1). y′′ − 2 y′ + y = 0 4). y′′ + 2 y′ = 0

2). y′′ − 2 y′ = 0 5). y′′ − 4 y′ = 0

77

3). y′′ + y = 0


Номер: 8.11.А Задача: Составьте линейное однородное дифференциальное уравнение, зная ее фундаментальную систему решений: e − x , xe − x Ответы: 1). y′′ + 2 y′ + y = 0 4). y′′ − 2 y′ = 0

2). y′′ + 2 y′ = 0

3). y′′ + y = 0

5). y′′ − 4 y′ = 0

Номер: 8.12.А Задача: Составьте линейное однородное дифференциальное уравнение, зная ее фундаментальную систему решений: e 2 x , e − x Ответы: 1). y′′ − y′ − 2 y = 0

2). y′′ + y′ − 2 y = 0

4). y′′ − y′ + 4 y = 0

5). y′′ + y′ + 4 y = 0

3). y′′ − 2 y′ = 0

Номер: 8.13.А Задача: Составьте линейное однородное дифференциальное уравнение, зная ее фундаментальную систему решений: e −2 x , e x Ответы: 1). y′′ + y′ − 2 y = 0

2). y′′ − y′ − 2 y = 0

4). y′′ + y′ + 2 y = 0

5). y′′ + y′ + 4 y = 0

Задача:

Укажите

3). y′′ + 2 y′ = 0

Номер: 8.14.А метод решения дифференциального

уравнения

y′′ = 3 x + sin x Ответы: 1). непосредственное интегрирование

2). понижение порядка с помощью подстановки y′x = p(x )

3). понижение порядка с помощью подстановки y′x = z(y )

4). поиск частного решения методом неопределенных коэффициентов 5). поиск частного решения только методом вариации постоянных Номер: 8.15.А Задача: Укажите метод решения дифференциального уравнения y′′ = x ⋅ y′ + x Ответы: 1). непосредственное интегрирование

2). понижение порядка с помощью подстановки y′x = p(x )

3). понижение порядка с помощью подстановки y′x = z(y ) 78


4). поиск частного решения методом неопределенных коэффициентов 5). поиск частного решения только методом вариации постоянных Номер: 8.16.А Задача: Укажите метод решения дифференциального уравнения y′′ = ln x Ответы: 1). непосредственное интегрирование

2). понижение порядка с помощью подстановки y′x = p(x )

3). понижение порядка с помощью подстановки y′x = z(y )

4). поиск частного решения методом неопределенных коэффициентов 5). поиск частного решения только методом вариации постоянных Номер: 8.17.А Задача: Укажите метод решения дифференциального уравнения xy′′ = y′ Ответы: 1). непосредственное интегрирование

2). понижение порядка с помощью подстановки y′x = p(x )

3). понижение порядка с помощью подстановки y′x = z(y )

4). поиск частного решения методом неопределенных коэффициентов 5). поиск частного решения только методом вариации постоянных Номер: 8.18.А Задача: Укажите метод решения дифференциального уравнения (y′′) = y′ 2

Ответы: 1). непосредственное интегрирование

2). понижение порядка с помощью подстановки y′x = p(x )

3). понижение порядка с помощью подстановки y′x = z(y )

4). поиск частного решения методом неопределенных коэффициентов 5). поиск частного решения только методом вариации постоянных

Задача:

Укажите

Номер: 8.19.А метод решения дифференциального

(y′)2 + 2 yy′′ = 0

79

уравнения


Ответы: 1). непосредственное интегрирование

2). понижение порядка с помощью подстановки y′x = p(x )

3). понижение порядка с помощью подстановки y′x = z(y ) 4). поиск частного решения методом неопределенных коэффициентов 5). поиск частного решения только методом вариации постоянных Номер: 8.20.А

1

Задача: Укажите метод решения дифференциального уравнения y′′ =

4 y

Ответы: 1). непосредственное интегрирование

2). понижение порядка с помощью подстановки y′x = p(x )

3). понижение порядка с помощью подстановки y′x = z(y )

4). поиск частного решения методом неопределенных коэффициентов 5). поиск частного решения только методом вариации постоянных

Задача:

Укажите

Номер: 8.21.А метод решения дифференциального

уравнения

2 xy′y′′ = (y′)2 + 1 Ответы: 1). непосредственное интегрирование

2). понижение порядка с помощью подстановки y′x = p(x )

3). понижение порядка с помощью подстановки y′x = z(y ) 4). поиск частного решения методом неопределенных коэффициентов 5). поиск частного решения только методом вариации постоянных Номер: 8.22.А Задача: Укажите метод решения дифференциального уравнения y′′ = arctg x Ответы: 1). непосредственное интегрирование

2). понижение порядка с помощью подстановки y′x = p(x )

3). понижение порядка с помощью подстановки y′x = z(y ) 4). поиск частного решения методом неопределенных коэффициентов 5). поиск частного решения только методом вариации постоянных

80


Задача:

Укажите

Номер: 8.23.А метод решения дифференциального

уравнения

xy′′ + x (y′)2 − y′ = 0 Ответы: 1). непосредственное интегрирование

2). понижение порядка с помощью подстановки y′x = p(x )

3). понижение порядка с помощью подстановки y′x = z(y )

4). поиск частного решения методом неопределенных коэффициентов 5). поиск частного решения только методом вариации постоянных Номер: 8.24.А Задача: Укажите метод решения дифференциального уравнения y′′ = e 2 y Ответы: 1). непосредственное интегрирование

2). понижение порядка с помощью подстановки y′x = p(x )

3). понижение порядка с помощью подстановки y′x = z(y )

4). поиск частного решения методом неопределенных коэффициентов 5). поиск частного решения только методом вариации постоянных Номер: 8.25.А Задача: Укажите метод решения дифференциального уравнения y′′′ = x 3 − 1 Ответы: 1). непосредственное интегрирование

2). понижение порядка с помощью подстановки y′x = p(x )

3). понижение порядка с помощью подстановки y′x = z(y )

4). поиск частного решения методом неопределенных коэффициентов 5). поиск частного решения только методом вариации постоянных Номер: 8.26.А Задача: Укажите метод решения дифференциального уравнения

y′′ − (2 y + 3) − 2 y′ 2 = 0 Ответы: 1). непосредственное интегрирование

2). понижение порядка с помощью подстановки y′x = p(x )

3). понижение порядка с помощью подстановки y′x = z(y ) 4). поиск частного решения методом неопределенных коэффициентов 5). поиск частного решения только методом вариации постоянных 81


Номер: 8.27.А Задача: Укажите метод решения дифференциального уравнения y′′′ = x ⋅ sin x Ответы: 1). непосредственное интегрирование

2). понижение порядка с помощью подстановки y′x = p(x )

3). понижение порядка с помощью подстановки y′x = z(y ) 4). поиск частного решения методом неопределенных коэффициентов 5). поиск частного решения только методом вариации постоянных

Задача:

y′′ −

Укажите

Номер: 8.28.А метод решения дифференциального

уравнения

y′ = x (x − 1) x −1

Ответы: 1). непосредственное интегрирование

2). понижение порядка с помощью подстановки y′x = p(x )

3). понижение порядка с помощью подстановки y′x = z(y )

4). поиск частного решения методом неопределенных коэффициентов 5). поиск частного решения только методом вариации постоянных

Задача:

Укажите

Номер: 8.29.А метод решения дифференциального

уравнения

2 yy′′ + (y′)2 = 0 Ответы: 1). непосредственное интегрирование

2). понижение порядка с помощью подстановки y′x = p(x )

3). понижение порядка с помощью подстановки y′x = z(y ) 4). поиск частного решения методом неопределенных коэффициентов 5). поиск частного решения только методом вариации постоянных

Задача:

Укажите

Номер: 8.30.А метод решения дифференциального

y′′′ ⋅ sin 4 x = sin 2x

82

уравнения


Ответы: 1). непосредственное интегрирование

2). понижение порядка с помощью подстановки y′x = p(x )

3). понижение порядка с помощью подстановки y′x = z(y ) 4). поиск частного решения методом неопределенных коэффициентов 5). поиск частного решения только методом вариации постоянных

Задача:

(

)

Укажите

Номер: 8.31.А метод решения дифференциального

уравнения

y′′ 1 − x 2 − xy′ = 2 Ответы: 1). непосредственное интегрирование

2). понижение порядка с помощью подстановки y′x = p(x )

3). понижение порядка с помощью подстановки y′x = z(y )

4). поиск частного решения методом неопределенных коэффициентов 5). поиск частного решения только методом вариации постоянных

Задача:

Укажите

Номер: 8.32.А метод решения дифференциального

уравнения

yy′′ − (y′)2 = 0 Ответы: 1). непосредственное интегрирование

2). понижение порядка с помощью подстановки y′x = p(x )

3). понижение порядка с помощью подстановки y′x = z(y ) 4). поиск частного решения методом неопределенных коэффициентов 5). поиск частного решения только методом вариации постоянных Номер: 8.33.А Задача: Укажите метод решения дифференциального уравнения y′′′ = xe − x Ответы: 1). непосредственное интегрирование

2). понижение порядка с помощью подстановки y′x = p(x )

3). понижение порядка с помощью подстановки y′x = z(y ) 4). поиск частного решения методом неопределенных коэффициентов 5). поиск частного решения только методом вариации постоянных

83


Задача:

Укажите

(1 + x )y′′ + 1 + y′ 2

2

Номер: 8.34.А метод решения дифференциального

уравнения

=0

Ответы: 1). непосредственное интегрирование

2). понижение порядка с помощью подстановки y′x = p(x )

3). понижение порядка с помощью подстановки y′x = z(y ) 4). поиск частного решения методом неопределенных коэффициентов 5). поиск частного решения только методом вариации постоянных

Задача:

Укажите

Номер: 8.35.А метод решения дифференциального

уравнения

yy′′ − (y′)2 = y 2 ln y Ответы: 1). непосредственное интегрирование

2). понижение порядка с помощью подстановки y′x = p(x )

3). понижение порядка с помощью подстановки y′x = z(y )

4). поиск частного решения методом неопределенных коэффициентов 5). поиск частного решения только методом вариации постоянных

Задача:

Укажите

Номер: 8.36.А метод решения дифференциального

уравнения

y′′ = 2 sin x ⋅ cos 2 x − sin 3 x Ответы: 1). непосредственное интегрирование

2). понижение порядка с помощью подстановки y′x = p(x )

3). понижение порядка с помощью подстановки y′x = z(y )

4). поиск частного решения методом неопределенных коэффициентов 5). поиск частного решения только методом вариации постоянных Номер: 8.37.А

y′ x 2 Задача: Укажите метод решения дифференциального уравнения y′′ = + x y′ Ответы: 1). непосредственное интегрирование

2). понижение порядка с помощью подстановки y′x = p(x ) 84


3). понижение порядка с помощью подстановки y′x = z(y ) 4). поиск частного решения методом неопределенных коэффициентов 5). поиск частного решения только методом вариации постоянных Номер: 8.38.А Задача: Укажите метод решения дифференциального уравнения y′′ =

y′ y

Ответы: 1). непосредственное интегрирование

2). понижение порядка с помощью подстановки y′x = p(x )

3). понижение порядка с помощью подстановки y′x = z(y )

4). поиск частного решения методом неопределенных коэффициентов 5). поиск частного решения только методом вариации постоянных

Задача:

y′′ + 4 y =

Укажите

Номер: 8.39.А метод решения дифференциального

уравнения

1 cos 2 x

Ответы: 1). непосредственное интегрирование

2). понижение порядка с помощью подстановки y′x = p(x )

3). понижение порядка с помощью подстановки y′x = z(y )

4). поиск частного решения методом неопределенных коэффициентов 5). поиск частного решения только методом вариации постоянных

Задача:

Укажите

y"−3y'+2 y =

1 x2 −1

Номер: 8.40.А метод решения дифференциального

уравнения

Ответы: 1). непосредственное интегрирование

2). понижение порядка с помощью подстановки y′x = p(x )

3). понижение порядка с помощью подстановки y′x = z(y ) 4). поиск частного решения методом неопределенных коэффициентов 5). поиск частного решения только методом вариации постоянных

85


Задача:

Укажите

y′′ − 4 y′ − 5 y =

Номер: 8.41.А метод решения дифференциального

уравнения

e2 x cos x

Ответы: 1). непосредственное интегрирование

2). понижение порядка с помощью подстановки y′x = p(x )

3). понижение порядка с помощью подстановки y′x = z(y ) 4). поиск частного решения методом неопределенных коэффициентов 5). поиск частного решения только методом вариации постоянных

Задача:

y"− y =

Укажите

Номер: 8.42.А метод решения дифференциального

уравнения

ex ex +1

Ответы: 1). непосредственное интегрирование

2). понижение порядка с помощью подстановки y′x = p(x )

3). понижение порядка с помощью подстановки y′x = z(y ) 4). поиск частного решения методом неопределенных коэффициентов 5). поиск частного решения только методом вариации постоянных

Задача:

Укажите

Номер: 8.43.А метод решения дифференциального

уравнения

y"−2 y = e x ⋅ x Ответы: 1). непосредственное интегрирование

2). понижение порядка с помощью подстановки y′x = p(x )

3). понижение порядка с помощью подстановки y′x = z(y )

4). поиск частного решения методом неопределенных коэффициентов 5). поиск частного решения только методом вариации постоянных

86


Задача:

y"+ y =

Укажите

Номер: 8.44.А метод решения дифференциального

уравнения

1 sin 3 x

Ответы: 1). непосредственное интегрирование

2). понижение порядка с помощью подстановки y′x = p(x )

3). понижение порядка с помощью подстановки y′x = z(y )

4). поиск частного решения методом неопределенных коэффициентов 5). поиск частного решения только методом вариации постоянных

Задача:

Укажите

y"−5 y'+6 y =

13 sin 3x

Номер: 8.45.А метод решения дифференциального

уравнения

Ответы: 1). непосредственное интегрирование

2). понижение порядка с помощью подстановки y′x = p(x )

3). понижение порядка с помощью подстановки y′x = z(y )

4). поиск частного решения методом неопределенных коэффициентов 5). поиск частного решения только методом вариации постоянных Номер: 8.46.А Задача: Укажите метод решения дифференциального уравнения y"+ y = tgx Ответы: 1). непосредственное интегрирование

2). понижение порядка с помощью подстановки y′x = p(x )

3). понижение порядка с помощью подстановки y′x = z(y )

4). поиск частного решения методом неопределенных коэффициентов 5). поиск частного решения только методом вариации постоянных

Задача:

y"−3y'+2 y =

Укажите

Номер: 8.47.А метод решения дифференциального

e2x ex + 1 87

уравнения


Ответы: 1). непосредственное интегрирование

2). понижение порядка с помощью подстановки y′x = p(x )

3). понижение порядка с помощью подстановки y′x = z(y ) 4). поиск частного решения методом неопределенных коэффициентов 5). поиск частного решения только методом вариации постоянных

Задача:

Укажите

( )

Номер: 8.48.А метод решения дифференциального

уравнения

y"− y = e 2 x ⋅ cos e x

Ответы: 1). непосредственное интегрирование

2). понижение порядка с помощью подстановки y′x = p(x )

3). понижение порядка с помощью подстановки y′x = z(y )

4). поиск частного решения методом неопределенных коэффициентов 5). поиск частного решения только методом вариации постоянных

Задача:

Укажите

Номер: 8.49.А метод решения дифференциального

уравнения

2e x y"−3y'+2 y = x cos 2 Ответы: 1). непосредственное интегрирование

2). понижение порядка с помощью подстановки y′x = p(x )

3). понижение порядка с помощью подстановки y′x = z(y )

4). поиск частного решения методом неопределенных коэффициентов 5). поиск частного решения только методом вариации постоянных Номер: 8.50.А Задача: Укажите метод решения дифференциального уравнения y"− y = Ответы: 1). непосредственное интегрирование

2). понижение порядка с помощью подстановки y′x = p(x )

3). понижение порядка с помощью подстановки y′x = z(y ) 88

ex ex + 1


4). поиск частного решения методом неопределенных коэффициентов 5). поиск частного решения только методом вариации постоянных

Задача:

Найдите

Номер: 8.51.В определитель Вронского

решений

уравнения

y′′ + 5 y′ + 6 y = 0 Ответы: 1). 5e − x

2). e −3x 3) 5e −2 x

4). e −5 x

5). e 2 x

Номер: 8.52.В Задача: Найдите определитель Вронского решений уравнения y′′ + 9 y = 0 Ответы: 1). 3 cos 3x

2). 9

3). 6

4). 3 sin 3x

5). 3

Номер: 8.53.В Задача: Найдите определитель Вронского решений уравнения y′′ − y′ − 2 y = 0 Ответы: 1). e − x

Задача:

2). e 2 x

Найдите

3). 2e − x

4). 3e 2 x

5). − 3e x

Номер: 8.54.В определитель Вронского

решений

уравнения

y′′ − 2 y′ + 10 y = 0 Ответы: 1). e x cos 3x

2). 6e 2 x

3). e x

4). e 3 x

5). − e x

Номер: 8.55.В Задача: Найдите определитель Вронского решений уравнения y′′ − 2 y′ + y = 0 Ответы: 1). 2e 2 x

2). e x

3). xe x

4). e 2 x

5). x

Номер: 8.56.В(6) Задача: Найдите определитель Вронского решений уравнения y′′ − 9 y = 0 Ответы: 1). 3e −3x

Задача:

2). e 3x

Найдите

3). − 6

4). 4

5). e

Номер: 8.57.В определитель Вронского

4 y′′ + 4 y′ + y = 0 89

решений

уравнения


Ответы: 1). − 2e

Задача:

−x

2). e

4x

3).

1 − x e 2

5). e − x

4). 1

Номер: 8.58.В определитель Вронского

Найдите

решений

уравнения

y′′ + 6 y′ + 13y = 0 Ответы: 1). 2e −3x

2). 4e −6 x

3). e −6 x

4). 8e −6 x

5). − e −6 x

Номер: 8.59.В Задача: Найдите определитель Вронского решений уравнения y′′ + 4 y = 0 Ответы: 1). − 2 sin 2x

2). 2

4). − 2

3). 4

5). 0

Номер: 8.60.В Задача: Найдите определитель Вронского решений уравнения y′′ + 2 y′ + y = 0 Ответы: 1). e −2 x

3). xe − x

2). x

5). xe −2 x

4). e 2 x

Номер: 8.61.В Задача: Найдите определитель Вронского решений уравнения y′′ − 7 y′ + 6 y = 0 Ответы: 1). 4e x

Задача:

2). − 5e 7 x

4). e −5 x

3). 5e 6 x

5). e − x

Номер: 8.62.В определитель Вронского

Найдите

решений

уравнения

y′′ + 6 y′ + 10 y = 0 Ответы: 1). e 3x cos x

2). sin x

3). 4e − x

4). e 3x

5). e −6 x

Номер: 8.63.В Задача: Найдите определитель Вронского решений уравнения y′′ + y = 0 Ответы: 1). cos x

Задача:

Найдите

2). 2

3). 1

4). 2x

5). sin x

Номер: 8.64.В определитель Вронского

решений

y′′ + 2 y′ + 2 y = 0 Ответы: 1). sin x

2). 2e 2 x

3). e − x

4). e −2 x 90

5). cos x

уравнения


Номер: 8.65.В Задача: Найдите определитель Вронского решений уравнения y′′ − 8y′ = 0 Ответы: 1). 3e −8 x

2). − 8e8 x

4). e8 x

3). 0

5). 1

Номер: 8.66.В Задача: Найдите определитель Вронского решений уравнения y′′ − 2 y′ = 0 Ответы: 1). e 2 x

Задача:

3). 2e 2 x

2). 1

4). e x

5). e3x

Номер: 8.67.В определитель Вронского

Найдите

решений

уравнения

y′′ − 3y′ − 10 y = 0 Ответы: 1). e5 x

3). 2e −2 x

2). 5e 2 x

4). − 7e3x

5). e x

Номер: 8.68.В Задача: Найдите определитель Вронского решений уравнения y′′ + 36 y = 0 Ответы: 1). 6

Задача:

2). 11

Найдите

3). 12

4). cos12 x

5). sin 6 x

Номер: 8.69.В определитель Вронского

решений

уравнения

6 y′′ + 7 y′ − 3y = 0 1

Ответы: 1). 4

Задача:

4

1 − x 2). e 3 6

Найдите

7

1 x 3). e 3 3

11 − x 4). e 6 6

5). 1

Номер: 8.70.В определитель Вронского

решений

уравнения

2 y′′ + 3y′ + y = 0 3

1 − x Ответы: 1). e 2 4

1

2). e

2x

2 − x 3). e 2 3

4). e

−x

5).

1 x e2

Номер: 8.71.В Задача: Найдите определитель Вронского решений уравнения y′′ + 3y′ = 0 Ответы: 1). 0

2). 3e −3x

3). 1

4). 2e3x 91

5). 5e3x


Номер: 8.72.В Задача: Найдите определитель Вронского решений уравнения y′′ − 6 y′ + 9 y = 0 Ответы: 1). 2e3x

Задача:

2). e3x

3). e6 x

4). xe3x

5). 3e6 x

Номер: 8.73.В определитель Вронского

Найдите

решений

уравнения

4 y′′ − 8 y′ + 3y = 0 Ответы: 1). e

Задача:

−x

2). − e

2x

3).

x e2

3

1 x 4). e 2 2

5). e 2 x

Номер: 8.74.В определитель Вронского

Найдите

решений

уравнения

9 y′′ + 3y′ − 2 y = 0 2

1 −3x 2). e 3

Ответы: 1). − 1

3).

2 − x e 3

4).

1 − x −e 3

5).

1 x 3 e

Номер: 8.75.В Задача: Найдите определитель Вронского решений уравнения y′′ − 7 y′ − 8 y = 0 Ответы: 1). 2

2). 8e8 x

3). − 9e7 x

4). e − x

5). − 6

Номер: 8.76.В Задача: Найдите определитель Вронского решений уравнения y′′ + 2 y′ + 5 y = 0 Ответы: 1). 2e −2 x

Задача:

2). 3e − x

Найдите

3). sin 2x

4). e − x

Номер: 8.77.В определитель Вронского

5). e 2 x

решений

уравнения

y′′ − 10 y′ + 16 y = 0 Ответы: 1). e −10 x

2). − 6e10 x

3). 9e3x

4). 4e x

5). 9

Номер: 8.78.В Задача: Найдите определитель Вронского решений уравнения y′′ − 6 y′ + 8 y = 0 Ответы: 1). e 4 x

2). 4e6 x

3). 3e 2 x

4). − 2e6 x 92

5). e 2 x


Задание:

Найдите

Номер: 8.79.В определитель Вронского

решений

уравнения

y′′ − 4 y′ + 20 y = 0 Ответы: 1). 3e −4 x

2). 2e 2 x cos 4 x

3). 5 sin 4x

4). 4e x

5). 4e 4 x

Номер: 8.80.В Задача: Найдите определитель Вронского решений уравнения y′′ − 64 y = 0 Ответы: 1). − 8

2). − 16

3). e −16 x

4). 8

93

5). − 16e −8 x


9. Дифференциальные уравнения n-ого порядка, допускающие понижение порядка. Три частных случая Номер: 9.1.А Задача: Найдите общее решение дифференциального уравнения y" = ax + b , где a, b – произвольные постоянные, a ≠ 0 методом непосредственного интегрирования 5 4a 2 Ответы: 1). y = ⋅ (ax + b ) 2 + c1x + c 2 15 2 4 ⋅ (ax + b ) 5 + c1x + c 2 3). y = 15 2 15 5). y = 2 ⋅ (ax + b ) 5 + c1x + c 2 4a

5 4 2 +c x +c ( ) ⋅ ax + b 1 2 2 15a 5 15 4). y = a 2 ⋅ (ax + b ) 2 + c1x + c 2 4

2). y =

Номер: 9.2.А Задача: Найдите общее решение дифференциального уравнения y"' = cos(ax + b), где a, b – произвольные постоянные, a ≠ 0 методом непосредственного интегрирования Ответы: 1). y = a −3 ⋅ cos(ax + b ) + c1x 2 + c 2 x + c 3 2). y = a 3 ⋅ sin (ax + b ) + c1x 2 + c 2 x + c 3

3). y = −a 3 ⋅ sin (ax + b ) + c1x 2 + c 2 x + c 3

4). y = −a −3 ⋅ cos(ax + b ) + c1x 2 + c 2 x + c 3 5). y = −a −3 ⋅ sin (ax + b ) + c1x 2 + c 2 x + c 3

Номер: 9.3.А Задача: Найдите общее решение дифференциального уравнения y′′′ =

1

(x − 3)3

методом непосредственного интегрирования

1 2

Ответы: 1). y = − ln x − 3 + c1x 2 + c 2 x + c 3 2). y = − ln x − 3 + c1x 2 + c 2 x + c 3 3). y =

1 ln x − 3 + c1x 2 + c 2 x + c 3 4). y = 2 ln x − 3 + c1x 2 + c 2 x + c 3 2

5). нет правильного ответа

Номер: 9.4.А Задача: Найдите общее решение дифференциального уравнения y′′ = методом непосредственного интегрирования

94

1 cos 2 x


Ответы: 1). y = ln cos x + c1x + c 2 3). y = ln cos x + c1 + c 2 5). нет правильного ответа Задача:

Найдите

общее

1 + c1x + c 2 cos x 4). y = ln sin x + c1x + c 2 2). y = ln

Номер: 9.5.А решение дифференциального

уравнения

1 + 6 x методом непосредственного интегрирования sin 2 x 1 Ответы: 1). y = ln + x 3 + c1x + c 2 2). y = ln sin x + x 3 + c1x + c 2 sin x 1 + x 3 + c1x + c 2 4). y = ln cos x + x 3 + c1x + c 2 3). y = ln cos x y′′ =

5). нет правильного ответа Номер: 9.6.А Задача: Найдите общее решение дифференциального 3x y′′′ = e − sin x методом непосредственного интегрирования

уравнения

Ответы: 1). y = 27 ⋅ e 3x − cos x + c1x 2 + c 2 x + c 3 2). y = 27 ⋅ e 3x + cos x + c1x 2 + c 2 x + c 3

1 3x e + cos x + c1x 2 + c 2 x + c3 27 1 3x e − cos x + c1x 2 + c 2 x + c 3 4). y = 27 3). y =

5). нет правильного ответа

Номер: 9.7.А Задача: Найдите общее решение дифференциального y′′ = cos x + e 2 x методом непосредственного интегрирования Ответы: 1). нет правильного ответа 3). y = cos x + 4e 2 x + c1x + c 2

уравнения

1 2x e + c1x + c 2 4 4). y = − cos x + 4e 2 x + c1x + c 2

2). y = cos x +

5). y = − cos x + 0,25 ⋅ e 2 x + c1x + c 2 Номер: 9.8.А Задача: Найдите общее решение дифференциального уравнения y′′ =

методом непосредственного интегрирования 95

1 x2


Ответы: 1). нет правильного ответа 1 4). y = ln + c1 x + c 2 x

2). y = ln x + c1x + c 2 1 5). y = ln + c1 + c 2 x

3). y = ln x + c1 + c 2

Номер: 9.9.А Задача: Найдите общее решение дифференциального уравнения y′′ = методом непосредственного интегрирования Ответы: 1). нет правильного ответа 3). y =

x 4e 2

5). y =

x 4e 2

x3 + + c1x + c 2 3

2). y =

x 0,25 ⋅ e 2

4). y =

x 0,25 ⋅ e 2

x e2

+ 2x

x3 + + c1x + c 2 3

+ c1x + c 2

+ 2 x + c1x + c 2

Номер: 9.10.А Задача: Найдите общее решение дифференциального уравнения y′′′ = 2 x методом непосредственного интегрирования Ответы: 1). нет правильного ответа 2). y = ln 3 2 ⋅ 2 x + c1x 2 + c 2 x + c3 x

2

3). y = 2 + c1x + c 2 x + c3

2x 4). y = + c1x 2 + c 2 x + c3 ln 2

2x 5). y = 3 + c1x 2 + c 2 x + c3 ln 2 Номер: 9.11.А Задача: Найдите общее решение дифференциального 2 y′′′ = cos 3x + 3x методом непосредственного интегрирования

x5 1 ⋅ sin 3x + + c1x 2 + c 2 x + c3 Ответы: 1). y = − 20 27 x5 1 2). y = ⋅ sin 3x + + c1x 2 + c 2 x + c3 20 27 x5 3). y = −27 ⋅ sin 3x + + c1x 2 + c 2 x + c3 20 3x 5 + c1x 2 + c 2 x + c3 4). y = +27 ⋅ cos 3x + 20 5). нет правильного ответа

96

уравнения


Номер: 9.12.А Задача: Найдите общее решение дифференциального уравнения y′′ =

1 (x − 1)2

методом непосредственного интегрирования Ответы: 1). y = ln x − 1 + c1x + c 2 3). y = − ln x − 1 + c1x + c 2 5). нет правильного ответа

1 + c1x + c 2 x −1 4). y = ln −1 x − 1 + c1x + c 2 2). y =

Номер: 9.13.А Задача: Найдите общее решение дифференциального уравнения y"⋅tgx − y'−1 = 0 методом понижения порядка, пользуясь подстановкой

y ' = p (x )

Ответы: 1). y = x + c1 ⋅ cos x + c 2 2). y = − x − c1 ⋅ cos x + c 2 3). y = −x + c1x + c 2 4). y = − x + c1 sin x + c 2 5). y = x − c1 sin x + c 2 Номер: 9.14.А Задача: Найдите общее решение дифференциального уравнения методом понижения порядка, пользуясь подстановкой y' = p(x ) 2

Ответы: 1). y = c1x − x + c 2 2

4). y = c1x + x + c 2

x2 2). y = − x + c1x + c 2 2 x2 5). y = + x + c1x + c 2 2

y"⋅x = y'+1

x2 3). y = − c1x + c 2 2

Номер: 9.15.А Задача: Решите дифференциальное уравнение y' = x ⋅ y"+ y" , удовлетворяющее

1 y' (0) = 1 методом понижения порядка, пользуясь y(0) = ; 2 подстановкой y' = p(x )

условиям:

2

Ответы: 1). 2 y = x + x + 1 4). y +

1 = (x + 1)2 2

x2 2 3). 2 y = (x + 1) 2 1 x2 5). y − = −x 2 2

2). y =

Номер: 9.16.А Задача: Найдите частное решение дифференциального уравнения x ⋅ y"⋅ ln x − y' = 0 , удовлетворяющее условиям: y(e ) = 0 ; y' (e ) = 1 методом понижения порядка, пользуясь подстановкой y' = p(x ) 97


x2 3 Ответы: 1). y = + (e − 1)x + e − e 2 2 2 4). y = x (ln x − 1)

2). y = ln x − 1

3). y = x − x ln x

5). y = 1 − ln x

Номер: 9.17.А Задача: Найдите общее решение дифференциального уравнения 2 x x ⋅ y"− y' = x ⋅ e методом понижения порядка, пользуясь подстановкой

y ' = p (x )

Ответы: 1). y = x ⋅ e x + e x + c1x 2 + c 2

2). y = e x + c1x + c 2 4). y = x ⋅ e x − e x + c1x + c 2

3). y = x ⋅ e x − e x + c1 + c 2 5). y = x ⋅ e x − e x + c1x 2 + c 2 Задача:

Найдите

общее

Номер: 9.18.А решение дифференциального

уравнения

x ⋅ y"− y' = x 2 ⋅ cos x методом понижения порядка, пользуясь подстановкой y ' = p (x )

Ответы: 1). y = −x ⋅ cos x + sin x + c1x 2 + c 2 3). y = x ⋅ cos x − sin x + c1x 2 + c 2 5). y = − x ⋅ cos x + sin x + c1x + c 2

2). y = − x ⋅ cos x − sin x + c1x 2 + c 2 4). y = x ⋅ sin x − cos x + c1x 2 + c 2

Номер: 9.19.А Задача: Решите дифференциальное уравнение y"−4 y ⋅ y' = 0 , удовлетворяющее условиям: y(0 ) = 0 ; y' (0 ) = 8 методом понижения порядка, пользуясь подстановкой y' = p(y ) Ответы: 1). arctgy = 2 x

2). y = 2 ⋅ tg 4 x

3). 2 y 3 = 3y

⎛ y⎞ ⎝ 2⎠

4). arctg⎜ ⎟ = 2 x

5). y = 2 ⋅ tg 2 x

Номер: 9.20.А Задача: Решите дифференциальное уравнение 3y'⋅y" = 2 y , удовлетворяющее условиям: y(0 ) = 1; y' (0 ) = 1 методом понижения порядка, пользуясь подстановкой y' = p(y ) Ответы: 1). y 4).

3

5

3

5 = x +1 3

y = x +1

2). y =

3

2

y ⋅ x +1

5). y − 1 =

y3 ⋅ x

98

⎛x ⎞ 3). y = ⎜ + 1⎟ ⎝3 ⎠

3


Номер: 9.21.А удовлетворяющее Задача: Решите дифференциальное уравнение y" = y , условиям: y(0 ) = 0 ; y' (0 ) = 1 методом понижения порядка, пользуясь подстановкой y' = p(y ) Ответы: 1). ln y + 4). y = tgx

y2 + 1 = x

(

)

2). y 2 = y 2 + 1 ⋅ x 2

3). arcsin y = ± x

5). y = ± y 2 + 1 ⋅ x Номер: 9.22.В

Задача:

Решите

дифференциальное

уравнение

непосредственного интегрирования

1 ln x + c1 + c 2 x + c 3 2 1 + c1x + c 2 3). y = 2x 1 5). y = − 2 + c1x + c 2 + c 3 2x

Ответы: 1). y =

2). y = ln

Номер: 9.23.В Задача: Решите дифференциальное уравнение непосредственного интегрирования

sin 4 x + c1x 3 + c 2 x 2 + c 3 256 cos 4x + c1x 3 + c 2 x 2 + c 3 3). y = 256 cos 4 x − c1x 3 + c 2 x 2 + c3 5). y = 256

1 x3

методом

x + c1x 2 + c 2 x + c3

4). y = − ln

Ответы: 1). y =

y′′′ =

x + c1x + c 2 x + c3

y IV = sin 4x

методом

sin 4 x + c1x 3 + c 2 x 2 + c 3 256 cos 4 x 4). y = − + c1x 3 + c 2 x 2 + c 3 256

2). y = −

Номер: 9.24.В Задача: Найдите общее решение дифференциального уравнения y v = e ax + b , где a, b – произвольные постоянные, a ≠ 0 методом непосредственного интегрирования Ответы: 1). y = a 5 ⋅ e ax + b + c1x 4 + c 2 x 3 + c 3 x 2 + c 4 x + c 5 2). y = a 4 ⋅ e ax + b + c1x 4 + c 2 x 3 + c 3 x 2 + c 4 x + c 5 3). y = a −5 ⋅ e ax + b + c1x 4 + c 2 x 3 + c 3 x 2 + c 4 x + c 5 4). y = a −4 ⋅ e ax + b + c1x 4 + c 2 x 3 + c 3 x 2 + c 4 x + c 5 5). y = e ax + b + c1x 4 + c 2 x 3 + c 3 x 2 + c 4 x + c 5 99


Номер: 9.25.В Задача: Найдите общее решение дифференциального уравнения y" =

1 , ax + b

где a, b – произвольные постоянные, a ≠ 0 методом непосредственного интегрирования

x b ⋅ (ln ax + b − 1) + 2 ⋅ ln ax + b + c1x + c 2 a a b y = xa ⋅ (ln ax + b − 1) + 2 ⋅ ln ax + b + c1x + c 2 a b y = xa ⋅ (ln ax + b + 1) − 2 ⋅ ln ax + b + c1x + c 2 a y = xa ⋅ (ln ax + b + 1) + b ⋅ a 2 ⋅ ln ax + b + c1x + c 2 x b y = ⋅ (ln ax + b + 1) − 2 ⋅ ln ax + b + c1x + c 2 a a

Ответы: 1). y = 2). 3). 4). 5).

Номер: 9.26.В Задача:

Решите

дифференциальное

уравнение

y" =

y' ⎛ y' ⎞ ⋅ ⎜1 + ln ⎟ , x ⎝ x⎠

удовлетворяющее условиям: y(1) = 0; y' (1) = e методом понижения порядка, пользуясь подстановкой y' = p(x ) Ответы: 1). y = e (x − 1) x

3). 3y = x 3 − 1

2). 2 y = x 2 − 1

4). y = x ⋅ e x − 1

2

5). y = x − 1 Номер: 9.27.В Задача: Решите дифференциальное уравнение y"+ условиям: y(2) = 1 ; y' (2 ) = 2 подстановкой y' = p(x ) Ответы: 1). 4 y = x 4 − 12

методом

2). 16 y = x 4

y' = x 2 , удовлетворяющее x

понижения 2

3). 2 y = x − 2

порядка,

пользуясь

4). 2 y = 6 − x 2

5). 4 y = 20 − x 4 Номер: 9.28.В Задача: Найдите общее решение дифференциального уравнения x ⋅ y"+ y'+ x = 0 методом понижения порядка, пользуясь подстановкой y' = p(x )

1 2 1 2 2 Ответы: 1). y = x ⋅ ln x − x + c1x + c 2 2 4 100

x2 + c2 2). y = − x − c1 2


x2 x3 + c1x + c 2 4). y = − + c1 ln x + c 2 3). y = − 4 9 2 x + c1 ln x + c 2 5). y = − 4 Номер: 9.29.В Задача: Найдите общее решение дифференциального уравнения x ⋅ y"− y' = 9x 2 ⋅ ln x методом понижения порядка, пользуясь подстановкой

y ' = p (x )

( ) 2 3 3). y = x (3 ln x − 4) + c1x + c 2 3 5). y = 3x (ln x − 1) + c1x + c 2 3

Ответы: 1). y = x 3 ln x − 4 + c1x + c 2

2 ) 3 2 4). y = 3x (ln x − 1) + c1x + c 2 3

(

2). y = x 3 ln x − 7 + c1x + c 2

Номер: 9.30.В Задача: Найдите общее решение дифференциального уравнения x 3 ⋅ y"+ x 2 ⋅ y'−1 = 0 методом понижения порядка, пользуясь подстановкой

y ' = p (x )

Ответы: 1). y = 3). y = −

1 + c1 ⋅ ln x + c 2 x

1 + c1 ⋅ ln x + c 2 x

2). y = −

4). y =

1 + c1 ⋅ ln x + c 2 2x 2

1 + c1 ⋅ x 2 + c 2 3x

5). y =

1 + c1 ⋅ ln x + c 2 2x 2

Номер: 9.31.В Задача: Найдите общее решение дифференциального уравнения x ⋅ y"+ y' = ln x + 1 методом понижения порядка, пользуясь подстановкой

y ' = p (x )

2). y = x (ln x − 1) + c1 ln x + c 2

Ответы: 1). y = − x (ln x + 1) + c1x 2 + c 2

3). y = x ⋅ ln x + x + c1x + c 2 4). y = x (ln x + 1) + c1x 2 + c 2 5). y = − x (ln x − 1) + c1 ln x + c 2 Номер: 9.32.В

Задача: Решите дифференциальное уравнение 2 y ⋅ y"+(y') + (y') = 0 , удовлетворяющее условиям: y(0 ) = 1; y' (0 ) = 1 методом понижения порядка, пользуясь подстановкой y' = p(y ) 2

2

Ответы: 1). x − 2 y + 1 = 0 4).

2). y =

(2 y − 1)2 = (3x + 1)3

2y − 1

3). (2 y − 1) = (3x + 1)

5). (2 y − 1)3 + (3x + 1)2 = 0 101

4

3

2


Номер: 9.33.В Задача: Решите дифференциальное уравнение y"⋅tgy понижения порядка, пользуясь подстановкой y' = p(y )

sin 3 y + c2 x Ответы: 1). y = c1 ⋅ 3 4). ctgy + c1x + c 2 = 0

2). tgy + c1x = c 2

= 2(y')2 методом

3). tgy − c1x − c 2 = 0

5). 4 x = c1 (2 y − sin 2 y ) + c 2 Номер: 9.34.В

Задача: Решите дифференциальное уравнение (1 + y ) ⋅ y"−5(y') = 0 методом понижения порядка, пользуясь подстановкой y' = p(y ) 2

2). (c1 x + c 2 ) ⋅ (y + 1) = 1

Ответы: 1). 6 y = (y + 1) ⋅c1 + c 2

4

6

3). (y + 1) ⋅ (c1x + c 2 ) + 4 = 0

4). ln y + 1 = 5c1x + c2

4

5). 6 x − c 2 = (y + 1) ⋅ c1 6

Номер: 9.35.В

2(y') = y" (y − 1) , Задача: Решите дифференциальное уравнение удовлетворяющее условиям: y(1) = 2; y' (1) = −1 методом понижения порядка, пользуясь подстановкой y' = p(y ) 2

Ответы: 1). y = (y − 1) ⋅ x + 1 2

4). y =

x −3 x−2

2). y + (y − 1)2 x − 3 = 0 5). y =

3).

1 = 2−x y −1

x +1 x

Номер: 9.36.В

Задача: Решите дифференциальное уравнение y"+ (y ') = 0 понижения порядка, пользуясь подстановкой y' = p(y ) 3

Ответы: 1). y 2 + c1 y = 2 x + c 2

2). y 2 + c1 y + 2 x = c 2

4). x = ln y + c1 + c 2

5). y = ln y + c1x + c 2

методом

3). y = ln y + c1 + c 2

Номер: 9.37.В 2 Задача: Решите дифференциальное уравнение y" 9 + y 2 − 2 y(y') = 0 методом понижения порядка, пользуясь подстановкой y' = p(y )

(

102

)


⎛ y⎞ Ответы: 1). arctg⎜ ⎟ = c1x + c 2 ⎝ 3⎠

(

3

)

3). 3x = c1 27 y + y + 3c 2

(

2

)

⎛ y3 ⎞ ⎜ ⎟ + c2 2). y = c1 ⎜ 9 y + 3 ⎟⎠ ⎝ 4). arctgy = 3c1 x + 3c 2

5). ln y + 9 = 3c1x + 3c 2 Номер: 9.38.В Задача:

Решите

дифференциальное

y"+ y'⋅ 1 − (y')2 = 0 ,

уравнение

удовлетворяющее условиям: y(0) = 0; y' (0 ) = 1 методом понижения порядка, пользуясь подстановкой y' = p(y ) Ответы: 1). 1 + sin y = e (1 − sin y ) x

4). y = x ⋅ cos y

⎛ y π⎞ x + ⎟=e 3). y − sin y = 0 ⎝ 2 4⎠ 2x 5). 1 − sin y = e (1 + sin y ) 2). tg⎜

Номер: 9.39.С Задача: Найдите общее решение дифференциального уравнения (x + 1) ⋅ y"−2y'+ x + 2 = 0 методом понижения порядка, пользуясь подстановкой

y ' = p (x )

1 (x + 1)2 ⎛⎜ 1 − ln(x + 1)⎞⎟ + c2 2 ⎝2 ⎠ 1 1 3 3 2). y = − x 2 + 3x + c1 (x + 1) + c 2 3). y = x 2 + 3x + c1 (x + 1) + c 2 2 2 x2 3 3 2 5). y = − x + c1 (x + 1)3 + c 2 4). y = x + 3x + c1 (x + 1) + c 2 2 2

Ответы: 1). y = c1x +

(

Задача:

)

Найдите

(

общее

1 x ⋅ (y')2 ⋅ y" = (y')3 + x 4 3

)

Номер: 9.40.С решение дифференциального методом

понижения

порядка,

уравнения пользуясь

подстановкой y' = p(x )

3c ⎞ 4 ⎛ 3 3 ⋅ x + c1 ⋅ ⎜ x − 1 ⎟ + c 2 4 ⎠ 7 ⎝ 7c ⎞ 4 ⎛ 3 2). y = ⋅ 3 x + c1 ⋅ ⎜ x − 1 ⎟ + c 2 4 ⎠ 7 ⎝ y 4 x 5 c1 x 4 = + + c2 4). 4 5 4

Ответы: 1). y =

103

x 5 c1 x 4 + + c2 3). y = 5 4 5). нет правильного ответа


Номер: 9.41.С

y ⋅ y"+ (y')2 + y 2 ⋅ ln y = 0 , 1 методом понижения удовлетворяющее условиям: y − 2 = 1; y' − 2 = 8 порядка, пользуясь подстановкой y' = p(y ) Задача:

Решите

дифференциальное

(

Ответы: 1). x 2 + 8 ln y = 2

уравнение

)

2). y = e 2− x

4). 4 ⋅ 2 − 8 ln y = x + 5 2

(

2

)

3).

2 ⋅ 1 − 4 ln y = x

5). 2 ⋅ 2 − 8 ln y = x + 3 2

104


10. Линейные однородные дифференциальные уравнения n-ого порядка с постоянными коэффициентами Номер: 10.1.А Задача: Найти общее решение дифференциального уравнения y"−4 y'+3y = 0 Ответы: 1). y = c1e − x + c 2e −3x 4). y = c1e x + c 2e −3x

3). y = c1e − x + ce3x

2). y = c1e x + c 2e3x 5). y = c1e x (1 + x )

Номер: 10.2.А Задача: Найти общее решение дифференциального уравнения y"−4 y'+4 y = 0 Ответы: 1). y = e 2 x (c1 + c 2 x )

2). y = e −2 x (c1 + c 2 x )

3). y = c1e 2 x + c 2e −2 x

4). y = c1e 2 x + c 2e −2 x ⋅ x 5). y = c1e 2 x + c 2e x Номер: 10.3.А Задача: Найти общее решение дифференциального уравнения y"−4 y'+13y = 0 Ответы: 1). y = e −2 x (c1 sin 3x − c 2 cos 3x ) 3). y = e 2 x (c1 sin 3x + c 2 cos 3x )

2). y = e 2 x (c1 sin 3x − c 2 cos 3x )

4). y = (c1 sin 3x + c 2 cos 3x )e −2 x

5). y = c1e x sin 3x + c 2e 2 x cos 3x Номер: 10.4.А Задача: Найти общее решение дифференциального уравнения y"+4 y = 0 Ответы: 1). y = c1 sin 4 x + c 2 cos 4 x 2). y = c1tg 2 x − c 2 ctg 2 x 4). y = c1 sin 2 x + c 2 cos 2 x 3). y = c1 sin x + c 2 cos x 5). y = c1 sin 3x + c 2 cos 3x Номер: 10.5.А Задача: Найти общее решение дифференциального уравнения 2 y"−3y'+ y = 0 Ответы: 1). y = c1e x + c 2e

x

2

4). y = c1e − x + c 2e 2 x

3). y = c1e − x + c 2e −2 x

2). y = c1e x + c 2e 2 x 5). y = c1e − x + c 2e

−x

2

Номер: 10.6.А Задача: Найти общее решение дифференциального уравнения 2 y"+ y'+3y = 0

23 23 ⎞ − x4 x + c 2 cos x ⎟⎟e 4 4 ⎠ ⎝ ⎛ 23 23 ⎞ 2 x 3). y = ⎜⎜ c1 sin x + c 2 cos x ⎟⎟e 4 4 ⎝ ⎠

Ответы: 1). y = ⎜⎜ c1 sin

105

2). y = c1e

− x4

+ c 2e 2 x

4). y = c1e − x + c 2e x


23 ⎞ 23 x + c 2 cos x⎟ 4 4 ⎟⎠

5). y = e − x ⎜⎜ c1 sin

Номер: 10.7.А Задача: Найти общее решение дифференциального уравнения 4 y"+4 y'+ y = 0 Ответы: 1). y = (c1 + c 2 x )e −2 x x

4). y = (c1 + c 2 x )e 2

2). y = (c1 + c 2 x )e 2 x

3). y = (c1 + c 2 x )e

− x2

5). y = (c1 + c 2 x )e x

Номер: 10.8.А Задача: Найти общее решение дифференциального уравнения 3y"+7 y'+4 y = 0 Ответы: 1). y = c1e 2 x + c 2e −3x 4). y = c1e − x + c 2e

− 43x

2). y = c1e −2 x + c 2e −3x

3). y = c1e x + c 2e 2 x

5). y = c1e x + c 2e3x

Номер: 10.9.А Задача: Найти общее решение дифференциального уравнения y"+3y'+2 y = 0 Ответы: 1). y = c1e 2 x + c 2e − x 4). y = c1e 2 x + c 2e x

2). y = c1e −2 x + c 2e − x

3). y = c1e −2 x + c 2e x

5). y = c1e 2 x + c 2e3x

Номер: 10.10.А Задача: Найти общее решение дифференциального уравнения y"+2ay'+ a 2 y = 0 , где a-произвольная постоянная Ответы: 1). y = e ax (c1 + c 2 x ) x

4). y = e a (c1 + c 2 x )

5).

− xa

(c1x + c2 ) y = e − ax (c1x + c 2 )

2). y = e

3). y = e

− xa

(c1x + c2 )

Номер: 10.11.А Задача: Найти общее решение дифференциального уравнения y"+2 y'+5 y = 0 Ответы: 1). y = e − x (c1 cos 2 x + c 2 sin 2 x )

3). y = e −2 x (c1 cos 2 x + c 2 sin 2 x )

5). y = e 2 x (c1 cos x + c 2 sin x )

2). y = e x (c1 cos 2 x + c 2 sin 2 x )

(

4). y = e − x c1 cos x 6 + c 2 sin x 6

Номер: 10.12.А Задача: Найти общее решение дифференциального уравнения y"−2 y'−3y = 0 Ответы: 1). y = c1e −3x + c 2e x 4). y = c1e3x + c 2e x

2). y = c1e −3x + c 2e − x 5). y = c1e3x + c 2e 2 x

106

3). y = c1e3x + c 2e − x

)


Номер: 10.13.А Задача: Найти общее решение дифференциального уравнения y"+ y = 0

x x + c 2 cos 2). y = c1 sin x + c 2 cos x 2 2 3). y = c1 sin 2 x + c 2 cos 2 x 4). y = c1 sin 3x + c 2 cos 3x x x 5). y = c1 sin + c 2 cos 3 3

Ответы: 1). y = c1 sin

Номер: 10.14.А Задача: Найти общее решение дифференциального уравнения y"−5 y'+6 y = 0 Ответы: 1). y = c1e −2 x + c 2e −3x 4). y = c1e 2 x + c 2e3x

2). y = c1e 2 x + c 2e −3x

3). y = c1e −2 x + c 2e3x

x

x

5). y = c1e 2 + c 2e 3

Номер: 10.15.А Задача: Найти общее решение дифференциального уравнения y"−5 y'+4 y = 0 Ответы: 1). y = c1e − x + c 2e −4 x 4). y = c1e − x + c 2e 4 x

2). y = c1e x + c 2e −4 x

3). y = c1e 2 x + c 2e 4 x

5). y = c1e x + c 2e 4 x

Номер: 10.16.А Задача: Найти общее решение дифференциального уравнения y"−6 y'+9 y = 0 Ответы: 1). y = e3x (c1 + c 2 x ) x

4). y = e 2 (c1 + c 2 x )

2). y = e 2 x (c1 + c 2 x )

3). y = e x (c1x + c 2 )

x

5). y = e 3 (c1 + c 2 x )

Номер: 10.17.А Задача: Найти общее решение дифференциального уравнения y"−2 y'+2 y = 0 Ответы: 1). y = e 2 x (c1 sin x + c 2 cos x )

2). y = e x (c1 sin x + c 2 cos x )

3). y = e 4 x (c1 sin x + c 2 cos x )

4). y = e 2 (c1 sin x + c 2 cos x )

x

5). y = e x (c1 sin 2 x + c 2 cos 2 x )

Номер: 10.18.А Задача: Найти общее решение дифференциального уравнения y"+4 y'+9 y = 0

(

Ответы: 1). y = e 2 x c1 cos 5x + c 2 sin 5x

(

)

3). y = e −2 x c1 cos 5x + c 2 sin 5x x 2

5). y = e (c1 cos 2x + c 2 sin 2 x )

107

)

2). y = e 4 x (c1 cos 3x + c 2 sin 3x )

4). y = e −4 x (c1 cos 3x + c 2 sin 3x )


Номер: 10.19.А Задача: Найти общее решение дифференциального уравнения y"−2 y'+10 y = 0 Ответы: 1). y = e x (c1 cos x + c 2 sin x )

2). y = e − x (c1 cos 2 x + c 2 sin 2 x )

3). y = e x (c1 cos 2 x + c 2 sin 2 x )

4). y = e − x (c1 cos 3x + c 2 sin 3x )

5). y = e x (c1 cos 3x + c 2 sin 3x )

Номер: 10.20.А Задача: Найти общее решение дифференциального уравнения y"−9 y = 0 Ответы: 1). y = c1e 2 x + c 2e −2 x 4). y = c1e x + c 2e − x

2). y = c1e3x + c 2e −3 x

x

3). y = c1e 3 + c 2e

− x3

5). y = c1e9 x + c 2e −9 x

Номер: 10.21.А Задача: Найти общее решение дифференциального уравнения y"−4 y'+2 y = 0 Ответы: 1). y = c1e 2 x + c 2e

(

3). y = e 2 x c1e x

2

+ c 2e − x

2

2x

)

(

2). y = e 2 x c1e 2 x + c 2e −2 x 4). y = c1e −2 x + c 2e 2 x

)

5). y = c1e(2 +

) +c 2

2 x

Номер: 10.22.А Задача: Найти общее решение дифференциального уравнения y"− y' = 0 Ответы: 1). y = c1x + c 2e x

2). y = c1 + c 2e x

4). y = c1 + c 2e 2 x

5). y = c1 + c 2e 2

3). y = c1 + c 2e − x

x

Номер: 10.23.А Задача: Найти общее решение дифференциального уравнения y"−2 y'+2 y = 0 Ответы: 1). y = e x (c1 cos 2 x + c 2 sin 2 x ) x

3). y = e 2 (c1 cos x + c 2 cos x )

2). y = e 2 x (c 2 cos 2 x + c1 sin 2 x ) 4). y = e 2 x (c1 cos x + c 2 sin x )

5). y = e x (c1 cos x + c 2 sin x )

Номер: 10.24.А Задача: Найти общее решение дифференциального уравнения 3y"− y'+ y = 0 x

2). y = e 3 (c1 cos 3x + c 2 sin 3x )

Ответы: 1). y = e3x (c1 cos x + c 2 sin x ) − x6 ⎛

11 11 ⎞ 3). y = e ⎜⎜ c1 cos x + c 2 sin x ⎟⎟ 6 6 ⎝ ⎠ 5). y = e 2 x (c1 cos 2 x + c 2 sin 2 x )

108

x 6

4). y = e ⎜⎜ c1 cos

11 11 ⎞ x + c 2 sin x⎟ 6 6 ⎟⎠


Номер: 10.25.А Задача: Найти общее решение дифференциального уравнения y"+4 y'+13y = 0 Ответы: 1). y = e 2 x (c1 cos 3x + c 2 sin 3x )

2). y = e −2 x (c1 cos 3x + c 2 sin 3x )

x

3). y = e 2 (c1 cos 3x + c 2 sin 3x )

4). y = e

5). y = e x (c1 cos 3x + c 2 sin 3x ) Задача:

Найти

общее

y" '−5y"+8y'−4 y = 0

− x2

(c1 cos 3x + c2 sin 3x )

Номер: 10.26.В решение дифференциального

Ответы: 1). y = c1e x + e 2 x (c 2 + c3 x )

уравнения

2). y = c1e 2 x + e x (c 2 + c3 x )

(

3). y = c1e x + c 2e 2 x + c3e3x

4). y = e x c1 + c 2 x + c3 x 2

5). y = (c1 + c 2 x )e x + c3e 2 x

)

Номер: 10.27.В Задача: Найти общее решение дифференциального уравнения y" '−8 y = 0 Ответы: 1). y = c1e − x + e 2 x (c 2 cos 3x + c3 sin 3x )

(

2). y = c1e 2 x + e − x c 2 sin 3x + c3 cos 3x

(

3). y = c1e 2 x + e x c 2 cos 3x + c3 sin 3x

(

)

4). y = c1e −2 x + e x c 2 cos 3x + c3 sin 3x

(

)

)

5). y = c1e −2 x + e − x c 2 cos 3x + c3 sin 3x

)

Номер: 10.28.В Задача: Найти общее решение дифференциального 2 3 y" '+3ay"+3a y'+ a y = 0 , где a-произвольная постоянная Ответы: 1). y = c1e − ax + c 2e ax + c3e 2ax

(

3). y = e − ax c1 + c 2 x + c3 x 2

)

уравнения

2). y = c1e ax + c 2e 2ax + c3e3ax 4). y = c1e − ax + c 2e −2ax + c3e −3ax

5). y = c1e − ax + c 2e 2ax + c3e −3x Номер: 10.29.В Задача: Найти общее решение дифференциального уравнения y" '−5 y"+6 y' = 0 Ответы: 1). y = c1e − x + c 2e 2 x + c3e −3x 3). y = c1e x + c 2e −2 x + c3e3x

2). y = c1e x + c 2e 2 x + c3e3x 4). y = c1 + c 2e 2 x + c3e3x

5). y = c1e 2 x + c 2 + c3e −3x

109


Задача:

Найти

y" '−5y"+8y'−4 y = 0

общее

Номер: 10.30.В решение дифференциального

Ответы: 1). y = c1e x + c 2e 2 x + c3e3x 3). y = c1e − x + c 2e −2 x + c3e −3x

5). y = c1e x + e 2 x (c 2 + c3 x )

2). y = e x (c1 + c 2 x ) + e 2 x 4). y = c1e 2 x + c 2e − x + c3e −2 x

110

уравнения


11. Линейные неоднородные дифференциальные уравнения n-ого порядка с постоянными коэффициентами со специальной правой частью Номер: 11.1.А Задача: Частное решение неоднородного уравнения y ′′ + 7 y ′ = −5 x ищем в виде Ответы: 1). A x 2 + B x 2). A x + B 3). A x 2 4). A x 5). нет правильного ответа Номер: 11.2.А Задача: Частное решение неоднородного уравнения y ′′ − 10 y ′ + 25 y = x ⋅ e 5 x ищем в виде

(

)

Ответы: 1). A x 2 + Bx ⋅ e 5 x 4). A x 3 ⋅ e 5 x

(

)

2). A x 3 + Bx 2 ⋅ e 5 x

3). (A x + B) ⋅ e 5 x

5). нет правильного ответа Номер: 11.3.А

Задача: Частное решение неоднородного уравнения y ′′ + 8 y ′ + 16 y = − 4 e −4 x ищем в виде Ответы: 1). − 4 A ⋅ e −4 x 2). A ⋅ e −4 x 3). x 2 ⋅ A ⋅ e −4 x 4). x ⋅ e 4 x 5). x 2 ⋅ A ⋅ e 4 x Номер: 11.4.А Задача: Частное решение неоднородного уравнения y ′′ + 5 y ′ = e −5 x ищем в виде Ответы: 1). A ⋅ e −5 x 2). x ⋅ e −5 x 3). x ⋅ A ⋅ e −5 x 5). нет правильного ответа

4). A ⋅ e −5 x

Номер: 11.5.А Задача: Частное решение неоднородного уравнения y ′′ − 14 y ′ + 53 y = cos 2 x ищем в виде Ответы: 1). A ⋅ cos 2 x 2). A ⋅ x ⋅ cos 2 x 4). A cos 2 x + B ⋅ sin 2 x

Задача:

Частное

3). (A ⋅ cos 2 x + B ⋅ sin 2 x ) ⋅ x 5). нет правильного ответа

Номер: 11.6.А решение неоднородного

y ′′ − 14 y ′ + 53 y = e 7 x ⋅ sin 2 x ищем в виде 111

уравнения


Ответы: 1). A ⋅ e 7 x ⋅ sin 2 x

2). e 7 x (A ⋅ cos 2 x + B ⋅ sin 2 x )

4). x ⋅ e 7 x ⋅ (A cos 2 x + B ⋅ sin 2 x ) 3). x ⋅ A ⋅ e 7 x ⋅ sin 2 x 5). нет правильного ответа Номер: 11.7.А Задача:

Частное решение неоднородного уравнения y ′′ + 36 y = e 6 x ⋅ cos 6 x

ищем в виде 2). A ⋅ e 6 x ⋅ cos 6 x

Ответы: 1). нет правильного ответа

3). x ⋅ e 6 x ⋅ (A ⋅ cos 6 x + B ⋅ sin 6 x ) 4). x 2 ⋅ e 6 x ⋅ (A cos 6 x + B ⋅ sin 6 x )

5). e 6 x (A ⋅ cos 6 x + B ⋅ sin 6 x )

Номер: 11.8.А Задача: Частное решение неоднородного уравнения y ′′ + 36 y = sin 6x ищем в виде Ответы: 1). нет правильного ответа 2). A ⋅ sin 6 x 3). x ⋅ A ⋅ sin 6x 4). A ⋅ cos 6x + B ⋅ sin 6 x 5). x ⋅ (A ⋅ cos 6 x + B ⋅ sin 6 x ) Номер: 11.9.А Задача:

Частное решение неоднородного уравнения 3 y ′′ + y ′ − 2 y = x ⋅ e − x

ищем в виде Ответы: 1). нет правильного ответа 2). e − x (A ⋅ x + B) 4). A ⋅ x ⋅ e − x

3). x ⋅ (A ⋅ x + B) ⋅ e − x

5). A ⋅ x 2 ⋅ e − x Номер: 11.10.А

Задача:

Частное решение неоднородного уравнения 4 y ′′ − 7 y ′ + 3 y = 4 e 3x

ищем в виде Ответы: 1). нет правильного ответа 2). 5). A ⋅ x

2

3x A⋅e 4

3). A ⋅ x

3x ⋅e 4

4). A ⋅ e x

3x ⋅e 4

Номер: 11.11.А Задача:

Частное решение неоднородного уравнения 4 y ′′ − 7 y ′ + 3 y = x ⋅ e x

ищем в виде 112


Ответы: 1). нет правильного ответа 4). x ⋅ (A x + B) ⋅ e x

2). A ⋅ x ⋅ e x

3). (A x + B) ⋅ e x

5). x 2 ⋅ (A x + B) ⋅ e x Номер: 11.12.А

Задача: Частное решение неоднородного уравнения y ′′ − 4 y ′ = x 2 ищем в виде Ответы: 1). A ⋅ x 2

3). A ⋅ x 3

2). A x 2 + B x + C

4). A ⋅ x 2 ⋅ e 4 x

5). A x 3 + B x 2 + C x Номер: 11.13.А Задача:

Частное решение неоднородного уравнения y ′′ − 8 y ′ + 16 y = e −4 x

ищем в виде Ответы: 1). A ⋅ e −4 x 2). A ⋅ x ⋅ e −4 x 3). A ⋅ x 2 ⋅ e −4 x 4). A ⋅ e 4 x 5). A ⋅ x 2 ⋅ e 4 x Номер: 11.14.А Задача: Частное решение неоднородного уравнения y ′′ − 5 y ′ = 5 ⋅ e −5 x ищем в виде Ответы: 1). A ⋅ e −5 x 2). A ⋅ x ⋅ e −5 x 5). нет правильного ответа

Задача:

Частное

3). A ⋅ x 2 ⋅ e −5 x

4). A ⋅ e 5 x

Номер: 11.15.А решение неоднородного

уравнения

y ′′ − 6 y ′ + 25 y = e 3x ⋅ (cos 4 x + x ⋅ sin 4 x ) ищем в виде Ответы: 1). e 3x ⋅ (A ⋅ cos 4 x + B ⋅ sin 4 x ) 3). e 3x ⋅ (A ⋅ cos 4 x + Bx ⋅ sin 4 x )

2). x ⋅ e 3x (A ⋅ cos 4 x + B ⋅ sin 4 x )

4). e 3x ⋅ ((A x + B) cos 4 x + (C x + D ) ⋅ sin 4 x )

5). x ⋅ e 3x ⋅ ((A x + B) cos 4 x + (C x + D ) ⋅ sin 4 x )

Задача:

Частное

Номер: 11.16.А решение неоднородного

уравнения

y ′′ − 4 y ′ + 5 y = e 2 x ⋅ (x ⋅ cos x + sin x ) ищем в виде Ответы: 1). e 2 x ⋅ (A ⋅ x ⋅ cos x + B ⋅ sin x )

2). x ⋅ e 2 x ⋅ ((A x + B) ⋅ cos x + (C x + D ) ⋅ sin x ) 4). e 2 x ⋅ ((A x + B) cos x + (C x + D ) ⋅ sin x )

113

3). x ⋅ e 2 x (A ⋅ cos x + B ⋅ sin x ) 5). нет правильного ответа


Задача:

Частное

Номер: 11.17.А решение неоднородного

уравнения

y ′′ − 4 y ′ + 5 y = e 2 x ⋅ x ⋅ sin x ищем в виде Ответы: 1). e 2 x ⋅ (A ⋅ x ⋅ cos x + B ⋅ sin x )

2). x ⋅ e 2 x ⋅ ((A x + B) ⋅ cos x + (C x + D ) ⋅ sin x ) 4). e 2 x ⋅ ((A x + B) cos x + (C x + D ) ⋅ sin x )

Задача:

Частное

3). x ⋅ e 2 x (A ⋅ cos x + B ⋅ sin x ) 5). нет правильного ответа

Номер: 11.18.А решение неоднородного

y ′′ − 2 y ′ + 2 y = x 2 ⋅ e x ⋅ sin x ищем в виде

(

Ответы: 1). A ⋅ x 2 ⋅ e x ⋅ sin x

((

)

)

2). A x 2 + B x + C ⋅ e x ⋅ sin x

3). e x ⋅ A x 2 + B x + C cos x + (A 1 x + B1 x + C 1 ) ⋅ sin x

((

)

уравнения

(

)

)

4). x ⋅ e x ⋅ A x 2 + B x + C cos x + A 1 x 2 + B1 x + C 1 ⋅ sin x 5). нет правильного ответа

)

Номер: 11.19.А Задача: Частное решение неоднородного уравнения y IV − 2 y ′′ + y = cos x ищем в виде Ответы: 1). A cos x 2). A cos x + B sin x 3). нет правильного ответа 5). (A cos x + B sin x )x 2 4). (A cos x + B sin x )x Номер: 11.20.А Задача: Частное решение неоднородного уравнения y ′′′ + 4 y ′ = sin x ищем в виде Ответы: 1). нет правильного ответа 2). A cos x 3). A cos x + B sin x 5). x ⋅ A cos x 4). (A cos x + B sin x )x Номер: 11.21.А Задача: Частное решение неоднородного уравнения y ′′′ + 9 y ′ = cos 3x ищем в виде 2). A cos 3x 3). A cos 3x + B sin 3x Ответы: 1). x ⋅ (A cos 3x + B sin 3x ) 5). нет правильного ответа 4). x ⋅ A cos 3x Номер: 11.22.В Задача: Найти общее решение дифференциального уравнения y"−2 y'+ y = e 2 x

114


2). y = (c1 + c 2 x )e x + e 2 x

Ответы: 1). y = e 2 x + c1e x + c 2e − x

3). y = (c1 + c 2 x )e − x + xe 2 x

4). y = (c1 + c 2 x )e 2 x + x 2e x

5). y = c1e x + c 2e 2 x + e3x Номер: 11.23.В Задача: Найти общее решение дифференциального уравнения y"+2 y'+ y = e − x

(

)

Ответы: 1). y = c1x + c 2 x 2 + x 3 e − x

(

3). y = e − x c1 + c 2 x + 0.5x 2

(

)

2). y = c1 + c 2 x + 2 x 2 e x

)

⎛ ⎝

4). y = ⎜ c1x + c 2 x 2 +

5). y = (c1 + c 2 x ) + xe x

1 ⎞ − x2 ⎟e x⎠

Номер: 11.24.В Задача: Найти общее решение дифференциального уравнения y"+3y' = 9 x Ответы: 1). y = c1x + c 2e −3x + 3x 2 − 2 x

2). y = c1x + c 2e x + 1.5x 2 + x 4). y = c1 + c 2e − x + x 2 − 1.5x

3). y = c1 + c 2e3x + 3x 2 − x 5). y = c1 + c 2e −3x + 1.5x 2 − x Задача:

Найти

общее

Номер: 11.25.В решение дифференциального

уравнения

2

y"−3y'+2 y = x − 1

x2 3 5 + x+ Ответы: 1). y = c1e + c 2e + 2). y = c1e − x + c 2e −2 x + x 2 + 3x + 5 2 2 4 3 x2 x 1 2x 3x x 2x 2 x + + 3). y = c1e + c 2e + x e + x + 5 4). y = c1e + c 2e + 2 2 4 2 3 1 5 5). y = c1e x + c 2e − 2 x + x 2 + + 2 2 2 x

Задача:

2x

Найти x

(

2

общее

y"−2 y' = e x + x − 3

)

Номер: 11.26.В решение дифференциального

(

)

Ответы: 1). y = c1e x + c 2e 2 x + e − x x 2 + x + 1

(

)

2). y = c1 + c 2e − 2 x + e x 2 x 2 + 2 x + 1

(

)

4). y = c1 + c 2e 2 x + e x − x 2 − x + 1

(

уравнения

3). y = c1 + c 2e x + e 2 x − x 2 − x + 2

(

)

5). y = c1x + c 2 x + e 2 x x 2 + x − 1

115

)


Задача:

Найти

общее

Номер: 11.27.В решение дифференциального

уравнения

2

y"−4 y'+13y = 13x + 5x − 15 Ответы: 1). y = e3x (c1 sin 3x + c 2 cos 3x ) + 3x 2 + 6 x + 18 2). y = e 2 x (c1 sin 3x + c 2 cos 3x ) + x 2 + x − 1

(

3). y = e − 2 x (c1 sin x + c 2 cos x ) + 3 x 2 + 2 x + 5

(

)

4). y = e x (c1 sin 5x + c 2 cos 5x ) + e − x 3x 2 + x + 6 5). y = e 4 x (c1 sin 5x + c 2 cos 5x ) + x 2 +

)

x − 13 2

Номер: 11.28.В Задача: Найти общее решение дифференциального уравнения y"+2 y'+ y = 5

(

)

2). y = (c1 + c 2 x )e 2 x + 5(x + 3)

Ответы: 1). y = c1 + c 2 x 2 e x + 5x

3). y = (c1 + c 2 x )e −5 x + 5x

(

4). y = (c1 + c 2 x )e − x + 5

)

5). y = c1x + c 2 x 2 e 2 x + 5x 2 Номер: 11.29.В Задача: Найти общее решение дифференциального уравнения y"−4 y = 8x 3 Ответы: 1). y = c1e 2 x + c 2e −2 x − 2 x 3 − 3x

2). y = c1e x + c 2e 2 x − 2 x 3 + 3x 4). y = c1e x + c 2e −2 x + 2 x 2 + 5x + 6

3). y = c1e 2 x + c 2e x + 5x 2 − 3 5). y = c1e x + c 2e − x − 2 x 3 − 3x 2

Номер: 11.30.В Задача: Найти общее решение дифференциального уравнения y"−3y'+2 y = e x Ответы: 1). y = c1e −2 x + c 2e x + xe x 3). y = c1e 2 x + c 2e − x − xe − x

2). y = c1e x + c 2 xe x − e x x 4). y = c1e 2 x + c 2e x − xe x

5). y = c1e − x + c 2e x − 2 xe x Задача:

Найти

общее

Номер: 11.31.В решение дифференциального

уравнения

y"+2 y'+2 y = 2 x 3 − 2 Ответы: 1). y = e − x (c1 cos x + c 2 sin x ) + (x − 1)

3

2). y = e x (c1 cos x + c 2 sin x ) + x 3 4). y = e

−2x

x3 (c1 cos x + c 2 sin x ) + 2

3). y = e 2 x (c1 cos x + c 2 sin x ) + (x − 1)

3

5). y = e − x (c1 cos x + c 2 sin x ) + x 3 − 1 116


Задача:

Найти

y"−5y'+6 y = 13 sin 3x

Номер: 11.32.С решение дифференциального

общее

уравнения

Ответы: 1). y = c1e x + c 2e 2 x + 5 cos 3x − sin 3x

5 6

1 6 5 1 3). y = c1e 2 x + c 2e3x + cos 3x − sin 3x 6 6 2x 4x 4). y = c1e + c 2e + 5 cos 3x − 3 sin 3x 5 1 5). y = c1e x + c 2e3x + cos x − sin x 6 6 2). y = c1e − 2 x + c 2e − 3x − cos x − sin x

Задача:

Найти

y"−3y'+2 y = 2e x cos

Номер: 11.33.С решение дифференциального

общее

уравнения

x 2 8 5

⎛ ⎝

Ответы: 1). y = c1e x + c 2e 2 x − e x ⎜ cos

x x⎞ + 2 sin ⎟ 2 2⎠

8 5 8 ⎛ x x⎞ 3). y = c1e x + c 2e − 2 x − e x ⎜ cos − sin ⎟ 5 ⎝ 2 2⎠ x ⎛ ⎞ 4). y = c1e x − c 2e 2 x + 8e − x ⎜ cos + 2 sin x ⎟ 2 ⎝ ⎠ 1 5). y = c1e x − c 2e − x + e x (cos x + 2 sin x ) 2

2). y = c1e − x + c 2e − 2 x − e x (cos x + 2 sin x )

Задача:

Найти

общее

y"+3y'+2 y = sin 2x + 2 cos 2x

Номер: 11.34.С решение дифференциального

уравнения

Ответы: 1). y = c1 cos 2 x + c 2 sin 2 x + 3x + 6e x 2). y = x 2 + e 2 x + c1 cos 2 x + c 2 sin 2 x 4). y = c1e − x + c 2e − 2 x +

1 ⎛π ⎞ sin ⎜ − x ⎟ 2 ⎝8 ⎠

2 ⎛π ⎞ sin ⎜ − 2x ⎟ 4 ⎝8 ⎠ 3 ⎛π ⎞ 5). y = (c1 + c 2 x )e − x + sin ⎜ − x ⎟ 2 ⎝4 ⎠

3). y = c1e − x + c 2e − 2 x +

117


Номер: 11.35.С Задача: Найти общее решение дифференциального уравнения y"+ y = x + 2e x Ответы: 1). y = c1 cos x + c 2 sin x + 3x + 6e x 3). y = c1 cos x + c 2 sin x + x + 2e x

2). y = c1 cos x + c 2 sin x + x + e x 4). y = c1 cos x + c 2 sin x +

x x +e 2

5). y = c1 cos x + c 2 sin x + 2 x + 4e x Задача:

Найти

y"−5 y'+6 y = 2 sin 4 x

общее

Номер: 11.36.С решение дифференциального

уравнения

Ответы: 1). y = c1e −2 x + c 2e −3x + cos 4 x + 2 sin 4 x

2). y = c1e 2 x + c 2e3x + 2(cos 4 x + sin 4 x )

1 (2 cos 4x − sin 4x ) 25 4). y = c1e −2 x + c 2e −3x + 5(2 cos 4 x − sin 4 x )

3). y = c1e 2 x + c 2e3x +

5). y = c1e 2 x + c 2e3x + 2 cos 4 x + sin 4 x Задача:

Найти

y"+6 y'+10 y = 2 sin 2x

общее

Номер: 11.37.С решение дифференциального

Ответы: 1). y = e − 3x (c1 cos x + c 2 sin x ) +

уравнения

1 (sin 2x + 2 cos 2x ) 15

1 (sin 2x − 2 cos 2x ) 15 1 3). y = e − 3x (c1 cos x + c 2 sin x ) − (sin 2 x − 2 cos 2 x ) 15 1 4). y = e − 3x (c1 cos x + c 2 sin x ) + (sin 2 x − 2 cos 2 x ) 15 1 5). y = e3x (c1 cos x + c 2 sin x ) + (sin 2 x − 2 cos 2 x ) 15 2). y = e3x (c1 cos x + c 2 sin x ) −

Задача:

Найти

общее

Номер: 11.38.С решение дифференциального

x

y"−2 y'−8y = e − 8 cos 2 x

118

уравнения


1 9

Ответы: 1). y = c1e 2 x + c 2e − 4 x − e x +

1 (3 cos 2x + sin 2x ) 5

1 1 (3 cos 2x + sin 2x ) 9 5 1 1 3). y = c1e − 2 x + c 2e 4 x − e − x + (3 cos 2 x + sin 2 x ) 9 5 1 1 4). y = c1e 2 x + c 2e − 4 x − e − x + (3 cos 2 x + sin 2 x ) 9 5 1 1 5). y = c1e − 2 x + c 2e 4 x − e x − (3 cos 2 x + sin 2 x ) 9 5 2). y = c1e − 2 x + c 2e 4 x − e x +

Задача:

Найти

общее

Номер: 11.39.С решение дифференциального

уравнения

y"+5 y'+6 y = e − x + e −2 x Ответы: 1). y = 0.5e x + xe 2 x + c1e 2 x + c 2e −3x

e x e −2 x 2). y = + + c1e − 2 x + c 2e − 3x 4 8 −2 x e −x 4). y = 0.25e − + c1e x + c 2e 2 x 8 Задача:

Найти

y"+9 y = 2 x sin x + xe

общее

3). y = 0.5e − x + xe −2 x + c1e −2 x + c 2e −3x 5). y = 0.5e −3x + xe −2 x + c1e x + c 2e3x

Номер: 11.40.С решение дифференциального

1 1 1 x sin x − cos x + (3x − 1)e3x 4 16 54 1 1 1 y = c1 cos 3x + c 2 sin 3x + x sin x − cos x + (3x − 1)e3x 4 16 54 1 1 y = c1 cos 3x + c 2 sin 3x + x sin x + cos x + (3x − 1)e3x 4 54 1 1 1 y = c1 cos 3x + c 2 sin 3x + x sin x − cos x + (3x − 1)e3x 4 10 27 1 1 1 y = c1 cos 3x + c 2 sin 3x + sin x + cos x + (3x − 1)e3x 2 8 27

Ответы: 1). y = c1 cos 3x + c 2 sin 3x + 2). 3). 4). 5).

уравнения

3x

119


Задача:

Найти

Номер: 11.41.С решение дифференциального

общее −x

x

уравнения

y"+2 y'+ y = e + e 1 ⎞ 1 ⎞ 1 ⎛ ⎛ Ответы: 1). y = ⎜ c1 + c 2 x + x 2 ⎟e − x + e x 2). y = ⎜ c1 + c 2 x + x 2 ⎟e − x + e x 2 ⎠ 2 ⎠ 2 ⎝ ⎝ 1 1 ⎞ 1 ⎛ 3). y = c1 + c 2 x + x 2 e − x + e x 4). y = ⎜ c1 + c 2 x + x 2 ⎟e − x + e x 2 ⎠ 4 2 ⎝ 1 5). y = c1 + c 2 x + x 2 e − x + e x 4

(

)

(

)

120


12. Линейные неоднородные дифференциальные уравнения n-ого порядка с постоянными коэффициентами. Метод вариации произвольных постоянных Номер: 12.1.А Задача: Какие из уравнений можно решить только методом вариации произвольных постоянных

ex Ответы: 1). y′′ − y = x 2). y′′ − 7 y′ + 12 y = 3e3x e +1 3). y′′ − 2 y′ + y = −12 cos 2 x − 9 sin 2x 4). y′′ − 7 y′ − 6 y = 0 5). (x + 1) ⋅ y"−2y'+ x + 2 = 0 Номер: 12.2.А Задача: Какие из уравнений можно решить только методом вариации произвольных постоянных

1 cos 2x 4). y"⋅tgx − y'−1 = 0

2). y′′ + 4 y =

Ответы: 1). y′′ − +9 y′ = 10e3x 3). y′′ − 2 y′ − 6 y = (6 x + 1)e3 x

5). y′′ + 4 y = 0

Номер: 12.3.А Задача: Какие из уравнений можно решить только методом вариации произвольных постоянных Ответы: 1). y"⋅x = y'+1 2). y′′ − 7 y′ + 12 y = 3e 4 x 3). y′′ + 6 y = e (cos 4 x − 8 sin 4 x ) x

e2 x 4). y′′ − 4 y′ − 5 y = cos x

5). y′′ − 5 y′ = 0

Номер: 12.4.А Задача: Какие из уравнений можно решить только методом вариации произвольных постоянных Ответы: 1). y′′ − 6 y′ + 25y = 9 sin 4 x − 24 cos 4 x 3). y ' = x ⋅ y"+ y"

4). y′′ − 2 y′ = 6 + 12 x − 24 x 2

sin x cos 2 x 5). y′′ + 5 y = 0

2). y′′′ + y′ =

Номер: 12.5.А Задача: Какие из уравнений можно решить только методом вариации произвольных постоянных

1 2). y′′ − 6 y′ + 34 y = 60 sin 5x sin 3x 3). y′′ − 14 y′ + 53y = 53x 3 − 42 x 2 + 59 x − 14 4). y′′′ + y′ = 0 5). x ⋅ y"⋅ ln x − y' = 0

Ответы: 1). y′′ + 9 y′ =

121


Номер: 12.6.А Задача: Какие из уравнений можно решить только методом вариации произвольных постоянных Ответы: 1). y IV − y′′ = 0 2). x ⋅ y"− y' = x 2 ⋅ e x 3). y′′ − 2 y′ = (4 x + 4 )e 2 x 4). y′′ + 6 y = e x (cos 4 x − 8 sin 4 x )

5). y′′ − 2 y′ + y =

1 xe x

Номер: 12.7.А Задача: Какие из уравнений можно решить только методом вариации произвольных постоянных Ответы: 1). y′′ + y′ + y = 4 x 3 + 24 x 2 + 22 x − 4 2). y′′ − 4 y′ + 20 y = 16 xe 2 x 2

3). x ⋅ y"− y' = x ⋅ cos x

e− x 4). y′′ − 2 y′ + 2 y = cos x

5). y IV − y′′ = 0

Номер: 12.8.А Задача: Какие из уравнений можно решить только методом вариации произвольных постоянных

y' ⎛ y' ⎞ ⋅ ⎜1 + ln ⎟ x ⎝ x⎠ 4). y′′′ + y′′ + 5 y′ + 3y = 0

Ответы: 1). y′′ − 12 y′ + 36 y = 32 cos 2 x + 24 sin 2 x 3). y′′ + 6 y′ + 10 y = 74e3x

2). y" =

ex 5). y′′ − 2 y′ + 2 y = sin 2 x Номер: 12.9.А Задача: Какие из уравнений можно решить только методом вариации произвольных постоянных Ответы: 1. y′′ + 2 y′ + 2 y = e − x ctg x 2). y′′ − 2 y′ + 2 y = 19 sin x + 3 cos x 3). y′′ − 8 y′ + 20 y = 16(sin 2 x − cos 2 x )

4). y′′′ + y′′ = 0

5). y"+

y' = x2 x

Номер: 12.10.А Задача: Какие из уравнений можно решить только методом вариации произвольных постоянных Ответы: 1). y′′ + y′ = 2 x − 1 2). y′′ − 6 y = 0 3). y′′ − 2 y′ + y = 4e x 4). y′′′ − 5 y′′ + 8 y′ − 4 y = 0

ex 5). y′′ − 2 y′ + 2 y = sin x

122


Номер: 12.11.А Задача: Какие из уравнений можно решить только методом вариации произвольных постоянных

ex Ответы: 1). y′′ − 2 y′ + y = 2 2). y′′ − 8 y′ = 72e x x 3). y′′ − 6 y′ + 13y = 34e −3x sin 2 x 4). y′′ + 3y′ + 2 y = 0

5). x ⋅ y"+ y'+ x = 0

Номер: 12.12.А Задача: Какие из уравнений можно решить только методом вариации произвольных постоянных Ответы: 1). y′′′ + 3y′′ + 3y′ + y = 0 2). y′′′ + 10 y′′ + 34 y′ = −9e −5 x 3). y′′ + y = tg x 4). y′′ − 5y′ − 3y = 3 cos x + 19 sin x 5). y' = x ⋅ y"+ y" Номер: 12.13.А Задача: Какие из уравнений можно решить только методом вариации произвольных постоянных Ответы: 1). y′′′ − 2 y′ − 8y = 12 sin 72 x − 38 cos 2x 2). y′′ − 7 y′ − 6 y = 0 3). y′′ + 4 y = ctg 2 x

2 4). y ⋅ y"+ (y ') + y ⋅ ln y = 0 2

5). y′′ + 6 y′ = 0

Номер: 12.14.А Задача: Какие из уравнений можно решить только методом вариации произвольных постоянных Ответы: 1). y′′ + y = ctg x 2). y"−4 y ⋅ y' = 0 3). y′′ + 25 y = e x (cos 5x − 10 sin 5x )

4). y′′ + y = 2 cos x − sin x 5). y′′ − 6 y = 0

Номер: 12.15.А Задача: Какие из уравнений можно решить только методом вариации произвольных постоянных Ответы: 1). y′′ + 6 y′ + 10 y = 74e3x 2). y′′ + 2 y′ + 5 y = −8e − x sin 2 x 3). y′′ − 13y′ + 12 y = 0

4). y′′ + 5 y′ + 6 = 0

ex 5). y′′ − 2 y′ + y = x

Номер: 12.16.А Задача: Какие из уравнений можно решить только методом вариации произвольных постоянных

e− x x 3). y′′ + 10 y′ + 28 y = e 4 x

Ответы: 1). y′′ + 2 y′ + y =

2). y IV − 5 y′ + 4 y = 0 4). y′′ + 36 y = 36

123

5). 3y'⋅y" = 2 y


Номер: 12.17.А Задача: Какие из уравнений можно решить только методом вариации произвольных постоянных Ответы: 1). y′′ + y = −4 cos x − 2 sin x 2). y′′ + 6 y′ + 9 y = 72e3x 3). y IV − 10 y′′ + 9 y = 0

4). y′′ + y =

1 2 4 5). 2 y ⋅ y"+ (y') + (y') = 0 cos x

Номер: 12.18.А Задача: Какие из уравнений можно решить только методом вариации произвольных постоянных

1 2 2). y"⋅tgy = 2(y') sin x 3). y′′ − 2 y′ + 5 y = 5x 2 − 5x + 12 4). y′′ − 2 y′ + 5 y = 10e − x cos 2 x 5). y′′ − 4 y′ = 0

Ответы: 1). y′′ + y =

Номер: 12.19.А Задача: Какие из уравнений можно решить только методом вариации произвольных постоянных Ответы: 1). y′′ + 6 y′ + 13y = −75 sin 2 x 2). y′′ + 16 y = 80e 2 x 3). y′′′ − 3y′′ + 3y′ − y = 0

4). y′′ + 4 y =

1 sin 2 x

5). (1 + y ) ⋅ y"−5(y') = 0 2

Номер: 12.20.А Задача: Какие из уравнений можно решить только методом вариации произвольных постоянных Ответы: 1). y′′′ − y′′ + 4 y′ − 4 y = 0 2). y′′ − 2 y′ + 37 y = 36e 4 x cos 6 x 3). y′′ + 4 y =

1 sin 2 x

4). y′′ + y = −4 cos x − 2 sin x

5). 2(y') = y" (y − 1) 2

Номер: 12.21.А Задача: Какие из уравнений можно решить только методом вариации произвольных постоянных

e −2 x Ответы: 1). y′′ + 4 y′ + 4 y = 3 x 3). y′′ − 8 y = 16 − 48x 2 − 128x 3

2). y′′ + 4 y′ = 15e x 4). y′′ + 2 y′ + y = 8e x

5). y′′ + 2 y′ + y = 0

Номер: 12.22.А Задача: Какие из уравнений можно решить только методом вариации произвольных постоянных

124


Ответы: 1). y′′ − 4 y′ + 5 y = (24 sin x + 8 cos x )e −2 x

e2 x 3). y′′ − 4 y′ + 4 y = 3 x

4). y"+ (y ') = 0 3

2). y′′ − 4 y = (− 24 x − 10 )e 2 x 5). y′′ + y′ − 2 y = 0

Номер: 12.23.А Задача: Какие из уравнений можно решить только методом вариации произвольных постоянных Ответы: 1). y′′ + 2 y′ + y = 3e − x x + 1 2). y′′ − 16 y = 0 3). y′′ + 3y′ = (40 x + 58x )e x

(

)

4). y′′ + 16 y = 8 cos 4 x 5). y" 9 + y 2 − 2 y(y') = 0 2

Номер: 12.24.А Задача: Какие из уравнений можно решить только методом вариации произвольных постоянных Ответы: 1. y′′ + 9 y = 9 x 4 + 12 x 2 − 27 2). y IV + y′′′ − 4 y′ − 4 = 0 3). y′′ + y = −ctg 2 x

4). y′′ + y′ − 2 y = 9 cos x − 7 sin x

5). y"+ y'⋅ 1 − (y') = 0 2

Номер: 12.25.А Задача: Какие из уравнений можно решить только методом вариации произвольных постоянных Ответы: 1). y′′′ + 2 y′′′ + 9 y + 18 y = 0 2). y′′ − 8 y′ = 18x + 60 x 2 − 32 x 2

( )

3). y′′ − y′ = e 2 x cos e x

4). y′′ + 8 y′ = 72e 2 x

5). y′′ + y′ − 2 y = 0

Номер: 12.26.А Задача: Какие из уравнений можно решить только методом вариации произвольных постоянных Ответы: 1). y′′ − y′ − 2 y = 0 2). y′′ + 4 y′ = e x (2 sin 2 x + 24 cos 2 x ) 3). y V − 6 y IV + 9 y′′′ = 0

4). y′′ + 3y′ = e x

( )

5). y′′ − y′ = e 2 x sin e x

Номер: 12.27.А Задача: Какие из уравнений можно решить только методом вариации произвольных постоянных Ответы: 1). y′′ + 2 y′ + y = 6e − x 2). 4 y′′ − 4 y′ + y = −25 cos x 3). y′′ + 2 y′ + y = 0

4). y′′ + y = tg 2 x

5). y′′ + y′ − 2 y = 0

Номер: 12.28.А Задача: Какие из уравнений можно решить только методом вариации произвольных постоянных Ответы: 1). y′′ + y =

2 sin 2 x

2). y IV + 10 y′ + 9 y = 0 125

3). y′′ + 4 y′ = 32e −4 x


5). y′′ − 4 y′ + 4 y = 0

4). y′′ + 2 y′ + 37 y = 37 x 2 − 33x + 74

Номер: 12.29.А Задача: Какие из уравнений можно решить только методом вариации произвольных постоянных Ответы: 1). 6 y′′ − y′ − y = 3e 2 x 2). y′′ + 4 y′ + 29 y = 26e − x 3). y′′ + 5y′ + 6 y = 0

e− x 4). y′′ + 2 y′ + 5 y = sin 2x

5). y′′′ =

1

(x − 3)3

Номер: 12.30.А Задача: Какие из уравнений можно решить только методом вариации произвольных постоянных

1 cos 3x 4). 2 y′′ + 7 y′ + 3y = 222 sin 3x 5). y′′ + 3y = 0 2). y′′ + 9 y =

Ответы: 1). y IV + 10 y′′ + 9 y = 0

3). y′′ − 4 y = 8e 2 x

Номер: 12.31.А Задача: Решите дифференциальное уравнение II порядка y"+ y = Ответы: 1). нет правильного ответа

2). y = c1 ⋅ e x + c 2 ⋅ e − x +

1 2 ⋅ sin x 1 4). y = c1 ⋅ cos x + c 2 ⋅ sin x − ctgx ⋅ cos x + 2 ⋅ sin x cos 2x 5). y = c1 ⋅ cos x + c 2 ⋅ sin x + 2 ⋅ sin x

1 sin 3 x

cos 2 x 2 ⋅ sin x

3). y = c1 ⋅ cos x + c 2 ⋅ sin x − ctgx ⋅ cos x −

Номер: 12.32.В Задача: Решите дифференциальное уравнение II порядка y"+ y = tgx Ответы: 1). y = c1 cos x + c 2 sin x + cos x ⋅ ln 2). y = c1 cos x + c 2 sin x + cos x ⋅ ln

sin x − 1 sin x + 1

sin x − 1 sin x + 1 4). y = c1 cos x + c 2 sin x + sin x ⋅ cos x 3). y = c1 cos x + c 2 sin x + sin x ⋅ ln

5). нет правильного ответа 126

sin x + 1 sin x − 1


Номер: 12.33.В Задача: Решите дифференциальное уравнение II порядка y"− y' = e 2 x ⋅ cos e x

( )

( )

( )

Ответы: 1). y = c1 + c 2 ⋅ e x − 2e x ⋅ sin e x − cos e x

( )

2). y = c1 + c 2 ⋅ e x + cos e x

( )

3). y = c1 + c 2 ⋅ e x − cos e x

( )

( )

4). y = c1 + c 2 ⋅ e x + 2e x ⋅ sin e x + cos e x

5). нет правильного ответа

Номер: 12.34.В Задача: Решите дифференциальное уравнение II порядка y"− y' = e 2 x ⋅ sin e x

( )

Ответы: 1). y = c1 + c 2 ⋅ e x + sin e x

( )

( )

2). y = c1 + c 2 ⋅ e x − sin e x

( ) ( ) ⋅ cos(e ) + sin (e )

3). y = c1 + c 2 ⋅ e x + 2e x ⋅ cos e x − sin e x 4). y = c1 + c 2 ⋅ e x − 2e x 5). нет правильного ответа

x

x

Номер: 12.35.В

ex Задача: Решите дифференциальное уравнение II порядка y"− y' = x e +1 Ответы: 1). y = c1 + c 2 ⋅ e x − x ⋅ e x − 1 + e x ⋅ ln e x + 1 2). y = c1 + c 2 ⋅ e x + x ⋅ e x 3). y = c1 + c 2 ⋅ e x + x ⋅ e x

( ) ( ) − ln (e + 1) − e ⋅ ln (e + 1) + (1 + e ) ⋅ ln (e + 1) + (1 − e ) ⋅ ln (e + 1) x

4). y = c1 + c 2 ⋅ e x + x ⋅ e x 5). нет правильного ответа

x

x

x

x

x

x

Номер: 12.36.В

ex Задача: Решите дифференциальное уравнение II порядка y"− y = 2 x e +1 1 1 Ответы: 1). y = c1e x + c 2 e − x + e x x − ln e 2 x + 1 + e − x ⋅ ln e 2 x + 1 2 4 1 ⎛1 ⎞ 4 2). y = c1e x + c 2e − x + e x ⎜ x − e 2 x + 1 − e − x ⋅ ln e 2 x + 1 ⎟ 4 ⎝2 ⎠ 1 ⎛ ⎞ 3). y = c1e x + c 2 e − x + e x ⎜ x − ln e 2 x + 1 ⎟ + e − x ⋅ ln 4 e 2 x + 1 2 ⎝ ⎠ 1 1 4). y = c1e x + c 2 e − x + e x x + ln e 2 x + 1 + e − x ⋅ ln e 2 x + 1 2 4

(

(

)

(

)

)

(

5). нет правильного ответа

127

(

)

)


Номер: 12.37.В Задача: Решите дифференциальное уравнение II порядка y"+ y = Ответы: 1). нет правильного ответа

1 − sin 2 x cos x

x + sin x + sin x sin x + cos x 2 x 3). y = c1 cos x + c 2 sin x + cos x ⋅ cos x + ln tg − sin x + sin x sin x − cos x 2

2). y = c1 cos x + c 2 sin x + cos x ⋅ cos x + ln tg

⎛ x π⎞ + ⎟ − sin x + sin x sin x − cos x ⎝ 2 4⎠

4). y = c1 cos x + c 2 sin x + cos x ⋅ cos x + ln tg⎜

⎛ x π⎞ + ⎟ − sin x + sin x sin x + cos x ⎝ 2 4⎠

5). y = c1 cos x + c 2 sin x + cos x ⋅ cos x + ln tg⎜

Номер: 12.38.В

ex Задание: Решите дифференциальное уравнение II порядка y"−2 y'+ y = x x Ответы: 1). нет правильного ответа 2). y = e (c1 + c 2 x + x − ln x ⋅ x ) 3). y = e − x (c1 + c 2 x − x + x ⋅ ln x ) 5). y = e − x (c1 + c 2 x − x + x ⋅ ln x )

4). y = e x (c1 + c 2 x − x + ln x ⋅ x )

Номер: 12.39.В Задача: Решите дифференциальное уравнение II порядка y"+4 y =

Ответы: 1). нет правильного ответа 2). y = (c1 − 2 x ) ⋅ cos 2 x + (c 2 − ln sin 2 x ) sin 2 x

4 sin 2 x

1 ⎞ ⎛ ln sin 2 x ⎟ sin 2 x 2 ⎝ ⎠ 4). y = (c1 − 2 x ) ⋅ cos 2 x + (c 2 + ln sin 2 x ) sin 2 x 5). y = (c1 − 2 x ) ⋅ sin 2 x + (c 2 + ln cos 2 x ) cos 2 x 3). y = (c1 − 2 x ) ⋅ cos 2 x + ⎜ c 2 −

Номер: 12.40.В

ex Задача: Решите дифференциальное уравнение II порядка y"− y = x e −1

128


⎛ 1 x⎞ e x − 1 ⎞⎟ − x ⎛ x ⎜ Ответы: 1). y = e ⋅ c1 − ln e c ln e 1 e ⎟ + − − + ⎜ 2 x ⎟ ⎜ 2 e ⎠ ⎝ ⎠ ⎝ ⎛ 1 ⎞ e x ⎞⎟ − x ⎛ x ⎜ 2). y = e ⋅ c1 − ln + e ⎜ c 2 + ln e x − 1 − e x ⎟ x ⎜ 2 ⎠ e − 1 ⎟⎠ ⎝ ⎝ ⎛ 1 x e x − 1 ⎞⎟ − x ⎛ ⎞ x ⎜ x e c e ln e 1 3). y = e ⋅ c1 + ln + − + − ⎜ ⎟ 2 x ⎟ ⎜ 2 e ⎝ ⎠ ⎝ ⎠ ⎛ 1 ⎞ e x − 1 ⎞⎟ − x ⎛ x ⎜ 4). y = e ⋅ c1 + ln + e ⎜ c 2 + ln e x − 1 + e x ⎟ x ⎜ 2 ⎠ e ⎟⎠ ⎝ ⎝ x

(

)

5). нет правильного ответа Задача:

Решите

y′′ − y′ + e

2x

( )= 0

⋅ cos e

Номер: 12.41.В дифференциальное уравнение

II

порядка

x

( ) 2). y = c + c ⋅ e + cos(e ) − e ⋅ sin (e ) + cos(e ) + 2e ⋅ sin (e ) 4). y = c + c ⋅ e + cos(e )

Ответы: 1). y = c1 + c 2 ⋅ e x − cos e x

x 3). y = c1 + c 2 ⋅ e x 5). нет правильного ответа

x

1 x

2

x

x

x

x

x

1

x

2

Номер: 12.42.В Задача: Решите дифференциальное уравнение II порядка y′′ − y′ +

(

e2x =0 ex + 1

) ( ) )⋅ ln(1 + e ) 3). y = c + c ⋅ e + (1 − e )⋅ ln(1 + e ) )⋅ ln(1 + e ) 5). нет правильного ответа

Ответы: 1). y = c1 + c 2 ⋅ e x + 1 + e x ⋅ ln 1 + e x

( − (1 − e

2). y = c1 + c 2 ⋅ e x − 1 + e x 4). y = c1 + c 2 ⋅ e x

x

x

1

2

x

x

x

x

Номер: 12.43.В

e2x Задача: Решите дифференциальное уравнение II порядка y′′ − y′ + 2 x =0 e −1 ex + 1 x 2x x Ответы: 1). y = c1 + c 2 ⋅ e + ln e − 1 + e ⋅ ln ex − 1 x

2). y = c1 + c 2 ⋅ e + ln e

2x

x

− 1 + e ⋅ ln

ex + 1 ex − 1

129


x

3). y = c1 + c 2 ⋅ e + ln x

4). y = c1 + c 2 ⋅ e + ln

e

2x

e

2x

x

− 1 + e ⋅ ln

ex + 1 ex − 1

x

− 1 + e ⋅ ln

ex − 1 ex + 1

5). нет правильного ответа Номер: 12.44.В

ex Задача: Решите дифференциальное уравнение II порядка y′′ − 2 y′ + y = x ⎛ ⎛ 8x x ⎞ x x⎞ ⎟⎟ 2). y = e x ⋅ ⎜⎜ c1 + c 2 ⋅ x + ⎟⎟ Ответы: 1). y = e x ⋅ ⎜⎜ c1 + c 2 ⋅ x + 3 2 ⎝ ⎠ ⎝ ⎠ ⎛ ⎛ 7x x ⎞ 4x x ⎞ ⎟⎟ 4). y = e x ⋅ ⎜⎜ c1 + c 2 ⋅ x + ⎟⎟ 3). y = e x ⋅ ⎜⎜ c1 + c 2 ⋅ x + 2 3 ⎝ ⎝ ⎠ ⎠ 5). нет правильного ответа Номер: 12.45.В Задача: Решите дифференциальное уравнение II порядка y′′ − 2 y′ + y =

ex x2 +1

Ответы: 1). y = e x ⋅ ⎛⎜ c1 + c 2 ⋅ x − x 2 + 1 + x ⋅ ln x + x 2 + 1 ⎞⎟

2). y = e x ⋅ ⎛⎜ c1 + c 2 ⋅ x + x 2 + 1 + x ⋅ ln x + x 2 + 1 ⎞⎟

( ⋅ (c + c ⋅ x + 2

) + 1 + x ⋅ arctgx )

3). y = e x ⋅ c1 + c 2 ⋅ x − 2 x 2 + 1 + x ⋅ arctgx

x2 4). y = e x 1 2 5). нет правильного ответа

Номер: 12.46.В

x ⋅ ex Задача: Решите дифференциальное уравнение II порядка y′′ − 2 y′ + y = − 2 x +1 Ответы: 1). y = e x ⋅ c1 − arctgx + x ⋅ c 2 − ln x 2 + 1

(

(

( ( ( ))) ⋅ (c − arctgx + x ⋅ (c − ln (x + 1))) ⋅ (c + arctgx + x ⋅ (c − ln (x + 1)))

2). y = e x ⋅ c1 − arctgx + x ⋅ c 2 − ln x 2 + 1 3). y = e x

1

2

4). y = e x 1 5) нет правильного ответа

2

2

2

130

))


Номер: 12.47.В Задача: Решите дифференциальное уравнение II порядка y"+ y = Ответы: 1). y = cos x ⋅ (c1 + x + ln cos x ) + sin x ⋅ (c 2 + x + ln cos x )

cos x + sin x cos 2 x

2). y = cos x ⋅ (c1 + x + ln cos x ) + sin x ⋅ (c 2 + x − ln cos x ) 3). y = cos x ⋅ (c1 + x − ln cos x ) + sin x ⋅ (c 2 + x + ln cos x ) 4). y = cos x ⋅ (c1 + x − ln cos x ) + sin x ⋅ (c 2 + x − ln cos x ) 5). нет правильного ответа Задача:

Номер: 12.48.В дифференциальное уравнение

Решите

y"+ y = 16 ⋅ sin 2 x ⋅ cos x

(

II

порядка

)

Ответы: 1). y = cos x ⋅ c1 + 4 sin 4 x + sin x ⋅ (c 2 + 2 x − sin 4 x )

( ) 3). y = cos x ⋅ (c − 4 sin x ) + sin x ⋅ (c 4). y = cos x ⋅ (c − 4 sin x ) + sin x ⋅ (c

2). y = cos x ⋅ c1 + 4 sin 4 x + sin x ⋅ (c 2 + 2 x + 0.5 sin 4 x ) 4

1

4

1

2 2

+ 2x − 0.5 sin 4 x )

− 2 x + 0.5 sin 4 x )

5). нет правильного ответа Номер: 12.49.В Задача: Решите дифференциальное уравнение II порядка y"−2 y'+ y = e x ⋅ x

⎛ ⎝

Ответы: 1). y = e x ⋅ ⎜ c1 + c 2 x +

(

16 2 ⎞ ⋅x ⋅ x⎟ 15 ⎠

3). y = e x ⋅ c1 + c 2 x + 4 x 2 ⋅ x

)

(

2). y = e x ⋅ c1 + c 2 x − x 2 ⋅ x

⎛ ⎝

4). y = e x ⋅ ⎜ c1 + c 2 x +

)

4 2 ⎞ ⋅x ⋅ x⎟ 15 ⎠

5). нет правильного ответа Номер: 12.50.В

− ex Задача: Решите дифференциальное уравнение II порядка y"−2 y'+ y = x +1 x Ответы: 1). y = e ⋅ (c1 + c 2 x − (1 + x ) ⋅ ln 1 + x ) 2). y = e x ⋅ (c1 + c 2 x − (1 − x ) ⋅ ln 1 + x )

4). y = e x ⋅ (c1 + c 2 x + (1 − x ) ⋅ ln 1 + x )

3). y = e x ⋅ (c1 + c 2 x + (1 + x ) ⋅ ln 1 + x ) 5). нет правильного ответа

Номер: 12.51.В

ex Задача: Решите дифференциальное уравнение II порядка y"−2 y'+ y = 2 x +1 131


(

(

)

Ответы: 1). y = e x ⋅ c1 + c 2 x − ln x 2 + 1 + x ⋅ arctgx

(

(

)

2). y = e x ⋅ c1 + c 2 x + ln x 2 + 1 + x ⋅ arctgx

( ⋅ (c + c x + ln

)

)

) + 1 + x ⋅ arctgx )

3). y = e x ⋅ c1 + c 2 x − ln x 2 + 1 + x ⋅ arctgx 4). y = e x 1 x2 2 5). нет правильного ответа

Номер: 12.52.B Задача: Решите дифференциальное уравнение II порядка y"− y' = e 2 x ⋅ e x + 1 2 2 3 2 3 x 2 5 x 2 x x Ответы: 1). y = c1 + c 2 ⋅ e + ⋅ e + 1 − ⋅ e + 1 + e ⋅ e + 1 3 5 3 3 5 2 2 2 2 2). y = c1 + c 2 ⋅ e x + ⋅ e x + 1 − ⋅ e x + 1 + e x ⋅ 3 e x + 1 3 5 3 3 5 3 2 2 2 3). y = c1 + c 2 ⋅ e x + ⋅ e x + 1 − ⋅ e x + 1 + e x ⋅ e x + 1 3 5 3 3 5 2 2 2 2 4). y = c1 + c 2 ⋅ e x − ⋅ e x + 1 + ⋅ e x + 1 + ⋅ 3 e x + 1 3 5 3 x

5). нет правильного ответа

Номер: 12.53.В Задача: Решите дифференциальное уравнение II порядка y"+ y + Ответы: 1). y = c1 cos x + c 2 sin x + cos x ⋅ ln tg

1 =0 sin 2 x

x +1 2

x + sin x 2 3). y = c1 cos x + c 2 sin x + cos x ⋅ ln tgx + 1 x 4). y = c1 cos x + c 2 sin x + cos x ⋅ ln tg − 1 2 2). y = c1 cos x + c 2 sin x + cos x ⋅ ln tg

5). нет правильного ответа Номер: 12.54.В Задача: Решите дифференциальное уравнение II порядка y"+ y = Ответы: 1). y = c1 cos x + c 2 sin x − cos x ⋅ ln cos x + tgx ⋅ sin x

1 cos x

2). y = c1 cos x + c 2 sin x − cos x ⋅ ln cos x − tgx ⋅ sin x 3). y = c1 cos x + c 2 sin x + cos x ⋅ ln cos x + tgx ⋅ sin x 4). y = c1 cos x + c 2 sin x + cos x ⋅ ln cos x − tgx ⋅ sin x 5). нет правильного ответа 132


Номер: 12.55.С Задача: Решите дифференциальное уравнение II порядка y"−3y'+2 y =

e2x ex + 1

⎛ ⎛ e x + 1 − 1 ⎞⎟ ⎞⎟ x x ⎜ ⎜ Ответы: 1). y = e ⋅ c1 − 2 ⋅ e + 1 + e ⋅ c 2 + ln ⎜ ⎜ e x + 1 + 1 ⎟⎠ ⎟⎠ ⎝ ⎝ ⎛ ⎛ e x + 1 − 1 ⎞⎟ ⎞⎟ x ⎜ x x ⎜ 2). y = e ⋅ c1 − 2 ⋅ e + 1 + e ⋅ c 2 − ln ⎜ ⎜ e x + 1 + 1 ⎟⎠ ⎟⎠ ⎝ ⎝ x

x

3). y = c1 ⋅ e + c 2 ⋅ e x

4). y = c1 ⋅ e + c 2 ⋅ e

−2x

2x

x

x

− 2 ⋅ e + 1 ⋅ e + ln x

ex + 1 −1 ex + 1 −1

ex + 1 −1

x

+ 2 ⋅ e + 1 ⋅ e + ln

ex + 1 −1

⋅ e− 2x

⋅ e2x

5). нет правильного ответа Номер: 12.56.С Задача: Решите дифференциальное уравнение II порядка: y"− y' = x

x

x

Ответы: 1). y = c1 + c 2 ⋅ e − 2 e + 1 + e ⋅ ln x

2). y = c1 ⋅ e + c 2 ⋅ e 3). y = c1 + c 2 ⋅ e

−x

−x

x

x

− 2 e +1⋅e + e x

+ 2 e +1 + e

x

4). y = c1 ⋅ e + c 2 ⋅ e

−x

x

−x

x

⋅ ln

+ 2e ⋅ e + 1 + e

−x

⋅ ln

ex ex + 1

ex + 1 − 1 ex + 1 + 1 ex + 1 − 1 ex + 1 + 1

ex + 1 − 1 ex + 1 + 1 −x

⋅ ln

ex + 1 + 1 ex + 1 − 1

5). нет правильного ответа Задача:

Решите

Номер: 12.57.С дифференциальное уравнение

x

y"−2 y'+ y = e ⋅ arctgx

133

II

порядка


⎛ ⎝

(

1 ⎛ ⋅ 2 ⎝ 1 ⎛ 3). y = e x ⋅ ⎜ c1 + c 2 x + ⋅ 2 ⎝

2). y = e x ⋅ ⎜ c1 + c 2 x +

(

)

1 ⎞ ⋅ x 2 ⋅ arctgx + arctgx − x + x 2 ⋅ arctgx − ln x 2 + 1 ⎟ 2 ⎠ ⎞ x 2 ⋅ arctgx + arctgx − x + x 2 ⋅ arctgx − x ⋅ ln x 2 + 1 ⎟ ⎠ ⎞ x 2 ⋅ arctgx + arctgx − x + x arctgx − ln x 2 + 1 ⎟ ⎠

Ответы: 1). y = e x ⋅ ⎜ c1 + c 2 x +

(

)

(

)

(

(

4). y = e x ⋅ c1 + c 2 x + x 2 ⋅ arctgx − arctgx − x + x arctgx − ln x 2 + 1 5). нет правильного ответа

))

)

Номер: 12.58.С Задача: Решите дифференциальное уравнение II порядка y"− y =

(

)

(

3). y = c1e x + c 2 e − x − e x 4). y = c1e x + c 2 e − x + e x

(

5). y = e x c1 + arctg e x

ex − 1

( ) 1 − 1) + c ⋅ e − e − 1 ⋅ (1 + 4e ) 3 ⋅ arctg e − 1 + e − 1 ⋅ (2 + e ) ⋅ (3 ⋅ e ) 1 ⋅ arctg e − 1 − e ⋅ e − 1 ⋅ (4 + e ) 3 1 ⎛ ⎞ e − 1 ⋅ (2 + e )⎟ − 1) + e ⎜ c + 3

Ответы: 1). y = e x c1 + arctg e x − 1 + c 2 ⋅ e −x − 2). y = e x c1 + arctg e x

ex

−x

2

x

x

−x

−x

x

x

−x

1 x e − 1 ⋅ 2e −x + 1 3

x

x

2

−x

x

x

x

Номер: 12.59.С Задача: Решите дифференциальное уравнение II порядка y"+ y + ctg 2 x = 0 Ответы: 1). y = c1 cos x + c 2 sin x + cos x ⋅ ln tg

x 2 x 3). y = c1 cos x + c 2 sin x − cos x ⋅ ln tg − 2 2 x 4). y = c1 cos x + c 2 sin x + sin x ⋅ ln tg + 2 2 2). y = c1 cos x + c 2 sin x + cos x ⋅ ln tg

5). нет правильного ответа

134

x +2 2


Номер: 12.60.С

e4x Задача: Решите дифференциальное уравнение II порядка y'−4 y'+3y = x e −1 Ответы: 1). y = t ⋅ c1 + c 2 ⋅ t 2 − 0,5 ⋅ t + ln t − 1 ⋅ t 2 − 1 , где t = e x

( 3). y = t ⋅ (c 4). y = t ⋅ (c

(

(

))

(

))

2). y = t ⋅ c1 + c 2 ⋅ t 2 − 0,5 ⋅ t + ln t − 1 ⋅ t 2 − 1 , где t = e x

(

))

1

+ c 2 ⋅ t 2 − t + ln t − 1 ⋅ t 2 − 1 , где t = e x

1

+ c 2 ⋅ t 2 − t + ln

(

))

t − 1 ⋅ t 2 − 1 , где t = e x

5). нет правильного ответа Номер: 12.61.С Задача: Решите дифференциальное уравнение II порядка y'+ y + Ответы: 1). y = cos x ⋅ (c1 + ln sin x ) + sin x ⋅ (c 2 − x )

cos x =0 sin 2 x

2). y = cos x ⋅ (c1 − ln sin x ) + sin x ⋅ (c 2 − x )

3). y = cos x ⋅ (c1 + ln sin x ) + sin x ⋅ (c 2 + x ) 4). y = cos x ⋅ (c1 − ln sin x ) + sin x ⋅ (c 2 + x ) 5). нет правильного ответа

Номер: 12.62.С

ex Задача: Решите дифференциальное уравнение II порядка y'−2 y + y = − cos 2 x Ответы: 1). y = e x ⋅ (c1 + c 2 ⋅ x + x ⋅ tgx + ln cos x )

2). y = e x ⋅ (c1 + c 2 ⋅ x + ln cos x )

4). y = e x ⋅ (c1 + c 2 ⋅ x + x ⋅ tgx − ln cos x )

135

3). y = e x ⋅ (c1 + c 2 ⋅ x − ln cos x ) 5). нет правильного ответа


13. Системы обыкновенных дифференциальных уравнений Задача:

Решите

систему

Номер: 13.1.В линейных дифференциальных

уравнений:

⎧x ' = 2 x + y , считая x,y функциями аргумента t ⎨ y ' = 3 x + 4 y ⎩ Ответы: 1). x = c1e 5 t + c 2 e t , y = 3c1e 5 t − c 2 e t 2). нет правильного ответа 3). x = c1e 5 t + c 2 e t , y = −c1e 5 t − c 2 e t 4). x = c1e −5 t + c 2 e − t , y = −7c1e −5 t − 3c 2 e − t 5). x = c1e −5 t + c 2 e − t , y = −c1e −5 t − c 2 e − t Номер: 13.2.В

⎧ x ' = 3x + y , ⎩ y' = x + 3y

Задача: Решите систему линейных дифференциальных уравнений: ⎨ считая x,y функциями аргумента t Ответы: 1). x = c1e 4 t + c 2 e 2 t , y = c1e 4 t − c 2 e 2 t 2). x = c1e 4 t + c 2 e 2 t , y = −2c1e 4 t − 2c 2 e 2 t 3). x = c1e −4 t + c 2 e −2 t , y = −7c1e −4 t − 5c 2 e −2 t 4). x = c1e −4 t + c 2 e −2 t , y = −2c1e −4 t − 2c 2 e −2 t 5). нет правильного ответа Номер: 13.3.В

⎧ x ' = 3x + y , ⎩ y' = y

Задача: Решите систему линейных дифференциальных уравнений: ⎨ считая x,y функциями аргумента t Ответы: 1). x = c1e − t + c 2 e −3t , y = −4c1e − t − 6c 2 e −3t 2). x = c1e t + c 2 e 3t , y = −2c1e t

3). x = c1e t + c 2 e −3t , y = −2c1e t − 6c 2 e −3t

4). x = c1e t + c 2 e 3t , y = −2c1e t − 2c 2 e 3 t Задача:

Решите

систему

5). нет правильного ответа

Номер: 13.4.В линейных дифференциальных

уравнений:

⎧x ' = x − y , считая x,y функциями аргумента t ⎨ y ' = − 4 x + y ⎩ Ответы: 1). нет правильного ответа 2). x = c1e 3t + c 2 e − t , y = 2c 2 e − t 3). x = c1e 3t + c 2 e − t , y = −2c1e 3 t + 2c 2 e − t 5). x = c1e 3t + c 2 e t , y = −2c1e 3t

136

4). x = c1e −3t + c 2 e t , y = 4c1e −3t


Задача:

Решите

систему

Номер: 13.5.В линейных дифференциальных

уравнений:

⎧x ' = x − y , считая x,y функциями аргумента t ⎨ y ' = − 4 x + 4 y ⎩ Ответы: 1). x = c1 + c 2 e 5 t , y = c1 2). x = c1 + c 2 e 5 t , y = c1 + 6c 2 e 5 t 3). x = c1 + c 2 e 5 t , y = c1 − 4c 2 e 5 t 5). нет правильного ответа Задача:

Решите

систему

4). x = c1e 5 t , y = −4c1e 5 t Номер: 13.6.В линейных дифференциальных

уравнений:

⎧x ' = − x − 2 y , считая x,y функциями аргумента t ⎨ y ' = 3 x + 4 y ⎩ Ответы: 1). нет правильного ответа 2). x = c1e t + c 2 e −2 t , y = −c1e t + 1,5c 2 e −2 t 3). x = c1e − t + c 2 e 2 t , y = −c1e − t − 1,5e 2 t 4). x = c1e t + c 2 e 2 t , y = −c1e t − 1,5c 2 e 2 t 5). x = c1e − t + c 2 e −2 t , y = c1e − t + 1,5ce −2 t Задача:

Решите

систему

Номер: 13.7.В линейных дифференциальных

уравнений:

⎧ x ' = 2 x + 3y , считая x,y функциями аргумента t ⎨ y ' = 5 x + 4 y ⎩ Ответы: 1). x = c1e −7 t + c 2 e − t , y = −3c1e −7 t − c 2 e − t 3). x = c1e 7 t + c 2 e − t , y =

2). нет правильного ответа 4). x = c1e 7 t + c 2 e t , y =

Задача:

Решите

5 7t 1 c1e − c 2 e t 3 3

систему

5 7t c1e − c 2 e − t 3

5). x = c1e 7 t + c 2 e − t , y = 5c1e 7 t − c 2 e − t

Номер: 13.8.В линейных дифференциальных

⎧ x ' = 3x − 2 y , считая x,y функциями аргумента t ⎨ y ' = 2 x − y ⎩ Ответы: 1). нет правильного ответа 2). x = e t (c1 + c 2 t ), y = e t (c1 − c 2 + c 2 t )

3). x = e − t (c1 + c 2 t ), y = e − t (c1 + c 2 − c 2 t )

4). x = e − t (c1 + c 2 t ), y = e − t (2c1 + 0,5c 2 + 0,5c 2 t )

5). x = e t (c1 + c 2 t ), y = e t (c1 − 0,5c 2 + c 2 t ) 137

уравнений:


Задача:

Решите

систему

Номер: 13.9.В линейных дифференциальных

уравнений:

⎧ x ' = 5x + 4 y , считая x, y функциями аргумента t ⎨ y ' = 4 x + 5 y ⎩ Ответы: 1). x = c1e t + c 2 e 9 t , y = −c1e t − 9c 2 e 9 t 2). x = c1e t + c 2 e 9 t , y = −4c1e t + 4c 2 e 9 t 3). x = c1e − t + c 2 e −9 t , y = −1,5c1e − t + 3,5c 2 e −9 t 4). нет правильного ответа 5). x = c1e t + c 2 e 9 t , y = −c1e t + c 2 e 9 t Задача:

Решите

систему

Номер: 13.10.В линейных дифференциальных

уравнений:

⎧x ' = 4 x − 8 y , считая x,y функциями аргумента t ⎨ y ' = − 8 x + 4 y ⎩ Ответы: 1). нет правильного ответа 2). x = c1e −12 t + c 2 e 4 t , y = c1e −12 t + c 2 e 4 t 3). x = c1e12 t + c 2 e −4 t , y = 2c1e12 t + c 2 e −4 t 4). x = c1e12 t + c 2 e −4 t , y = −c1e12 t + c 2 e −4 t 5). x = c1e −12 t + c 2 e 4 t , y = 2c1e −12 t Задача:

Решите

систему

Номер: 13.11.В линейных дифференциальных

⎧ x ' = 6 x + 3y , считая x,y функциями аргумента t ⎨ y ' = − 8 x − 5 y ⎩ 4 Ответы: 1). x = c1e − 3t + c 2 e 2 t , y = −3c1e − 3t − c 2 e 2 t 3 5 5 2). x = c1e 3t + c 2 e − 2 t , y = − c1e3t − c 2 e − 2 t 3 3 8 3). x = c1e 3t + c 2 e − 2 t , y = −c1e3 t − c 2 e − 2 t 3 4). нет правильного ответа

5). x = c1e 3t + c 2 e 2 t , y = −c1e 3t −

4 c 2e 2t 3

138

уравнений:


Задача:

Решите

систему

Номер: 13.12.C линейных дифференциальных

⎧x ′ = 2 x − y , считая x,y функциями аргумента t ⎨ ′ y = x + 2 y ⎩ Ответы: 1). x = e t ⋅ (c1 cos t + c 2 ⋅ sin t ) ; y = e t ⋅ (c1 sin t − c 2 ⋅ cos t )

уравнений:

2). x = e 2 t ⋅ (c1 cos t + c 2 ⋅ sin t ) ; y = 2 ⋅ e 2 t ⋅ (c1 sin t − c 2 ⋅ cos t ) 3). x = e 2 t ⋅ (c1 cos t + c 2 ⋅ sin t ) ; y = e 2 t ⋅ (c1 sin t − c 2 ⋅ cos t )

4). x = e 2 t ⋅ (c1 cos t + c 2 ⋅ sin t ) ; y = e 2 t ⋅ (c1 cos t + c1 ⋅ sin t ) 5). нет правильного ответа Задача:

Решите

систему

Номер: 13.13.C линейных дифференциальных

уравнений:

Номер: 13.14.C линейных дифференциальных

уравнений:

⎧x ′ = − x + 5 y , считая x,y функциями аргумента t ⎨ ′ y = − 2 x + 5 y ⎩ 1 Ответы: 1). x = e 2 t ⋅ (c1 cos t + c 2 ⋅ sin t ) ; y = e 2 t ⋅ ((3c1 + c 2 ) cos t + (3c 2 − c1 ) ⋅ sin t ) 5 2). x = e t ⋅ (c1 cos 2 t + c 2 ⋅ sin 2 t ) ; y = e t ⋅ (2c1 sin 2 t − 2c 2 ⋅ cos 2 t ) 1 3). x = e 2 t ⋅ (c1 cos t + c 2 ⋅ sin t ) ; y = e 2 t ⋅ (3c1 cos t + 3c 2 ⋅ sin t ) 5 4). x = e 2 t ⋅ (c1 cos t + c 2 ⋅ sin t ) ; y = e 2 t ⋅ ((3c1 + c 2 ) cos t + (3c 2 − c1 ) sin t ) 5). нет правильного ответа Задача:

Решите

систему

⎧x ′ = x − 4 y , считая x,y функциями аргумента t ⎨ ′ y = x + y ⎩ 1 Ответы: 1). x = e t ⋅ (c1 cos 2 t + c 2 ⋅ sin 2 t ) ; y = e t ⋅ (c 2 cos 2 t + c1 sin 2 t ) 2 1 2). x = e t ⋅ (c1 cos 2 t + c 2 ⋅ sin 2 t ) ; y = e t ⋅ (− c 2 cos 2 t + c1 ⋅ sin 2 t ) 2 t 3). x = e ⋅ (c1 cos 2 t + c 2 ⋅ sin 2 t ) ; y = 2e t ⋅ (− c 2 cos 2 t + c1 ⋅ sin 2 t ) 4). x = e 2 t ⋅ (c1 cos t + c 2 ⋅ sin t ) ; y = 2e 2 t ⋅ (− c 2 cos t + c1 sin t ) 5). нет правильного ответа

139


Номер: 13.15.C

⎧ x ' = x − 3y , y ' 3 x y = + ⎩

Задача: Решите систему линейных дифференциальных уравнений: ⎨ считая x,y функциями аргумента t Ответы: 1). x = e t (c 2 cos 3t − c1 ⋅ sin 3t ), y = e t (c1 cos 3t + c 2 ⋅ sin 3t ) 2). x = e 2 t (c1 cos 6 t − c 2 ⋅ sin 6 t ), y = e 2 t (c1 cos 6 t + c 2 ⋅ sin 6 t )

1 t e (c 2 cos 3t + c1 ⋅ sin 3t ), y = e t (c1 cos 3t + c 2 ⋅ sin 3t ) 3 4). x = 3e t (c 2 cos 3t − c1 ⋅ sin 3t ), y = e t (c1 cos 3t + c 2 ⋅ sin 3t ) 3). x =

5). нет правильного ответа Задача:

Решите

систему

Номер: 13.16.C линейных дифференциальных

⎧x ′ = x + y , считая x,y функциями аргумента t ⎨ ′ y = − 5 x + 3 y ⎩

уравнений:

Ответы: 1). x = e 2 t (c1 cos 2 t + c 2 sin 2 t ), y = e 2 t ((c1 + 2c 2 ) ⋅ cos 2 t + (c 2 − 2c1 ) ⋅ sin 2 t ) 2). x = e 2 t (c1 cos t + c 2 sin t ), y = e 2 t ((c1 + 2c 2 ) ⋅ cos t + (c 2 − 2c1 ) ⋅ sin t )

3). x = e 2 t (c1 cos 2 t + c 2 sin 2 t ), y = e 2 t ((c1 + 2c 2 ) ⋅ cos 2 t + (c 2 + 2c1 ) ⋅ sin 2 t ) 4). x = e t (c1 cos 2 t + c 2 sin 2 t ), y = e t ((c1 − 2c 2 ) ⋅ cos 2 t + (c 2 + 2c1 ) ⋅ sin 2 t ) 5). нет правильного ответа Задача:

Решите

систему

Номер: 13.17.C линейных дифференциальных

⎧ x ′ = 3x + y , считая x,y функциями аргумента t ⎨ ′ y = − 5 x + y ⎩

уравнений:

Ответы: 1). x = e 2 t (c1 cos 2 t + c 2 sin 2 t ), y = e 2 t ((c1 + c 2 ) ⋅ cos 2 t − (2c 2 + c1 ) ⋅ sin 2 t )

2). x = e 2 t (c1 cos 2 t + c 2 sin 2 t ), y = e 2 t ((2c 2 − c1 ) ⋅ cos 2 t − (2c1 + 3c 2 ) ⋅ sin 2 t ) 3). x = e 2 t (c1 cos 2 t + c 2 sin 2 t ), y = e 2 t ((c 2 − c1 ) ⋅ cos 2 t − (c1 + c 2 ) ⋅ sin 2 t )

4). x = e t (c1 cos 2 t + c 2 sin 2 t ), y = e t ((2c 2 − c1 ) ⋅ cos 2 t − (2c1 + 3c 2 ) ⋅ sin 2 t ) 5). нет правильного ответа Задача:

Решите

систему

Номер: 13.18.C линейных дифференциальных

⎧ x ′ = 3x + 5 y , считая x,y функциями аргумента t ⎨ ⎩ y′ = − x + y 140

уравнений:


Ответы: 1). x = e 2 t (c1 cos 2 t + c 2 sin 2 t ), y = e 2 t ((2c 2 − 13c1 ) ⋅ cos 2 t − (2c1 + 13c 2 ) ⋅ sin 2 t )

2). x = e 2 t (c1 cos 2 t + c 2 sin 2 t ), y = e 2 t ((2c1 + 13c 2 ) ⋅ cos 2 t + (2c1 + 13c 2 ) ⋅ sin 2 t ) 3). x = e 2 t (c1 cos 2 t + c 2 sin 2 t ), y = 4). x = e 2 t (c1 cos t + c 2 sin t ), y = 5). нет правильного ответа

1 2t e ((2c 2 − 13c1 ) ⋅ cos 2 t − (2c1 + 13c 2 ) ⋅ sin 2 t ) 5

1 2t e ((2c 2 − 13c1 ) ⋅ cos t − (2c1 + 13c 2 ) ⋅ sin t ) 5 Номер: 13.19.C

⎧ x ′ = 3x − y , ′ y 5 x y = + ⎩

Задача: Решите систему линейных дифференциальных уравнений: ⎨

считая x,y функциями аргумента t Ответы: 1). x = e 2 t (c1 cos 2 t + c 2 sin 2 t ), y = e 2 t ((c1 + 2c 2 ) ⋅ cos 2 t − (2c1 + c 2 ) ⋅ sin 2 t ) 2). x = e 2 t (c1 cos 2 t + c 2 sin 2 t ), y = e 2 t ((c1 − 2c 2 ) ⋅ cos 2 t + (c1 + 2c 2 ) ⋅ sin 2 t ) 3). x = e 2 t (c1 cos 2 t + c 2 sin 2 t ), y = e 2 t ((c 2 − 2c1 ) ⋅ cos 2 t − (2c1 + c 2 ) ⋅ sin 2 t ) 4). x = e 2 t (c1 cos 2 t + c 2 sin 2 t ), y = e 2 t ((c1 − 2c1 ) ⋅ cos 2 t + (2c1 + c 2 ) ⋅ sin 2 t ) 5). нет правильного ответа Задача:

Решите

систему

Номер: 13.20.C линейных дифференциальных

⎧ x ′ = 3x − y , считая x,y функциями аргумента t ⎨ ′ y = 10 x + y ⎩

уравнений:

Ответы: 1). x = e 2 t (c1 cos 3t + c 2 sin 3t ), y = e 2 t ((c1 − 3c 2 ) ⋅ cos 3t + (c 2 − 3c1 ) ⋅ sin 3t )

2). x = e 2 t (c1 cos 3t + c 2 sin 3t ), y = e 2 t ((c1 + 3c 2 ) ⋅ cos 3t − (c 2 + 3c1 ) ⋅ sin 3t ) 3). x = e 2 t (c1 cos 3t + c 2 sin 3t ), y = e 2 t ((c1 − 3c 2 ) ⋅ cos 3t + (3c1 + c 2 ) ⋅ sin 3t ) 4). x = e t (c1 cos 3t + c 2 sin 3t ), y = e t ((c 2 − 3c1 ) ⋅ cos 3t + (c 2 − 3c1 ) ⋅ sin 3t ) 5). нет правильного ответа Задача:

Решите

систему

Номер: 13.21.C линейных дифференциальных

⎧ x ′ = 3x + y , считая x,y функциями аргумента t ⎨ ′ y = − 10 x + y ⎩

уравнений:

Ответы: 1). x = e 2 t (c1 cos 3t + c 2 sin 3t ), y = e 2 t ((c1 − 3c 2 ) ⋅ cos 3t + (3c1 + c 2 ) ⋅ sin 3t ) 141


2). x = e 2 t (c1 cos 3t + c 2 sin 3t ), y = e 2 t ((3c 2 − c1 ) ⋅ cos 3t − (3c1 + c 2 ) ⋅ sin 3t )

3). x = e 2 t (c1 cos 3t + c 2 sin 3t ), y = e 2 t ((3c1 − c 2 ) ⋅ cos 3t − (3c1 + c 2 ) ⋅ sin 3t )

4). x = e 2 t (c1 cos 3t + c 2 sin 3t ), y = e 2 t ((c1 − 3c 2 ) ⋅ cos 3t − (3c1 + 3c 2 ) ⋅ sin 3t ) 5). нет правильного ответа Задача:

Решите

систему

Номер: 13.22.C линейных дифференциальных

⎧x ′ = x − y , считая x,y функциями аргумента t ⎨ ′ y = 10 x + 3 y ⎩

уравнений:

Ответы: 1). x = e 2 t (c1 cos 3t + c 2 sin 3t ), y = e 2 t ((c1 + 3c 2 ) ⋅ cos 3t − (3c1 − c 2 ) ⋅ sin 3t )

2). x = e 2 t (c1 cos 3t + c 2 sin 3t ), y = e 2 t ((− c1 − 3c 2 ) ⋅ cos 3t + (3c1 + c 2 ) ⋅ sin 3t ) 3). x = e 2 t (c1 cos 3t + c 2 sin 3t ), y = e 2 t (− (c1 + 3c 2 ) ⋅ cos 3t + (3c1 − c 2 ) ⋅ sin 3t )

4). x = e 2 t (c1 cos 3t + c 2 sin 3t ), y = e 2 t ((c1 + 3c 2 ) ⋅ cos 3t + (3c1 − c 2 ) ⋅ sin 3t ) 5). нет правильного ответа Задача:

Решите

систему

Номер: 13.23.C линейных дифференциальных

⎧x ′ = x + y , считая x,y функциями аргумента t ⎨ ′ y = − 10 x + 3 y ⎩

уравнений:

Ответы: 1). x = e 2 t (c1 cos 3t + c 2 sin 3t ), y = e 2 t ((c1 + 3c 2 ) ⋅ cos 3t + (3c 2 − c1 ) ⋅ sin 3t ) 2). x = e 2 t (c1 cos 3t + c 2 sin 3t ), y = e 2 t ((c1 − 3c 2 ) ⋅ cos 3t + (3c1 − c 2 ) ⋅ sin 3t )

3). x = e 2 t (c1 cos 3t + c 2 sin 3t ), y = e 2 t ((c 2 + 3c1 ) ⋅ cos 3t + (c 2 − 3c1 ) ⋅ sin 3t ) 4). x = e 2 t (c1 cos 3t + c 2 sin 3t ), y = e 2 t ((c1 + 3c 2 ) ⋅ cos 3t + (c 2 − 3c1 ) ⋅ sin 3t ) 5). нет правильного ответа Задача:

Решите

систему

Номер: 13.24.C линейных дифференциальных

⎧x ′ = 4x − y , считая x,y функциями аргумента t ⎨ ′ y = 2 x + 2 y ⎩

уравнений:

Ответы: 1). нет правильного ответа 2). x = e 3 t (c1 cos t + c 2 sin t ), y = e 3t ((7c1 − c 2 ) ⋅ cos t + (c1 + c 2 ) ⋅ sin t ) 3). x = e 3 t (c1 cos t + c 2 sin t ), y = e 3t ((c1 + c 2 ) ⋅ cos t + (c1 − c 2 ) ⋅ sin t )

4). x = e 3t (c1 cos t + c 2 sin t ), y = e 3t ((7c1 + c 2 ) ⋅ cos t + (7c 2 − c1 ) ⋅ sin t ) 5). x = e 3t (c1 cos t + c 2 sin t ), y = e 3t ((c1 − c 2 ) ⋅ cos t + (c1 + c 2 ) ⋅ sin t ) 142


Номер: 13.25.C

⎧ x ′ = 5x − y , ′ y 5 x y = + ⎩

Задача: Решите систему линейных дифференциальных уравнений: ⎨

считая x,y функциями аргумента t Ответы: 1). x = e3 t (c1 cos t + c 2 sin t ), y = e 3t ((2c1 − c 2 ) ⋅ cos t + (c1 + 2c 2 ) ⋅ sin t ) 2). x = e 3t (c1 cos t + c 2 sin t ), y = e 3t ((2c1 + c 2 ) ⋅ cos t + (2c1 + c 2 ) ⋅ sin t )

3). x = e 3t (c1 cos t + c 2 sin t ), y = e 3t ((c1 − 2c 2 ) ⋅ cos t + (c1 + 2c 2 ) ⋅ sin t )

4). x = e 3t (c1 cos t + c 2 sin t ), y = e 3t ((c1 + 2c 2 ) ⋅ cos 3t + (2c1 − c 2 ) ⋅ sin t ) 5). нет правильного ответа Задача:

Решите

систему

Номер: 13.26.C линейных дифференциальных

уравнений:

⎧x ′ = 2 x + y , считая x,y функциями аргумента t ⎨ ′ y = − 2 x + 4 y ⎩ Ответы: 1). x = e 3 t (c1 cos t + c 2 sin t ), y = e 3t ((c1 + c 2 ) ⋅ cos t + (c1 − c 2 ) ⋅ sin t ) 2). x = e 3t (c1 cos t + c 2 sin t ), y = e 3t (− (c1 + c 2 ) ⋅ cos t + (c1 − c 2 ) ⋅ sin t )

3). x = e 3t (c1 cos t + c 2 sin t ), y = e 3t (− (c1 + c 2 ) ⋅ cos t + (c 2 − c1 ) ⋅ sin t )

4). x = e 3t (c1 cos t + c 2 sin t ), y = e 3t ((3c1 − c 2 ) ⋅ cos t + (3c 2 − c1 ) ⋅ sin t ) 5). нет правильного ответа Задача:

Решите

систему

Номер: 13.27.C линейных дифференциальных

⎧ x ′ = 5x + y , считая x,y функциями аргумента t ⎨ ′ y = − 5 x + y ⎩

уравнений:

Ответы: 1). x = e t (c1 cos 3t + c 2 sin 3t ), y = e t ((2c1 − c 2 ) ⋅ cos 3t + (c1 + 2c 2 ) ⋅ sin 3t ) 2). x = e 3t (c1 cos t + c 2 sin t ), y = e 3t ((c1 − 3c 2 ) ⋅ cos t + (c1 + 3c 2 ) ⋅ sin t )

3). x = e 3t (c1 cos t + c 2 sin t ), y = e 3t ((− 2c1 + c 2 ) ⋅ cos t − (c1 + 2c 2 ) ⋅ sin t ) 4). x = e 3t (c1 cos t + c 2 sin t ), y = e 3t ((2c1 + c 2 ) ⋅ cos t − (c1 + 2c 2 ) ⋅ sin t ) 5). нет правильного ответа Номер: 13.28.C Задача: Решите систему линейных дифференциальных уравнений:

⎧x ′ = 4 x + y , считая x,y функциями аргумента t ⎨ ′ y = − 2 x + 2 y ⎩

Ответы: 1). x = e t (c1 cos 3t + c 2 sin 3t ), y = e t ((c1 − 3c 2 ) ⋅ cos 3t + (c 2 − 3c1 ) ⋅ sin 3t ) 143


2). x = e t (c1 cos 3t + c 2 sin 3t ), y = e t ((c1 + 3c 2 ) ⋅ cos 3t + (c 2 − 3c1 ) ⋅ sin 3t ) 3). x = e 3t (c1 cos t + c 2 sin t ), y = e 3t ((c1 + c 2 ) ⋅ cos t − (c1 − c 2 ) ⋅ sin t )

4). x = e 3t (c1 cos t + c 2 sin t ), y = e 3t ((c 2 − c1 ) ⋅ cos t − (c1 + c 2 ) ⋅ sin t ) 5). нет правильного ответа Задача:

Решите

систему

Номер: 13.29.C линейных дифференциальных

⎧ x ′ = 3x + y , считая x,y функциями аргумента t ⎨ ′ y = − x + 3 y ⎩

уравнений:

Ответы: 1). нет правильного ответа 2). x = e 3 t (c1 cos t + c 2 sin t ), y = e 3t ((c 2 − c1 ) ⋅ cos t + (c1 + c 2 ) ⋅ sin t )

3). x = e 3t (c1 cos t + c 2 sin t ), y = e 3t ((c1 + c 2 ) ⋅ cos t − (c1 + c 2 ) ⋅ sin t )

4). x = e3 t (c1 cos t + c 2 sin t ), y = e 3t (− c 2 cos t + c1 sin t ) 5). x = e 3t (c1 cos t + c 2 sin t ), y = e 3t (c 2 cos t − c1 sin t )

Задача:

Решите

систему

Номер: 13.30.C линейных дифференциальных

уравнений:

⎧x ′ = 6 x − y , считая x,y функциями аргумента t ⎨ ′ y = 13 x ⎩ Ответы: 1) x = e 3t ⋅ (c1 cos 2 t + c 2 sin 2 t ), y = e 3t (3c1 cos 2 t + 2c 2 sin 2 t )

2). x = e3t ⋅ (c1 cos 2 t + c 2 sin 2 t ), y = e 3t ((3c1 + 2c 2 ) cos 2 t + (2c1 − 3c 2 ) ⋅ sin 2 t ) 3). x = e3t ⋅ (c1 cos 2 t + c 2 sin 2 t ), y = e 3t ((3c1 − 2c 2 ) cos 2 t + (2c1 + 3c 2 ) ⋅ sin 2 t )

4). x = e3t ⋅ (c1 cos 2 t + c 2 sin 2 t ), y = e 3t ((3c 2 − 2c1 ) cos 2 t + (2c1 + 3c 2 ) ⋅ sin 2 t ) 5). нет правильного ответа Задача:

Решите

систему

Номер: 13.31.C линейных дифференциальных

⎧ x ′ = 3x + y , считая x,y функциями аргумента t ⎨ ′ y = − 4 x + 3 y ⎩

уравнений:

Ответы: 1). нет правильного ответа 2). x = e3 t ⋅ (c1 cos 2 t + c 2 sin 2 t ), y = e 3t ((c1 − 2c 2 ) cos 2 t + (2c1 − c 2 ) ⋅ sin 2 t )

3). x = e 3t ⋅ (c1 cos 2 t + c 2 sin 2 t ), y = e 3t ⋅ (2c 2 cos 2 t + 2c1 sin 2 t )

4). x = e3t ⋅ (c1 cos 2 t + c 2 sin 2 t ), y = e3t ⋅ (2c 2 cos 2 t − 2c1 sin 2 t )

5). x = e 3t ⋅ (c1 cos 2 t + c 2 sin 2 t ), y = e 3t ⋅ (2c1 cos 2 t + 2c 2 sin 2 t )

144


Задача:

Решите

систему

Номер: 13.32.C линейных дифференциальных

уравнений:

⎧ x ′ = 3x − y , считая x,y функциями аргумента t ⎨ ′ y = 4 x + 3 y ⎩

Ответы: 1). нет правильного ответа 2). x = e3 t ⋅ (c1 cos 2 t + c 2 sin 2 t ), y = e 3t ((c1 − 2c 2 ) cos 2 t + (2c1 − c 2 ) ⋅ sin 2 t ) 3). x = e 3t ⋅ (c1 cos 2 t + c 2 sin 2 t ), y = e 3t ⋅ (2c 2 cos 2 t + 2c1 sin 2 t )

4). x = e3t ⋅ (c1 cos 2 t + c 2 sin 2 t ), y = e3t ⋅ (2c 2 cos 2 t − 2c1 sin 2 t )

5). x = e 3t ⋅ (c1 cos 2 t + c 2 sin 2 t ), y = e 3t ⋅ (− 2c 2 cos 2 t + 2c1 sin 2 t ) Задача:

Решите

систему

Номер: 13.33.C линейных дифференциальных

уравнений:

⎧ x ′ = 5x + y , считая x,y функциями аргумента t ⎨ ′ y = − 8 x + y ⎩

Ответы: 1). x = e3t ⋅ (c1 cos 2 t + c 2 sin 2 t ), y = e 3t ((− 2c1 + 2c 2 ) cos 2 t + (− 2c1 − 2c 2 ) ⋅ sin 2 t ) 2). x = e3t ⋅ (c1 cos 2 t + c 2 sin 2 t ), y = e 3t ⋅ (2c 2 cos 2 t + 2c1 sin 2 t )

3). x = e3 t ⋅ (c1 cos 2 t + c 2 sin 2 t ), y = e 3t ((c1 + 2c 2 ) cos 2 t − (c1 + 2c 2 ) ⋅ sin 2 t )

4). x = e3 t ⋅ (c1 cos 2 t + c 2 sin 2 t ), y = e 3t ((2c1 + 2c 2 ) cos 2 t − (2c1 + 2c 2 ) ⋅ sin 2 t ) 5). нет правильного ответа Номер: 13.34.C

⎧ x ′ = 5x − y , ′ y 8 x y = + ⎩

Задача: Решите систему линейных дифференциальных уравнений: ⎨

считая x,y функциями аргумента t Ответы: 1). x = e3 t (c1 cos 2 t + c 2 sin 2 t ), y = e3 t ((2c1 + 2c 2 ) ⋅ cos 2 t + (2c1 + 2c 2 ) ⋅ sin 2 t )

2). x = e 3t (c1 cos 2 t + c 2 sin 2 t ), y = e 3t ((2c1 − 2c 2 ) ⋅ cos 2 t + (2c1 + 2c 2 ) ⋅ sin 2 t ) 3). x = e 3 t (c1 cos 2 t + c 2 sin 2 t ), y = e 3t (c1 − 2c 2 ) cos 2 t + (c1 + 2c 2 ) sin 2 t 4). x = e 3 t (c1 cos 2 t + c 2 sin 2 t ), y = e 3t (c1 + 2c 2 ) cos 2 t + (c1 − 2c 2 ) sin 2 t 5). нет правильного ответа Задача:

Решите

систему

Номер: 13.35.C линейных дифференциальных

⎧ x ′ = 5x + y , считая x,y функциями аргумента t ⎨ ′ y = − 2 x + 3 y ⎩ 145

уравнений:


Ответы: 1). x = e −4 t (c1 cos t + c 2 sin t ), y = e −4 t ((− c1 + c 2 ) ⋅ cos t − (c1 + c 2 ) ⋅ sin t )

2). x = e −4 t (c1 cos t + c 2 sin t ), y = e −4 t ((c1 − c 2 ) ⋅ cos t + (c1 + c 2 ) ⋅ sin t )

3). x = e 4 t (c1 cos t + c 2 sin t ), y = e 4 t ((− c1 + c 2 ) cos t − (c1 + c 2 ) sin t )

4). x = e 4 t (c1 cos t + c 2 sin t ), y = e 4 t ((c1 − c 2 ) cos t + (c1 + c 2 ) sin 2 t ) 5). нет правильного ответа

146


Issuu converts static files into: digital portfolios, online yearbooks, online catalogs, digital photo albums and more. Sign up and create your flipbook.